Nursing Final Test Question Examples Flashcards

1
Q

The nurse prescribes strategies and alternatives to attain the expected outcome. Which standard of
nursing practice is the nurse following?
a. Assessment
b. Diagnosis
c. Planning
d. Implementation

A

ANS: C
In planning, the registered nurse develops a plan that prescribes strategies and alternatives to attain
expected outcomes. During the assessment, the registered nurse collects comprehensive data pertinent to
the patient’s health and/or the situation. In diagnosis, the registered nurse analyzes the assessment
data to determine the diagnoses or issues. During implementation, the registered nurse implements
(carries out) the identified plan.

How well did you know this?
1
Not at all
2
3
4
5
Perfectly
2
Q
  1. An experienced medical-surgical nurse chooses to work in obstetrics. Which level of proficiency is
    the nurse upon initial transition to the obstetrical floor?
    a. Novice
    b. Proficient
    c. Competent
    d. Advanced beginner
A

ANS: A
A beginning nursing student or any nurse entering a situation in which there is no previous level of
experience (e.g., an experienced operating room nurse chooses to now practice in home health) is an
example of a novice nurse. A proficient nurse perceives a patient’s clinical situation as a whole, is
able to assess an entire situation, and can readily transfer knowledge gained from multiple previous
experiences to a situation. A competent nurse understands the organization and specific care required
by the type of patients (e.g., surgical, oncology, or orthopedic patients). This nurse is a competent
practitioner who is able to anticipate nursing care and establish long-range goals. A nurse who has
had some level of experience with the situation is an advanced beginner. This experience may only
be observational in nature, but the nurse is able to identify meaningful aspects or principles of
nursing care.

How well did you know this?
1
Not at all
2
3
4
5
Perfectly
3
Q

A nurse assesses a patient’s fluid status and decides that the patient needs to drink more fluids. The
nurse then encourages the patient to drink more fluids. Which concept is the nurse demonstrating?
a. Licensure
b. Autonomy
c. Certification
d. Accountability

A

ANS: B
Autonomy is an essential element of professional nursing that involves the initiation of independent
nursing interventions without medical orders. To obtain licensure in the United States, the RN
candidate must pass the NCLEX-RN®. Beyond the NCLEX-RN®, the nurse may choose to work
toward certification in a specific area of nursing practice. Accountability means that you are
responsible, professionally and legally, for the type and quality of nursing care provided.

How well did you know this?
1
Not at all
2
3
4
5
Perfectly
4
Q
. A nurse prepares the budget and policies for an intensive care unit. Which role is the nurse
 implementing?
a. Educator
b. Manager
c. Advocate
d. Caregiver
A

ANS: B
A manager coordinates the activities of members of the nursing staff in delivering nursing care and
has personnel, policy, and budgetary responsibility for a specific nursing unit or facility. As an
educator, you explain concepts and facts about health, describe the reason for routine care activities,
demonstrate procedures such as self-care activities, reinforce learning or patient behavior, and
evaluate the patient’s progress in learning. As a patient advocate, you protect your patient’s human
and legal rights and provide assistance in asserting these rights if the need arises. As a caregiver, you
help patients maintain and regain health, manage disease and symptoms, and attain a maximal level
function and independence through the healing process.

How well did you know this?
1
Not at all
2
3
4
5
Perfectly
5
Q

A nurse teaches a group of nursing students about nurse practice acts. Which information
is most important to include in the teaching session about nurse practice acts?
a. Protects the nurse
b. Protects the public
c. Protects the provider
d. Protects the hospital

A

ANS: B
The nurse practice acts regulate the scope of nursing practice and protect public health, safety, and
welfare. They do not protect the nurse, provider, or hospital.

How well did you know this?
1
Not at all
2
3
4
5
Perfectly
6
Q

A nurse is using a guide that provides principles of right and wrong to provide care to patients.
Which guide is the nurse using?
a. Code of ethics
b. Standards of practice
c. Standards of professional performance
d. Quality and safety education for nurses

A

ANS: A
The code of ethics is the philosophical ideals of right and wrong that define the principles you will
use to provide care to your patients. The Standards of Practice describe a competent level of nursing
care. The ANA Standards of Professional Performance describe a competent level of behavior in the
professional role. Quality and safety education for nurses addresses the challenge to prepare nurses
with the competencies needed to continuously improve the quality of care in their work
environments.

How well did you know this?
1
Not at all
2
3
4
5
Perfectly
7
Q

A nurse is preparing a teaching session about contemporary influences on nursing. Which
examples should the nurse include? (Select all that apply.)
a. Human rights
b. Affordable Care Act
c. Demographic changes
d. Medically underserved
e. Decreasing health care costs

A

ANS: A, B, C, D
Multiple external forces affect nursing, including the need for nurses’ self-care, Affordable Care Act
(ACA) and rising (not decreasing) health care costs, demographic changes of the population, human
rights, and increasing numbers of medically underserved.

How well did you know this?
1
Not at all
2
3
4
5
Perfectly
8
Q
. A nurse wants to become an advanced practice registered nurse. Which options should the nurse
 consider? (Select all that apply.)
a. Patient advocate
b. Nurse administrator
c. Certified nurse-midwife
d. Clinical nurse specialist
e. Certified nurse practitioner
A

ANS: C, D, E
Although all nurses should function as patient advocates, “advanced practice nurse” is an umbrella
term for an advanced clinical nurse such as a certified nurse practitioner, clinical nurse specialist,
certified registered nurse anesthetist, or certified nurse-midwife. A nurse administrator is not an
example of advanced practice.

How well did you know this?
1
Not at all
2
3
4
5
Perfectly
9
Q

A nurse is teaching the staff about managed care. Which information should the nurse include in
the teaching session?
a. Managed care insures full coverage of health care costs.
b. Managed care only assumes the financial risk involved.
c. Managed care allows providers to focus on illness care.
d. Managed care causes providers to focus on prevention.

A

ANS: D
Managed care describes health care systems in which the provider or the health care system receives
a predetermined capitated (fixed amount) payment for each patient enrolled in the program.
Therefore, the focus of care shifts from individual illness care to prevention, early intervention, and
outpatient care. The actual cost of care is the responsibility of the provider. The managed care
organization (provider) assumes financial risk, in addition to providing patient care.

How well did you know this?
1
Not at all
2
3
4
5
Perfectly
10
Q

The nurse is applying for a position with a home care organization that specializes in spinal cord

injury. In which type of health care facility does the nurse want to work?
a. Secondary acute
b. Continuing
c. Restorative
d. Tertiary

A

ANS: C
Patients recovering from an acute or chronic illness or disability often require additional services
(restorative care) to return to their previous level of function or reach a new level of function limited
by their illness or disability. Restorative care includes cardiovascular and pulmonary rehabilitation,
sports medicine, spinal cord injury programs, and home care. Secondary acute care involves
emergency care, acute medical-surgical care, and radiological procedures. Continuing care involves
assisted living, psychiatric care, and older-adult day care. Tertiary care includes intensive care and
subacute care.

How well did you know this?
1
Not at all
2
3
4
5
Perfectly
11
Q

A nurse provides immunization to children and adults through the public healthdepartment.
Which type of health care is the nurse providing?
a. Primary care
b. Preventive care
c. Restorative care
d. Continuing care

A

ANS: B
Preventive care includes immunizations, screenings, counseling, crisis prevention, and community
safety legislation. Primary care is health promotion that includes prenatal and well-baby care,
nutrition counseling, family planning, and exercise classes. Restorative care includes rehabilitation,
sports medicine, spinal cord injury programs, and home care. Continuing care is assisted living and
psychiatric care and older-adult day care

How well did you know this?
1
Not at all
2
3
4
5
Perfectly
12
Q

An older adult patient has extensive wound care needs after discharge from the hospital.Which
facility should the nurse discuss with the patient?
a. Hospice
b. Respite care
c. Assisted living
d. Skilled nursing

A

ANS: D
An intermediate care or skilled nursing facility offers skilled care from a licensed nursing staff. This
often includes administration of IV fluids, wound care, long-term ventilator management, and
physical rehabilitation. A hospice is a system of family-centered care that allows patients to live with
comfort, independence, and dignity while easing the pains of terminal illness. Respite care is a
service that provides short-term relief or “time off” for people providing home care to an individual
who is ill, disabled, or frail. Assisted living offers an attractive long-term care setting with an
environment more like home and greater resident autonomy

How well did you know this?
1
Not at all
2
3
4
5
Perfectly
13
Q

A nurse working in a community hospital’s emergency department provides care to a patient
having chest pain. Which level of care is the nurse providing?
a. Continuing care
b. Restorative care
c. Preventive care
d. Tertiary care

A

ANS: D
Hospital emergency departments, urgent care centers, critical care units, and inpatient medicalsurgical units provide secondary and tertiary levels of care. Patients recovering from an acute or
chronic illness or disability often require additional services (restorative care) to return to their
previous level of function or reach a new level of function limited by their illness or disability.
Continuing care is available within institutional settings (e.g., nursing centers or nursing homes,
group homes, and retirement communities), communities (e.g., adult day care and senior centers), or
the home (e.g., home care, home-delivered meals, and hospice). Preventive care is more disease
oriented and focused on reducing and controlling risk factors for disease through activities such as
immunization and occupational health programs.

How well did you know this?
1
Not at all
2
3
4
5
Perfectly
14
Q

A nurse is teaching the staff about the Institute of Medicine competencies. Which examples
indicate the staff has a correct understanding of the teaching? (Select all that apply.)
a. Use informatics.
b. Use transparency.
c. Apply globalization.
d. Apply quality improvement.
e. Use evidence-based practice.

A

ANS: A, D, E
The Institute of Medicine competencies include: Provide patient-centered care; work in
interdisciplinary teams; use evidence-based practice; apply quality improvement; and use
informatics. Transparency is included in the 10 rules of performance in a redesigned health care
system, not a competency. While globalization is important in health care, it is not a competency

How well did you know this?
1
Not at all
2
3
4
5
Perfectly
15
Q

A nurse uses evidence-based practice (EBP) to provide nursing care. What is the best rationale for
the nurse’s behavior?
a. EBP is a guide for nurses in making clinical decisions.
b. EBP is based on the latest textbook information.
c. EBP is easily attained at the bedside.
d. EBP is always right for all situations.

A

ANS: A
Evidence-based practice (EBP) is a guide for nurses to structure how to make appropriate, timely,
and effective clinical decisions. A textbook relies on the scientific literature, which may be outdated
by the time the book is published. Unfortunately, much of the best evidence never reaches the
bedside. EBP is not to be blindly applied without using good judgment and critical thinking skills.

How well did you know this?
1
Not at all
2
3
4
5
Perfectly
16
Q

In caring for patients, what must the nurse remember about evidence-based practice (EBP)?

a. EBP is the only valid source of knowledge that should be used.
b. EBP is secondary to traditional or convenient care knowledge.
c. EBP is dependent on patient values and expectations.
d. EBP is not shown to provide better patient outcomes.

A

ANS: C
Even when the best evidence available is used, application and outcomes will differ based on patient
values, preferences, concerns, and/or expectations. Nurses often care for patients on the basis of
tradition or convenience. Although these sources have value, it is important to learn to rely more on
research evidence than on nonresearch evidence. Evidence-based care improves quality, safety,
patient outcomes, and nurse satisfaction while reducing costs.

How well did you know this?
1
Not at all
2
3
4
5
Perfectly
17
Q
A patient has a head injury and damages the hypothalamus. Which vital sign will the nurse
monitor most closely?
a. Pulse
b. Respirations
c. Temperature
d. Blood pressure
A

ANS: C
Disease or trauma to the hypothalamus or the spinal cord, which carries hypothalamic messages,
causes serious alterations in temperature control. The hypothalamus does not control pulse,
respirations, or blood pressure.

How well did you know this?
1
Not at all
2
3
4
5
Perfectly
18
Q

The patient with heart failure is restless with a temperature of 102.2° F (39° C). Which action will
the nurse take?
a. Place the patient on oxygen.
b. Encourage the patient to cough.
c. Restrict the patient’s fluid intake.
d. Increase the patient’s metabolic rate.

A

ANS: A
Interventions during a fever include oxygen therapy. During a fever, cellular metabolism increases
and oxygen consumption rises. Myocardial hypoxia produces angina. Cerebral hypoxia produces
confusion. Dehydration is a serious problem through increased respiration and diaphoresis. The
patient is at risk for fluid volume deficit. Fluids should not be restricted, even though the patient has
heart failure; the patient needs fluids at this time due to the fever. Increasing the metabolic rate
further would not be advisable. Coughing will increase muscular activity, which will increase fever.

How well did you know this?
1
Not at all
2
3
4
5
Perfectly
19
Q

The nurse is caring for an infant and is obtaining the patient’s vital signs. Which artery will the
nurse use to bestobtain the infant’s pulse?
a. Radial
b. Brachial
c. Femoral
d. Popliteal

A

ANS: B
The brachial or apical pulse is the best site for assessing an infant’s or a young child’s pulse because
other peripheral pulses such as the radial, femoral, and popliteal arteries are deep and difficult to
palpate accurately.

How well did you know this?
1
Not at all
2
3
4
5
Perfectly
20
Q

The patient is found to be unresponsive and not breathing. Which pulse site will the nurse use?

a. Radial
b. Apical
c. Carotid
d. Brachial

A

ANS: C
The heart continues to deliver blood through the carotid artery to the brain as long as possible. The
carotid pulse is easily accessible during physiological shock or cardiac arrest. The radial pulse is used
to assess peripheral circulation or to assess the status of circulation to the hand. The brachial site is
used to assess the status of circulation to the lower arm. The apical pulse is used to auscultate the
apical area.

How well did you know this?
1
Not at all
2
3
4
5
Perfectly
21
Q

The nurse needs to obtain a radial pulse from a patient. What must the nurse do to obtain a
correct measurement?
a. Place the tips of the first two fingers over the groove along the thumb side of the patient’s wrist.
b. Place the tips of the first two fingers over the groove along the little finger side of the patient’s wrist.
c. Place the thumb over the groove along the little finger side of the patient’s wrist.
d. Place the thumb over the groove along the thumb side of the patient’s wrist

A

ANS: A
Place the tips of the first two or middle three fingers of the hand over the groove along the radial or
thumb side of the patient’s inner wrist. Fingertips are the most sensitive parts of the hand to palpate
arterial pulsation. The thumb has a pulsation that interferes with accuracy. The groove along the little
finger is the ulnar pulse.

How well did you know this?
1
Not at all
2
3
4
5
Perfectly
22
Q

The nurse is assessing the patient’s respirations. Which action by the nurse is most appropriate?

a. Inform the patient that she is counting respirations.
b. Do not touch the patient until completed.
c. Obtain without the patient knowing.
d. Estimate respirations

A

ANS: C
Do not let a patient know that you are assessing respirations. A patient aware of the assessment can
alter the rate and depth of breathing. Assess respirations immediately after measuring pulse rate, with
your hand still on the patient’s wrist as it rests over the chest or abdomen. Respirations are the easiest
of all vital signs to assess, but they are often the most haphazardly measured. Do not estimate
respirations.

How well did you know this?
1
Not at all
2
3
4
5
Perfectly
23
Q

The patient is being admitted to the emergency department with reports of shortness of breath.
The patient has had chronic lung disease for many years but still smokes. What will the nursedo?
a. Allow the patient to breathe into a paper bag.
b. Use oxygen cautiously in this patient.
c. Administer high levels of oxygen.
d. Give CO2 via mask

A

ANS: B
Oxygen must be used cautiously in these types of patients. Hypoxemia helps to control ventilation in
patients with chronic lung disease. Because low levels of arterial O2 provide the stimulus that allows
a patient to breathe, administration of high oxygen levels may be fatal for patients with chronic lung
disease. Patients with chronic lung disease have ongoing hypercarbia (elevated CO2 levels) and do
not need to have CO2 administered or “rebreathed” with a paper bag.

How well did you know this?
1
Not at all
2
3
4
5
Perfectly
24
Q

The patient was found unresponsive in an apartment and is being brought to the emergency
department. The patient has arm, hand, and leg edema, temperature is 95.6° F, and hands are cold
secondary to a history of peripheral vascular disease. It is reported that the patient has a latex allergy.
What should the nurse do to quickly measure the patient’s oxygen saturation?
a. Attach a finger probe to the patient’s index finger.
b. Place a nonadhesive sensor on the patient’s earlobe.
c. Attach a disposable adhesive sensor to the bridge of the patient’s nose.
d. Place the sensor on the same arm that the electronic blood pressure cuff is on.

A

ANS: B
A nonadhesive sensor is best for latex allergy, and the earlobe site is the best choice for this patient
with peripheral vascular disease and edema. Select forehead, ear or bridge of nose if an adult patient
has a history of peripheral vascular disease. Do not attach probe to finger, ear, forehead, or bridge of
nose if area is edematous or skin integrity is compromised. Do not use disposable adhesive probes if
the patient has latex allergy. Do not attach probe to fingers that are hypothermic. Do not place the
sensor on the same extremity as the electronic blood pressure cuff because blood flow to the finger
will be temporarily interrupted when the cuff inflates.

How well did you know this?
1
Not at all
2
3
4
5
Perfectly
25
Q

When taking the pulse of an infant, the nurse notices that the rate is 145 beats/min and the rhythm
is regular. How should the nurse interpret thisfinding?
a. This is normal for an infant.
b. This is too fast for an infant.
c. This is too slow for an infant.
d. This is not a rate for an infant but for a toddler.

A

ANS: A
The normal rate for an infant is 120 to 160 beats/min. The rate obtained (145 beats/min) is within the
normal range for an infant. The normal rate for a toddler is between 90 and 140 beats/min; 145 is too
high for a toddler.

How well did you know this?
1
Not at all
2
3
4
5
Perfectly
26
Q

The nurse is caring for a small child and needs to obtain vital signs. Which site choice from the nursing assistive personnel (NAP) will cause the nurse to praise the NAP?

a. Ulnar site
b. Radial site
c. Brachial site
d. Femoral site

A

ANS: C
The nurse will praise the NAP when obtaining the pulse from the brachial site. The brachial or apical
pulse is the best site for assessing an infant’s or a young child’s pulse because other peripheral pulses
are deep and difficult to palpate accurately

How well did you know this?
1
Not at all
2
3
4
5
Perfectly
27
Q

The nurse is preparing to obtain an oxygen saturation reading on a toddler. Which action will the
nurse take?
a. Secure the sensor to the toddler’s earlobe.
b. Determine whether the toddler has a latex allergy.
c. Place the sensor on the bridge of the toddler’s nose.
d. Overlook variations between an oximeter pulse rate and the toddler’s pulse rate.

A

ANS: B
The nurse should determine whether the patient has latex allergy because disposable adhesive probes
should not be used on patients with latex allergies. Earlobe and bridge of the nose sensors should not
be used on infants and toddlers because of skin fragility. Oximeter pulse rate and the patient’s apical
pulse rate should be the same. Any difference requires re-evaluation of oximeter sensor probe
placement and reassessment of pulse rates

How well did you know this?
1
Not at all
2
3
4
5
Perfectly
28
Q

A nurse is caring for a group of patients. Which patient will the nurse see first?

a. A crying infant with P-165 and R-54
b. A sleeping toddler with P-88 and R-23
c. A calm adolescent with P-95 and R-26
d. An exercising adult with P-108 and R-24

A

ANS: C
A calm adolescent should have the following findings: P—60-90 and R—16-20. Since both findings
are elevated, the nurse should see this patient first. An infant should have the following findings: P—
120-160 and R—30-50; however, since the infant is crying these values will be elevated and this is
normal. A toddler should have the following findings: P—90-140 and R—25-32; however, since the
toddler is sleeping these values can be slightly decreased and this is normal. An adult should havethe
following findings: P—60-100 and R—12-20; however, since the adult is exercising these values
will be elevated and this is normal.

How well did you know this?
1
Not at all
2
3
4
5
Perfectly
29
Q

The nurse is caring for a patient who reports feeling light-headed and “woozy.” The nurse checks
the patient’s pulse and finds that it is irregular. The patient’s blood pressure is 100/72. It was113/80
an hour earlier. What should the nurse do?
a. Apply more pressure to the radial artery to feel pulse.
b. Perform an apical/radial pulse assessment.
c. Call the health care provider immediately.
d. Obtain arterial blood gases.

A

ANS: B
If the pulse is irregular, do an apical/radial pulse assessment to detect a pulse deficit. If pulse count
differs by more than 2, a pulse deficit exists, which sometimes indicates alterations in cardiac output.
The nurse needs to gather as much information as possible before calling the health care provider.
The radial pulse is more accurately assessed with moderate pressure. Too much pressure occludes the
pulse and impairs blood flow. Arterial blood gases is a laboratory test that measures blood pH and
oxygenation status. Arterial blood gases would be appropriate if respirations were abnormal or if
pulse oximetry results were severely low.

How well did you know this?
1
Not at all
2
3
4
5
Perfectly
30
Q

A nurse is caring for a group of patients. Which patient will the nurse see first?

a. A 17-year-old male who has just returned from outside “for a smoke” who needs a temperature taken
b. A 20-year-old male postoperative patient whose blood pressure went from 128/70 to 100/60
c. A 27-year-old male patient reporting pain whose blood pressure went from 124/70 to 130/74
d. An 87-year-old male suspected of hypothermia whose temperature is below normal

A

ANS: B
When a blood pressure drops in a postoperative patient, bleeding may be occurring and lead to shock.
The nurse should assess this patient first. Pain will cause the blood pressure to elevate so this is an
expected finding, and while it does need to be assessed, it is not the first one to assess. A teenager
who has returned from smoking will have to wait at least 20 minutes before a temperature can be
taken, so this is not the first one to see. A patient with hypothermia is expected to have a temperature
below normal, so this is not the first one to see.

How well did you know this?
1
Not at all
2
3
4
5
Perfectly
31
Q

The patient has new-onset restlessness and confusion. Pulse rate is elevated, as is respiratoryrate.
Oxygen saturation is 94%. The nurse ignores the pulse oximeter reading and calls the health care
provider for orders because the pulse oximetry reading is inaccurate. Which factors can cause
inaccurate pulse oximetry readings? (Select all that apply.)
a.
O2 saturations (SaO2) > 70%
b. Carbon monoxide inhalation
c. Hypothermic fingers
d. Intravascular dyes
e. Nail polish
f. Jaundice

A

ANS: B, C, D, E, F
Inaccurate pulse oximetry readings can be caused by outside light sources, carbon monoxide (caused
by smoke inhalation or poisoning), patient motion, jaundice, intravascular dyes (methylene blue),
nail polish, artificial nails, metal studs, or dark skin. SpO2 is a reliable estimate of SaO2 when the
SaO2 is over 70%.

How well did you know this?
1
Not at all
2
3
4
5
Perfectly
32
Q

A head and neck physical examination is completed on a 50-year-old female patient. All physical findings are normal except for fine brittle hair. Which laboratory test will the nurse expect to be ordered, based upon the physical findings?

a. Oxygen saturation
b. Liver function test
c. Carbon monoxide
d. Thyroid-stimulating hormone test

A

ANS: D
Thyroid disease can make hair thin and brittle. Liver function testing is indicated for a patient who
has jaundice. Oxygen saturation will be used for cyanosis. Cherry-colored lips indicate carbon
monoxide poisoning.

How well did you know this?
1
Not at all
2
3
4
5
Perfectly
33
Q

A nurse identifies lice during a child’s scalp assessment. The nurse teaches the parents about hair

care. Which information from the parents indicates the nurse needs to followup?
a. We will use lindane-based shampoos.
b. We will use the sink to wash hair.
c. We will use a fine-toothed comb.
d. We will use a vinegar hair rinse.

A

ANS: A
Products containing lindane, a toxic ingredient, often cause adverse reactions; the nurse will need to
follow up to correct the misconception. All the rest are correct. Instruct parents who have children
with head lice to shampoo thoroughly with pediculicide (shampoo available at drugstores) in cold
water at a basin or sink, comb thoroughly with a fine-toothed comb, and discard the comb. A dilute
solution of vinegar and water helps loosen nits.

How well did you know this?
1
Not at all
2
3
4
5
Perfectly
34
Q

A nurse identifies lice during a child’s scalp assessment. The nurse teaches the parents about hair

care. Which information from the parents indicates the nurse needs to followup?
a. We will use lindane-based shampoos.
b. We will use the sink to wash hair.
c. We will use a fine-toothed comb.
d. We will use a vinegar hair rinse.

A

ANS: A
Products containing lindane, a toxic ingredient, often cause adverse reactions; the nurse will need to
follow up to correct the misconception. All the rest are correct. Instruct parents who have children
with head lice to shampoo thoroughly with pediculicide (shampoo available at drugstores) in cold
water at a basin or sink, comb thoroughly with a fine-toothed comb, and discard the comb. A dilute
solution of vinegar and water helps loosen nits.

How well did you know this?
1
Not at all
2
3
4
5
Perfectly
35
Q

A male student comes to the college health clinic. He hesitantly describes that he found
something wrong with his testis when taking a shower. Which assessment finding will alert thenurse
to possible testicular cancer?
a. Hard, pea-sized testicular lump
b. Rubbery texture of testes
c. Painful enlarged testis
d. Prolonged diuretic use

A

ANS: A
The most common symptoms of testicular cancer are a painless enlargement of one testis and the
appearance of a palpable, small, hard lump, about the size of a pea, on the front or side of the testicle.
Normally, the testes feel smooth, rubbery, and free of nodules. Use of diuretics, sedatives, or
antihypertensive can lead to erection or ejaculation problems.

How well did you know this?
1
Not at all
2
3
4
5
Perfectly
36
Q

The patient is a 45-year-old African-American male who has come in for a routine annual

physical. Which type of preventive screening does the nurse discuss with the patient?
a. Digital rectal examination of the prostate
b. Complete eye examination every year
c. CA 125 blood test once a year
d. Colonoscopy every 3 years

A

ANS: A
Recommended preventive screenings include a digital rectal examination of the prostate and
prostate-specific antigen test starting at age 50. CA 125 blood tests are indicated for women at high
risk for ovarian cancer. Patients over the age of 65 need to have complete eye examinations yearly.
Colonoscopy every 10 years is recommended in patients 50 years of age and older.

How well did you know this?
1
Not at all
2
3
4
5
Perfectly
37
Q

An advanced practice nurse is preparing to assess the external genitalia of a 25-year-old
American woman of Chinese descent. Which action will the nurse do first?
a. Place the patient in the lithotomy position.
b. Drape the patient to enhance patient comfort.
c. Assess the patient’s feelings about the examination.
d. Ask the patient if she would like her mother to be present in the room.

A

ANS: C
Patients who are Chinese American often believe that examination of the external genitalia is
offensive. Before proceeding with the examination, the nurse first determines how the patient feels
about the procedure and explains the procedure to answer any questions and to help the patient feel
comfortable with the assessment. Once the patient is ready to have her external genitalia examined,
the nurse places the patient in the lithotomy position and drapes the patient appropriately. Typically,
nurses ask adolescents if they want a parent present during the examination. The patient in this
question is 25 years old; asking if she would like her mother to be present is inappropriate.

How well did you know this?
1
Not at all
2
3
4
5
Perfectly
38
Q
An older-adult patient is being seen for chronic entropion. Which condition will the nurse assess
 for in this patient?
a. Ptosis
b. Infection
c. Borborygmi
d. Exophthalmos
A

ANS: B
The diagnosis of entropion can lead to lashes of the lids irritating the conjunctiva and cornea.
Irritation can lead to infection. Exophthalmos is a bulging of the eyes and usually indicates
hyperthyroidism. An abnormal drooping of the lid over the pupil is called ptosis. In the older adult,
ptosis results from a loss of elasticity that accompanies aging. Hyperactive sounds are loud,
“growling” sounds called Borborygmi, which indicate increased GI motility

How well did you know this?
1
Not at all
2
3
4
5
Perfectly
39
Q

. During a school physical examination, the nurse reviews the patient’s current medical history.
The nurse discovers the patient has allergies. Which assessment finding is consistent with allergies?
a. Clubbing
b. Yellow discharge
c. Pale nasal mucosa
d. Puffiness of nasal mucosa

A

ANS: C
Pale nasal mucosa with clear discharge indicates allergy. Clubbing is due to insufficient oxygenation
at the periphery resulting from conditions such as chronic emphysema and congenital heart disease; it
is noted in the nails. A sinus infection results in yellowish or greenish discharge. Habitual use of
intranasal cocaine and opioids causes puffiness and increased vascularity of the nasal mucosa.

How well did you know this?
1
Not at all
2
3
4
5
Perfectly
40
Q

. A teen female patient reports intermittent abdominal pain for 12 hours. No dysuria is present.
Which action will the nurse take when performing an abdominal assessment?
a. Assess the area that is most tender first.
b. Ask the patient about the color of her stools.
c. Recommend that the patient take more laxatives.
d. Avoid sexual references such as possible pregnancy.

A

ANS: B
Abdominal pain can be related to bowels. If stools are black or tarry (melena), this may indicate
gastrointestinal alteration. The nurse should caution patients about the dangers of excessive use of
laxatives or enemas. There is not enough information about the abdominal pain to recommend
laxatives. Determine if the patient is pregnant, and note her last menstrual period. Pregnancy causes
changes in abdominal shape and contour. Assess painful areas last to minimize discomfort and
anxiety.

How well did you know this?
1
Not at all
2
3
4
5
Perfectly
41
Q

During a genitourinary examination of a 30-year-old male patient, the nurse identifies a small
amount of a white, thick substance on the patient’s uncircumcised glans penis. What isthe
nurse’s next step?
a. Record this as a normal finding.
b. Avoid embarrassing questions about sexual activity.
c. Notify the provider about a suspected sexually transmitted infection.
d. Tell the patient to avoid doing self-examinations until symptoms clear.

A

ANS: A
A small amount of thick, white smegma sometimes collects under the foreskin in the uncircumcised
male and is considered normal. Penile pain or swelling, genital lesions, and urethral discharge are
signs and symptoms that may indicate sexually transmitted infections (STI). All men 15 years and
older need to perform a male-genital self-examination monthly. The nurse needs to assess a patient’s
sexual history and use of safe sex habits. Sexual history reveals risks for STI and HIV.

How well did you know this?
1
Not at all
2
3
4
5
Perfectly
42
Q

During a routine physical examination of a 70-year-old patient, a blowing sound is auscultated
over the carotid artery. Which assessment finding will the nurse report tothe health care provider?
a. Bruit
b. Thrill
c. Phlebitis
d. Right-sided heart failure

A

ANS: A
A bruit is the sound of turbulence of blood passing through a narrowed blood vessel and is
auscultated as a blowing sound. A bruit can reflect cardiovascular disease in the carotid artery of
middle-aged to older adults. Intensity or loudness is related to the rate of blood flow through the
heart or the amount of blood regurgitated. A thrill is a continuous palpable sensation that resembles
the purring of a cat. Jugular venous distention, not bruit, is a possible sign of right-sided heart failure.
Some patients with heart disease have distended jugular veins when sitting. Phlebitis is an
inflammation of a vein that occurs commonly after trauma to the vessel wall, infection,
immobilization, and prolonged insertion of IV catheters. It affects predominantly peripheral veins.

How well did you know this?
1
Not at all
2
3
4
5
Perfectly
43
Q

While assessing the skin of an 82-year-old patient, a nurse discovers nonpainful, ruby red papules
on the patient’s trunk. What is the nurse’s next action?
a. Explain that the patient has basal cell carcinoma and should watch for spread.
b. Document cherry angiomas as a normal older adult skin finding.
c. Tell the patient that this is a benign squamous cell carcinoma.
d. Record the presence of petechiae

A

ANS: B
The skin is normally free of lesions, except for common freckles or age-related changes such as skin
tags, senile keratosis (thickening of skin), cherry angiomas (ruby red papules), and atrophic warts.
Basal cell carcinoma is most common in sun-exposed areas and frequently occurs in a background of
sun-damaged skin; it almost never spreads to other parts of the body. Squamous cell carcinoma is
more serious than basal cell and develops on the outer layers of sun-exposed skin; these cells may
travel to lymph nodes and throughout the body. Report abnormal lesions to the health care provider
for further examination. Petechiae are nonblanching, pinpoint-size, red or purple spots on the skin
caused by small hemorrhages in the skin layers.

How well did you know this?
1
Not at all
2
3
4
5
Perfectly
44
Q

A nurse is caring for a group of patients. Which patient will the nurse see first?
a.
An adult with an S4 heart sound
b.
A young adult with an S3 heart sound
c. An adult with vesicular lung sounds in the lung periphery
d. A young adult with Broncho vesicular breath sounds between the scapula posteriorly

A

ANS: A
A fourth heart sound (S4) occurs when the atria contract to enhance ventricular filling. An S4 is often
heard in healthy older adults, children, and athletes, but it is not normal in adults. Because S4 also
indicates an abnormal condition, report it to a health care provider. An S3 is considered abnormal in
adults over 31 years of age but can often be heard normally in children and young adults. Vesicular
lungs sounds in the periphery and bronchovesicular lung sounds in between the scapula are normal
findings.

How well did you know this?
1
Not at all
2
3
4
5
Perfectly
45
Q

A nurse is auscultating different areas on an adult patient. Which technique should the nurse use
during an assessment?
a. Uses the bell to listen for lung sounds
b. Uses the diaphragm to listen for bruits
c. Uses the diaphragm to listen for bowel sounds
d. Uses the bell to listen for high-pitched murmurs

A

ANS: C
The bell is best for hearing low-pitched sounds such as vascular (bruits) and certain heart sounds
(low-pitched murmurs), and the diaphragm is best for listening to high-pitched sounds such as bowel
and lung sounds and high-pitched murmurs.

How well did you know this?
1
Not at all
2
3
4
5
Perfectly
46
Q

. A patient in the emergency department is reporting left lower abdominal pain. Which proper
order will the nurse follow to perform the comprehensive abdominalexamination?
a. Percussion, palpation, auscultation
b. Percussion, auscultation, palpation
c. Inspection, palpation, auscultation
d. Inspection, auscultation, palpation

A

ANS: D
The order of an abdominal examination differs slightly from that of other assessments. Begin with
inspection and follow with auscultation. By using auscultation before palpation, the chance of
altering the frequency and character of bowel sounds is lessened

How well did you know this?
1
Not at all
2
3
4
5
Perfectly
47
Q

The patient presents to the clinic with dysuria and hematuria. How does the nurse proceed to
assess for kidney inflammation?
a. Uses deep palpation posteriorly.
b. Lightly palpates each abdominal quadrant.
c. Percusses posteriorly the costovertebral angle at the scapular line.
d. Inspects abdomen for abnormal movement or shadows using indirect lighting.

A

ANS: C
With the patient sitting or standing erect, use direct or indirect percussion to assess for kidney
inflammation. With the ulnar surface of the partially closed fist, percuss posteriorly the
costovertebral angle at the scapular line. If the kidneys are inflamed, the patient feels tenderness
during percussion. Use a systematic palpation approach for each quadrant of the abdomen to assess
for muscular resistance, distention, abdominal tenderness, and superficial organs or masses. Light
palpation would not detect kidney tenderness because the kidneys sit deep within the abdominal
cavity. Posteriorly, the lower ribs and heavy back muscles protect the kidneys, so they cannot be
palpated. Kidney inflammation will not cause abdominal movement. However, to inspect the
abdomen for abnormal movement or shadows, the nurse should stand on the patient’s right side and
inspect from above the abdomen using direct light over the abdomen.

How well did you know this?
1
Not at all
2
3
4
5
Perfectly
48
Q
An older-adult patient is taking aminoglycoside for a severe infection. Which assessment is
 the priority?
a. Eyes
b. Ears
c. Skin
d. Reflexes
A

ANS: B
Older adults are especially at risk for hearing loss caused by ototoxicity (injury to auditory nerve)
resulting from high maintenance doses of antibiotics (e.g., aminoglycosides). While eyes and skin are
important, they are not the priority. Reflexes are expected to be diminished in older adults.

How well did you know this?
1
Not at all
2
3
4
5
Perfectly
49
Q

The patient has had a stroke that has affected the ability to speak. The patient becomes extremely
frustrated when trying to speak. The patient responds correctly to questions and instructions but
cannot form words coherently. Which type of aphasia is the patientexperiencing?
a. Sensory
b. Receptive
c. Expressive
d. Combination

A

ANS: C
The two types of aphasias are sensory (or receptive) and motor (or expressive). The patient cannot
form words coherently, indicating expressive or motor aphasia is present. The patient responds
correctly to questions and instructions, indicating receptive or sensory aphasia is not present. Patients
sometimes suffer a combination of receptive and expressive aphasia, but this is not the case here.

How well did you know this?
1
Not at all
2
3
4
5
Perfectly
50
Q

A nurse is assessing a patient’s cranial nerve IX. Which items does the nurse gather before
conducting the assessment? (Select all that apply.)
a. Vial of sugar
b. Snellen chart
c. Tongue blade
d. Ophthalmoscope
e. Lemon applicator

A

ANS: A, C, E
Cranial nerve IX is the glossopharyngeal, which controls taste and ability to swallow. The nurse asks
the patient to identify sour (lemon) or sweet (sugar) tastes on the back of the tongue and uses a
tongue blade to elicit a gag reflex. Ophthalmoscopes are used for vision. A Snellen chart is used to
test cranial nerve II (optic).

How well did you know this?
1
Not at all
2
3
4
5
Perfectly
51
Q
A nurse is assessing several patients. Which assessment findings will cause the nurse to follow up?
 (Select all that apply.)
a. Orthopnea
b. Nonpalpable lymph nodes
c. Pleural friction rub present
d. Crackles in lower lung lobes
e. Grade 5 muscle function level
f. A 160-degree angle between nail plate and nail
A

ANS: A, C, D
Abnormal findings will cause a nurse to follow up. Orthopnea is abnormal and indicates
cardiovascular or respiratory problems. Pleural friction rub is abnormal and indicated an inflamed
pleura. Crackles are adventitious breath sounds and indicate random, sudden reinflation of groups of
alveoli, indicating disruptive passage of air through small airways. Lymph nodes should be
nonpalpable; palpable lymph nodes are abnormal. Grade 5 muscle function is normal. A 160-degree
angle between nail plate and nail is normal; a larger degree angle is abnormal and indicates clubbing.

How well did you know this?
1
Not at all
2
3
4
5
Perfectly
52
Q

Which action should the nurse take when using critical thinking to make clinicaldecisions?

a. Make decisions based on intuition.
b. Accept one established way to provide care.
c. Consider what is important in a given situation.
d. Read and follow the heath care provider’s orders.

A

ANS: C
A critical thinker considers what is important in each clinical situation, imagines and explores
alternatives, considers ethical principles, and makes informed decisions about the care of patients.
Patient care can be provided in many ways. The use of evidence-based knowledge, or knowledge
based on research or clinical expertise, makes you an informed critical thinker. Following health care
provider’s orders is not considered a critical thinking skill. If your knowledge causes you to question
a health care provider’s order, do so.

How well did you know this?
1
Not at all
2
3
4
5
Perfectly
53
Q

A patient continues to report postsurgical incision pain at a level of 9 out of 10 after pain
medicine is given. The next dose of pain medicine is not due for another hour. What shouldthe
critically thinking nurse do first?
a. Explore other options for pain relief.
b. Discuss the surgical procedure and reason for the pain.
c. Explain to the patient that nothing else has been ordered.
d. Offer to notify the health care provider after morning rounds are completed.

A

ANS: A
The critically thinking nurse should explore all options for pain relief first. The nurse should use
critical thinking to determine the cause of the pain and determine various options for pain, not just
ordered pain medications. The nurse can act independently to determine all options for pain relief
and does not have to wait until after the health care provider rounds are completed. Explaining the
cause of the pain does not address options for pain relief.

How well did you know this?
1
Not at all
2
3
4
5
Perfectly
54
Q

A nurse is completing an assessment. Which findings will the nurse report as subjective data?
(Select all that apply.)
a. Patient’s temperature
b. Patient’s wound appearance
c. Patient describing excitement about discharge
d. Patient pacing the floor while awaiting test results
e. Patient’s expression of fear regarding upcoming surgery

A

ANS: C, E
Subjective data include patient’s feelings, perceptions, and reported symptoms. Expressing feelings
such as excitement or fear is an example of subjective data. Objective data are observations or
measurements of a patient’s health status. In this question, the appearance of the wound and the
patient’s temperature are objective data. Pacing is an observable patient behavior and is also
considered objective data.

How well did you know this?
1
Not at all
2
3
4
5
Perfectly
55
Q

The nurse is gathering data on a patient. Which data will the nurse report as objectivedata?

a. States “doesn’t feel good”
b. Reports a headache
c. Respirations 16
d. Nauseated

A

ANS: C
Objective data are observations or measurements of a patient’s health status, like respirations.
Inspecting the condition of a surgical incision or wound, describing an observed behavior, and
measuring blood pressure are examples of objective data. States “doesn’t feel good,” reports a
headache, and nausea are all subjective data. Subjective data include the patient’s feelings,
perceptions, and reported symptoms. Only patients provide subjective data relevant to their health
condition.

How well did you know this?
1
Not at all
2
3
4
5
Perfectly
56
Q

A patient expresses fear of going home and being alone. Vital signs are stable and the incision is
nearly completely healed. What can the nurse infer from the subjective data?
a. The patient can now perform the dressing changes without help.
b. The patient can begin retaking all of the previous medications.
c. The patient is apprehensive about discharge.
d. The patient’s surgery was not successful.

A

ANS: C
Subjective data include expressions of fear of going home and being alone. These data indicate (use
inference) that the patient is apprehensive about discharge. Expressing fear is not an appropriate sign
that a patient is able to perform dressing changes independently. An order from a health care provider
is required before a patient is taught to resume previous medications. The nurse cannot infer that
surgery was not successful if the incision is nearly completely healed.

How well did you know this?
1
Not at all
2
3
4
5
Perfectly
57
Q

. The nurse is using critical thinking skills during the first phase of the nursing process. Which
action indicates the nurse is in the first phase?
a. Completes a comprehensive database
b. Identifies pertinent nursing diagnoses
c. Intervenes based on priorities of patient care
d. Determines whether outcomes have been achieved

A

ANS: A
The assessment phase of the nursing process involves data collection to complete a thorough patient
database and is the first phase. Identifying nursing diagnoses occurs during the diagnosis phase or
second phase. The nurse carries out interventions during the implementation phase (fourth phase),
and determining whether outcomes have been achieved takes place during the evaluation phase (fifth
phase) of the nursing process.

How well did you know this?
1
Not at all
2
3
4
5
Perfectly
58
Q

A nurse is using the problem-oriented approach to data collection. Which action will the nurse
take first?
a. Complete the questions in chronological order.
b. Focus on the patient’s presenting situation.
c. Make accurate interpretations of the data.
d. Conduct an observational overview.

A

ANS: B
A problem-oriented approach focuses on the patient’s current problem or presenting situation rather
than on an observational overview. The database is not always completed using a chronological
approach if focusing on the current problem. Making interpretations of the data is not data collection.
Data interpretation occurs while appropriate nursing diagnoses are assigned. The question is asking
about data collection.

How well did you know this?
1
Not at all
2
3
4
5
Perfectly
59
Q

A nurse is conducting a nursing health history. Which component will the nurse address?

a. Nurse’s concerns
b. Patient expectations
c. Current treatment orders
d. Nurse’s goals for the patient

A

ANS: B
Some components of a nursing health history include chief concern, patient expectations, spiritual
health, and review of systems. Current treatment orders are located under the Orders section in the
patient’s chart and are not a part of the nursing health history. Patient concerns, not nurse’s concerns,
are included in the database. Goals that are mutually established, not nurse’s goals, are part of the
nursing care plan.

How well did you know this?
1
Not at all
2
3
4
5
Perfectly
60
Q

After assessing a patient, a nurse develops a standard formal nursing diagnosis. What is the
rationale for the nurse’s actions?
a. To form a language that can be encoded only by nurses
b. To distinguish the nurse’s role from the physician’s role
c. To develop clinical judgment based on other’s intuition
d. To help nurses focus on the scope of medical practice

A

ANS: B
The standard formal nursing diagnosis serves several purposes. Nursing diagnoses distinguish the
nurse’s role from that of the physician/health care provider and help nurses focus on the scope of
nursing practice (not medical) while fostering the development of nursing knowledge. A nursing
diagnosis provides the precise definition that gives all members of the health care team a common
language for understanding the patient’s needs. A diagnosis is a clinical judgment based on
information.

How well did you know this?
1
Not at all
2
3
4
5
Perfectly
61
Q

The nurse is reviewing a patient’s plan of care, which includes the nursing diagnostic statement, Impaired physical mobility related to tibial fracture as evidenced by the patient’s inability to ambulate. Which part of the diagnostic statement does the nurse need to revise?

a. Etiology
b. Nursing diagnosis
c. Collaborative problem
d. Defining characteristic

A

ANS: A
The etiology, or related to factor, of tibial fracture is a medical diagnosis and needs to be revised. The nursing diagnosis is appropriate because the patient is unable to ambulate. A collaborative problem is
an actual or potential physiological complication that nurses monitor to detect the onset of changes in
a patient’s health status; there is no collaborative problem listed. The defining characteristic
(subjective and objective data that support the diagnosis) is appropriate for Impaired physical
mobility.

How well did you know this?
1
Not at all
2
3
4
5
Perfectly
62
Q

A nurse is using assessment data gathered about a patient and combining critical thinking to
develop a nursing diagnosis. What is the nurse doing?
a. Assigning clinical cues
b. Defining characteristics
c. Diagnostic reasoning
d. Diagnostic labeling

A

ANS: C
Diagnostic reasoning is defined as a process of using the assessment data gathered about a patient to
logically explain a clinical judgment, in this case a nursing diagnosis. Defining characteristics are
assessment findings that support the nursing diagnosis. Defining characteristics are the subjective and
objective clinical cues, which a nurse gathers intentionally and unintentionally. The nurse organizes
all of the patient’s data into meaningful and usable data clusters, which lead to a diagnostic
conclusion. Diagnostic labeling is simply the name of the diagnosis.

How well did you know this?
1
Not at all
2
3
4
5
Perfectly
63
Q

A patient with a spinal cord injury is seeking to enhance urinary elimination abilities by learning
self-catheterization versus assisted catheterization by home health nurses and family members. The
nurse adds Readiness for enhanced urinary elimination in the care plan. Which type of diagnosisdid
the nurse write?
a. Risk
b. Problem focused
c. Health promotion
d. Collaborative problem

A

ANS: C
A health promotion nursing diagnosis is a clinical judgment concerning motivation and desire to
increase well-being and actualize human health potential. A problem-focused nursing diagnosis
describes a clinical judgment concerning an undesirable human response to a health condition/life
process that exists in an individual, family, or community. A risk nursing diagnosis is a clinical
judgment concerning the vulnerability of an individual, family, group or community for developing
an undesirable human response to health conditions/life processes. A collaborative problem is an
actual or potential physiological complication that nurses monitor to detect the onset of changes in a
patient’s health status.

How well did you know this?
1
Not at all
2
3
4
5
Perfectly
64
Q

A nurse is developing a care plan for a patient with a pelvic fracture on bed rest. Which goal
statement is realistic for the nurse to assign to this patient?
a. Patient will increase activity level this shift.
b. Patient will turn side to back to side with assistance every 2 hours.
c. Patient will use the walker correctly to ambulate to the bathroom as needed.
d. Patient will use a sliding board correctly to transfer to the bedside commode as needed.

A

ANS: A
A goal is a broad statement of desired change; the patient will increase activity level is a broad
statement. Turning is the expected outcome. When determining goals, the nurse needs to ensure that
the goal is individualized and realistic for the patient. Since the patient is on bed rest, using a walker
and bedside commode is contraindicated.

How well did you know this?
1
Not at all
2
3
4
5
Perfectly
65
Q

. Which information indicates a nurse has a good understanding of a goal?

a. It is a statement describing the patient’s accomplishments without a time restriction.
b. It is a realistic statement predicting any negative responses to treatments.
c. It is a broad statement describing a desired change in a patient’s behavior.
d. It is a measurable change in a patient’s physical state.

A

ANS: C
A goal is a broad statement that describes a desired change in a patient’s condition or behavior. A
goal is mutually set with the patient. An expected outcome is the measurable changes (patient
behavior, physical state, or perception) that must be achieved to reach a goal. Expected outcomes are
time limited, measurable ways of determining if a goal is met.

How well did you know this?
1
Not at all
2
3
4
5
Perfectly
66
Q

SKIP

A

.

How well did you know this?
1
Not at all
2
3
4
5
Perfectly
67
Q

A patient has reduced muscle strength following a left-sided stroke and is at risk for falling.
Which intervention ismost appropriate for the nursing diagnostic statement Risk forfalls?
a. Keep all side rails down at all times.
b. Encourage patient to remain in bed most of the shift.
c. Place patient in room away from the nurses’ station if possible.
d. Assist patient into and out of bed every 4 hours or as tolerated.

A

ANS: D
Risk for falls is a risk (potential) nursing diagnosis; therefore, the nurse needs to implement actions
that will prevent a fall. Assisting the patient into and out of bed is the most appropriate intervention
to prevent the patient from falling. Encouraging activity builds muscle strength, and helping the
patient with transfers ensures patient safety. Encouraging the patient to stay in bed will not promote
muscle strength. Decreased muscle strength is the risk factor placing the patient in jeopardy of
falling. The side rails should be up, not down, according to agency policy. This will remind the
patient to ask for help to get up and will keep the patient from rolling out of bed. The patient should
be placed near the nurses’ station, so a staff member can quickly get to the room and assist the
patient if necessary

How well did you know this?
1
Not at all
2
3
4
5
Perfectly
68
Q

Which action will the nurse take after the plan of care for a patient isdeveloped?

a. Place the original copy in the chart, so it cannot be tampered with or revised.
b. Communicate the plan to all health care professionals involved in the patient’s care.
c. File the plan of care in the administration office for legal examination.
d. Send the plan of care to quality assurance for review.

A

ANS: B
Setting realistic goals and outcomes often means you must communicate these goals and outcomes to
caregivers in other settings who will assume responsibility for patient care. The plan of care
communicates nursing care priorities to nurses and other health care professionals. Know also that a
plan of care is dynamic and changes as the patient’s needs change. All health care professionals
involved in the patient’s care need to be informed of the plan of care. The plan of care is not sent to
the administrative office or quality assurance office.

How well did you know this?
1
Not at all
2
3
4
5
Perfectly
69
Q

A nurse is evaluating goals and expected outcomes for a confused patient. Which finding
indicates positive progress toward resolving the confusion?
a. Patient wanders halls at night.
b. Patient’s side rails are up with bed alarm activated.
c. Patient denies pain while ambulating with assistance.
d. Patient correctly states names of family members in the room.

A

ANS: D
The goal for this patient would address a decrease or absence of confusion. Thus, one possible sign
that a patient’s confusion is improving is seen when a patient can correctly state the names of family
members in the room. You examine the results of care by using evaluative measures that relate to
goals and expected outcomes. Keeping the side rails up and using a bed alarm are interventions to
promote patient safety and prevent falls. The patient’s denying pain indicates positive progress
toward resolving pain. The patient’s wandering the halls is a sign of confusion.

How well did you know this?
1
Not at all
2
3
4
5
Perfectly
70
Q

A nurse notices that a patient has a structural curvature of the spine associated with vertebral

rotation. Which condition will the nurse most likely find documented in the patient’s medicalrecord?
a. Scoliosis
b. Arthritis
c. Osteomalacia
d. Osteogenesis

A

ANS: A
Scoliosis is a structural curvature of the spine associated with vertebral rotation. Osteogenesis
imperfecta is an inherited disorder that makes bones porous, short, bowed, and deformed.
Osteomalacia is an uncommon metabolic disease characterized by inadequate and delayed
mineralization, resulting in compact and spongy bone. Arthritis is an inflammatory joint disease
characterized by inflammation or destruction of the synovial membrane and articular cartilage and by
systemic signs of inflammation.

How well did you know this?
1
Not at all
2
3
4
5
Perfectly
71
Q

The nurse is providing care to a patient who is bedridden. The nurse raises the height of the bed.
What is the rationale for the nurse’s action?
a. Narrows the nurse’s base of support.
b. Allows the nurse to bring feet closer together.
c. Prevents a shift in the nurse’s base of support.
d. Shifts the nurse’s center of gravity farther away from the base of support.

A

ANS: C
Raising the height of the bed when performing a procedure prevents bending too far at the waist and
shifting the base of support. Balance is maintained by proper body alignment and posture through
two simple techniques. First, widen the base of support by separating the feet to a comfortable
distance. Second, increase balance by bringing the center of gravity closer to the base of support.

How well did you know this?
1
Not at all
2
3
4
5
Perfectly
72
Q

A nurse is following the no-lift policy when working to prevent personal injury. Which type of
personal back injury is the nurse most likely trying to prevent?
a. Thoracic
b. Cervical
c. Lumbar
d. Sacral

A

ANS: C
The most common back injury for nurses is strain on the lumbar muscle group, which includes the
muscles around the lumbar vertebrae. While cervical, thoracic, and sacral can occur, lumbar is the
most common.

How well did you know this?
1
Not at all
2
3
4
5
Perfectly
73
Q

The nurse is caring for a patient who cannot bear weight but needs to be transferred from the bed
to a chair. The nurse decides to use a transportable hydraulic lift. What will the nursedo?
a. Place a horseshoe-shaped base on the opposite side from the chair.
b. Remove straps before lowering the patient to the chair.
c. Hook longer straps to the bottom of the sling.
d. Attach short straps to the bottom of the sling.

A

ANS: C
The nurse should attach the hooks on the strap to the holes in the sling. Short straps hook to top holes
of the sling; longer straps hook to the bottom of the sling. The horseshoe-shaped base goes under the
side of the bed on the side with the chair. Position the patient and lower slowly into the chair in
accordance with manufacturer guidelines to safely guide the patient into the back of the chair as the
seat descends; then remove the straps and the mechanical/hydraulic lift.

How well did you know this?
1
Not at all
2
3
4
5
Perfectly
74
Q

The nurse is preparing to move a patient to a wheelchair. Which action indicates the nurse is
following recommendations for safe patient handling?
a. Mentally reviews the transfer steps before beginning
b. Uses own strength to transfer the patient
c. Focuses solely on body mechanics
d. Bases decisions on intuition

A

ANS: A
Safe patient handling includes mentally reviewing the transfer steps before beginning the procedure
to ensure both the patient’s and your safety. Use the patient’s strength when lifting, transferring, or
moving when possible. Body mechanics alone do not protect the nurse from injury to the
musculoskeletal system when moving, lifting, or transferring patients. After completing the
assessment, nurses use an algorithm to guide decisions about safe patient handling

How well did you know this?
1
Not at all
2
3
4
5
Perfectly
75
Q

A nurse is working in a facility that follows a comprehensive safe patient-handling program.
Which finding will alert the nurse to intervene?
a. Mechanical lifts are in a locked closet.
b. Algorithms for patient handling are available.
c. Ergonomic assessment protocols are being followed.
d. A no-lift policy is in place with adherence by all staff.

A

ANS: A
The nurse will follow up when lifts are not kept in convenient locations. Comprehensive safe patienthandling programs include the following elements: an ergonomics assessment protocol for health
care environments, patient assessment criteria, algorithms for patient handling and movement,
special equipment kept in convenient locations to help transfer patients, back injury resource nurses,
an “after-action review” that allows the health care team to apply knowledge about moving patients
safely in different settings, and a no-lift policy.

How well did you know this?
1
Not at all
2
3
4
5
Perfectly
76
Q

The patient is brought to the emergency department with possible injury to the left shoulder.
Which area will the nurse assess to best determine joint mobility?
a. The patient’s gait
b. The patient’s range of motion
c. The patient’s ethnic influences
d. The patient’s fine-motor coordination

A

ANS: B
Assessing range of motion is one assessment technique used to determine the degree of joint mobility
and injury to a joint. Gait is the manner or style of walking. It has little bearing on the shoulder
damage. Assessing fine-motor coordination would be beneficial in helping to assess the patient’s
ability to perform tasks such as feeding and dressing but would not help in evaluating the shoulder.
Ethnic influences would not have a direct bearing on the amount of mobility in the joint.

How well did you know this?
1
Not at all
2
3
4
5
Perfectly
77
Q

. The patient reports being tired and weak and lacks energy. Upon assessment, the nurse findsthat
patient has gained weight, and blood pressure and pulse are elevated after climbing stairs.Which
nursing diagnosis will the nurse add to the care plan?
a.
Fatigue
b.
Ineffective coping
c.
Activity intolerance
d. Decreased cardiac output

A

ANS: C
You consider nursing diagnoses of Activity intolerance or Fatigue in a patient who reports being tired
and weak. Further review of assessed defining characteristics (e.g., abnormal heart rate and verbal
report of weakness and the assessment findings occurring during the activity of climbing the stairs)
leads to the definitive diagnosis (Activity intolerance). There is no data to support ineffective coping
or decreased cardiac output.

How well did you know this?
1
Not at all
2
3
4
5
Perfectly
78
Q

The patient weighs 450 lbs (204.5 kg) and reports shortness of breath with any exertion. The
health care provider has recommended beginning an exercise program. The patient states that shecan
hardly get out of bed and just cannot do anything around the house. Which nursing diagnosis willthe
nurse add to the care plan?
a.
Activity intolerance related to excessive weight
b.
Impaired physical mobility related to bed rest
c.
Imbalanced nutrition: less than body requirements
d. Impaired gas exchange related to shortness of breath

A

ANS: A
In this case, activity intolerance is related to the patient’s excessive weight. The patient is not on bed
rest although claims that it is difficult to get out of bed, making this diagnosis inappropriate.
Shortness of breath is a symptom, not a cause, of Impaired gas exchange, making this nursing
diagnosis ineffective. The patient certainly has an imbalance of nutrition, but it is more than body
requirements (obesity).

How well did you know this?
1
Not at all
2
3
4
5
Perfectly
79
Q

The nurse is teaching a patient how to use a cane. Which information will the nurse include in the
teaching session?
a. Place the cane at the top of the hip bone.
b. Place the cane on the stronger side of the body.
c. Place the cane in front of the body and then move the good leg.
d. Place the cane 10 to 15 inches in front of the body when walking

A

ANS: B
Have the patient keep the cane on the stronger side of the body. A person’s cane length is equal to the
distance between the greater trochanter and the floor. The cane should be moved first and then the
weaker leg. For maximum support when walking, the patient places the cane forward 15 to 25 cm (6
to 10 inches), keeping body weight on both legs. The weaker leg is then moved forward to the cane,
so body weight is divided between the cane and the stronger leg.

How well did you know this?
1
Not at all
2
3
4
5
Perfectly
80
Q

The nurse is preparing to transfer an uncooperative patient who does not have upper body

strength. Which piece of equipment will be best for the nurses to obtain?
a. Drawsheet
b. Full body sling
c. Overhead trapeze
d. Friction-reducing slide sheet

A

ANS: B
Using a mechanical lift and full body sling to transfer an uncooperative patient who can bear partial
weight or a patient who cannot bear weight and is either uncooperative or does not have upper body
strength to move from bed to chair prevents musculoskeletal injuries to health care workers. The
nurse should not attempt to move the patient with a drawsheet. The patient does not have upper body
strength so an overhead trapeze is not appropriate. A friction-reducing slide sheet that minimizes
shearing forces is not as effective as a full body sling.

How well did you know this?
1
Not at all
2
3
4
5
Perfectly
81
Q

A nurse is preparing to move a patient who is able to assist. Which principles will the nurse
consider when planning for safe patient handling? (Select all that apply.)
a. Keep the body’s center of gravity high.
b. Face the direction of the movement.
c. Keep the base of support narrow.
d. Use the under-axilla technique.
e. Use proper body mechanics.
f. Use arms and legs.

A

ANS: B, E, F
When a patient is able to assist, remember the following principles: The wider the base of support,
the greater the stability of the nurse; the lower the center of gravity, the greater the stability of the
nurse; facing the direction of movement prevents abnormal twisting of the spine. The use of assistive
equipment and continued use of proper body mechanics significantly reduces the risk of
musculoskeletal injuries. Use arms and legs (not back) because the leg muscles are stronger, larger
muscles capable of greater work without injury. The under-axilla technique is physically stressful for
nurses and uncomfortable for patients.

How well did you know this?
1
Not at all
2
3
4
5
Perfectly
82
Q

A nurse is working in a facility that uses no-lift policies. Which benefits will the nurse observe in
the facility? (Select all that apply.)
a. Reduced number of work-related injuries
b. Increased musculoskeletal accidents
c. Reduced safety of patients
d. Improved health of nurses
e. Increased indirect costs

A

ANS: A, D
Implementing evidence-based interventions and programs (e.g., lift teams) reduces the number of
work-related injuries, which improves the health of the nurse and reduces indirect costs to the health
care facility (e.g., workers’ compensation and replacing injured workers). Knowing the movements
and functions of muscles in maintaining posture and movement and implementing evidence-based
knowledge about safe patient handling are essential to protecting the safety of both the patient and
the nurse.

How well did you know this?
1
Not at all
2
3
4
5
Perfectly
83
Q

A nurse is assessing body alignment. What is the nursemonitoring?

a. The relationship of one body part to another while in different positions
b. The coordinated efforts of the musculoskeletal and nervous systems
c. The force that occurs in a direction to oppose movement
d. The inability to move about freely

A

ANS: A
The terms body alignment and posture are similar and refer to the positioning of the joints, tendons,
ligaments, and muscles while standing, sitting, and lying. Body alignment means that the individual’s
center of gravity is stable. Body mechanics is a term used to describe the coordinated efforts of the
musculoskeletal and nervous systems. Friction is a force that occurs in a direction to oppose
movement. Immobility is the inability to move about freely.

How well did you know this?
1
Not at all
2
3
4
5
Perfectly
84
Q
A nurse is preparing a care plan for a patient who is immobile. Which psychosocial aspect will
 the nurse consider?
a. Loss of bone mass
b. Loss of strength
c. Loss of weight
d. Loss of hope
A

ANS: D
Loss of hope is a psychosocial aspect. Patients with restricted mobility may have some depression.
Depression is an affective disorder characterized by exaggerated feelings of sadness, melancholy,
dejection, worthlessness, emptiness, and hopelessness out of proportion to reality. All the rest are
physiological aspects: bone mass, strength, and weight.

How well did you know this?
1
Not at all
2
3
4
5
Perfectly
85
Q

The nurse is preparing to lift a patient. Which action will the nurse take first?

a. Position a drawsheet under the patient.
b. Assess weight and determine assistance needs.
c. Delegate the task to a nursing assistive personnel.
d. Attempt to manually lift the patient alone before asking for assistance.

A

ANS: B
When lifting, assess the weight you will lift, and determine the assistance you will need. The nurse
has to assess before positioning a drawsheet or delegating the task. Manual lifting is the last resort,
and it is used when the task at hand does not involve lifting most or all of the patient’s weight; most
facilities have a no-lift policy.

How well did you know this?
1
Not at all
2
3
4
5
Perfectly
86
Q

The nurse is observing the way a patient walks. Which aspect is the nurseassessing?

a. Activity tolerance
b. Body alignment
c. Range of motion
d. Gait

A

ANS: D
Gait describes a particular manner or style of walking. Activity tolerance is the type and amount of
exercise or work that a person is able to perform. Body alignment refers to the position of the joints,
tendons, ligaments, and muscles while standing, sitting, and lying. Range of motion is the maximum
amount of movement available at a joint in one of the three planes of the body: sagittal, frontal, or
transverse.

How well did you know this?
1
Not at all
2
3
4
5
Perfectly
87
Q

Which patient will cause the nurse to select a nursing diagnosis of Impaired physical mobility for
a care plan?
a. A patient who is completely immobile
b. A patient who is not completely immobile
c. A patient at risk for single-system involvement
d. A patient who is at risk for multisystem problems

A

ANS: B
The diagnosis of Impaired physical mobility applies to the patient who has some limitation but is not
completely immobile. The diagnosis of Risk for disuse syndrome applies to the patient who is
immobile and at risk for multisystem problems because of inactivity. Beyond these diagnoses, the list
of potential diagnoses is extensive because immobility affects multiple body systems.

How well did you know this?
1
Not at all
2
3
4
5
Perfectly
88
Q

The patient has the nursing diagnosis of Impaired physical mobility related to pain in the left

shoulder. Whichpriority action will the nurse take?
a. Encourage the patient to do self-care.
b. Keep the patient as mobile as possible.
c. Encourage the patient to perform ROM.
d. Assist the patient with comfort measures.

A

ANS: D
The diagnosis related to pain requires the nurse to assist the patient with comfort measures so that the
patient is then willing and more able to move. Pain must be controlled so the patient will not be
reluctant to initiate movement. The diagnosis related to reluctance to initiate movement requires
interventions aimed at keeping the patient as mobile as possible and encouraging the patient to
perform self-care and ROM.

How well did you know this?
1
Not at all
2
3
4
5
Perfectly
89
Q

The nurse is caring for a young-adult patient on the medical-surgical unit. When doing midnight
checks, the nurse observes the patient awake, putting a puzzle together. Which information will the
nurse consider to best explain this finding?
a. The patient misses family and is lonely.
b. The patient was waiting to talk with the nurse.
c. The patient has been kept up with the noise on the unit.
d. The patient’s sleep-wake cycle preference is late evening

A

ANS: D
This patient is awake and alert enough to do a puzzle. The individual’s sleep-wake preference is
probably late evening. All persons have biological clocks that synchronize their sleep-wake cycle.
This explains why some individuals fall asleep in the early evening, whereas others go to bed at
midnight or early morning. Waiting to talk with the nurse, being lonely, and noise on the unit may
contribute to lack of sleep, but the best explanation for the patient being awake is the biological
clock.

How well did you know this?
1
Not at all
2
3
4
5
Perfectly
90
Q

The nurse is caring for a patient who is having trouble sleeping. Which action will the nurse take?

a. Suggest snug-fitting nightwear.
b. Provide a favorite beverage.
c. Encourage deep breathing.
d. Walk with the patient.

A

ANS: C
Relaxation exercises such as slow, deep breathing for 1 or 2 minutes relieve tension and prepare the
body for rest. Instruct patients to wear loose-fitting nightwear. Walking and drinking a favorite
beverage would not necessarily encourage sleep.

How well did you know this?
1
Not at all
2
3
4
5
Perfectly
91
Q

Which nursing observation of the patient in intensive care indicates the patient issleeping
comfortably during NREM sleep?
a. Eyes closed, lying quietly, respirations 12, heart rate 60
b. Eyes closed, tossing in bed, respirations 18, heart rate 80
c. Eyes closed, mumbling to self, respirations 16, heart rate 68
d. Eyes closed, lying supine in bed, respirations 22, heart rate 66

A

ANS: A
During NREM sleep, biological functions slow. During sleep, the heart rate decreases to 60
beats/min or less. The patient experiences decreased respirations, blood pressure, and muscle tone.
Heart rates above 60 are too high and respirations of 22 are too high to indicate comfortable NREM
sleep.

How well did you know this?
1
Not at all
2
3
4
5
Perfectly
92
Q

The nurse is discussing lack of sleep with a middle-aged adult. Which area should the
nurse most likely assess to determine a possible cause of the lack ofsleep?
a. Anxiety
b. Loud teenagers
c. Caring for pets
d. Late night television

A

ANS: A
During middle adulthood, the total time spent sleeping at night begins to decline. Anxiety,
depression, and certain physical illnesses can affect sleep, and women can experience menopausal
symptoms. Insomnia is common because of the changes and stresses associated with middle age.
Teenagers, caring for pets, and late night television can influence the amount of sleep; however,
these are not the most common causes of insomnia in this age group

How well did you know this?
1
Not at all
2
3
4
5
Perfectly
93
Q

The nurse is discussing lack of sleep with a middle-aged adult. Which area should the
nurse most likely assess to determine a possible cause of the lack ofsleep?
a. Anxiety
b. Loud teenagers
c. Caring for pets
d. Late night television

A

ANS: A
During middle adulthood, the total time spent sleeping at night begins to decline. Anxiety,
depression, and certain physical illnesses can affect sleep, and women can experience menopausal
symptoms. Insomnia is common because of the changes and stresses associated with middle age.
Teenagers, caring for pets, and late night television can influence the amount of sleep; however,
these are not the most common causes of insomnia in this age group

How well did you know this?
1
Not at all
2
3
4
5
Perfectly
94
Q

The nurse is completing an assessment on an older-adult patient who is having difficultyfalling

asleep. Which condition will the nurse further assess for in this patient?
a. Depression
b. Mild fatigue
c. Hypertension
d. Hypothyroidism

A

ANS: A
Older adults and other individuals who experience depressive mood problems experience delays in
falling asleep, earlier appearance of REM sleep, frequent awakening, feelings of sleeping poorly, and
daytime sleepiness. A person who is moderately fatigued usually achieves restful sleep, especially if
the fatigue is the result of enjoyable work or exercise. Hypertension often causes early-morning
awakening and fatigue. Alcohol speeds the onset of sleep. Hypothyroidism decreases stage 4 sleep.

How well did you know this?
1
Not at all
2
3
4
5
Perfectly
95
Q

.A patient has obstructive sleep apnea. Which assessment is the priority?

a. Gastrointestinal function
b. Neurological function
c. Respiratory status
d. Circulatory status

A

ANS: C
In obstructive sleep apnea, the upper airway becomes partially or completely blocked, diminishing
nasal airflow or stopping it. The person still attempts to breathe because the chest and abdominal
movement continue, which results in loud snoring and snorting sounds. According to the ABCs of
prioritizing care, airway and respiratory status takes priority over gastrointestinal, circulatory, and
neurologic functioning.

How well did you know this?
1
Not at all
2
3
4
5
Perfectly
96
Q

The nurse is caring for a patient who has been in holding in the emergency department for 24
hours. The nurse is concerned about the patient’s experiencing sleep deprivation. Which actionwill
be best for the nurse to take?
a. Expedite the process of obtaining a medical-surgical room for the patient.
b. Pull the curtains shut, dim the lights, and decrease the number of visitors.
c. Obtain an order for a hypnotic medication to help the patient sleep.
d. Ask everyone in the unit to try to be quiet so the patient can sleep.

A

ANS: A
The most effective treatment for sleep deprivation is elimination or correction of factors that disrupt
the sleep pattern. Obtaining a private room in the medical-surgical unit for the patient will help with
decreasing stimuli and promoting more rest than an individual can obtain in an emergency
department even with the interventions mentioned

How well did you know this?
1
Not at all
2
3
4
5
Perfectly
97
Q

The nurse is beginning a sleep assessment on a patient. Which question will be most appropriate
for the nurse to ask initially?
a. “What is going on?”
b. “How are you sleeping?”
c. “Are you taking any medications?”
d. “What did you have for dinner last night?”

A

ANS: B
Sleep is a subjective experience. Only the patient is able to report whether or not it is sufficient and
restful. Asking patients how they are sleeping is an introductory question. After this beginning
question is asked, problems with sleep such as the nature of the problem, signs and symptoms, onset
and duration of the issue, severity, predisposing factors, and the effect on the patient can be assessed.
What is going on is too broad and open ended for information about sleep to be obtained specifically.
Medications and food intake can be part of the detailed assessment of sleep issues.

How well did you know this?
1
Not at all
2
3
4
5
Perfectly
98
Q

The nurse adds a nursing diagnosis of ineffective breathing pattern to a patient’s care plan.Which
sleep condition caused the nurse to assign this nursing diagnosis?
a. Insomnia
b. Narcolepsy
c. Sleep deprivation
d. Obstructive sleep apnea

A

ANS: D
Obstructive sleep apnea (OSA) occurs when the muscles or structures of the oral cavity or throat
relax during sleep. The upper airway becomes partially or completely blocked, diminishing airflow
or stopping it for as long as 30 seconds. The person still attempts to breathe because chest and
abdominal movements continue, resulting in snoring or snorting sounds. With narcolepsy, the person
feels an overwhelming wave of sleepiness and falls asleep. Insomnia is characterized by chronic
difficulty falling asleep. Sleep deprivation is a condition caused by dyssomnia. OSA is the only one
of these conditions that results in blockage of the airway and impacts the ability to breathe.

How well did you know this?
1
Not at all
2
3
4
5
Perfectly
99
Q
The patient presents to the clinic with reports of irritability, being sleepy during the day,
chronically not being able to fall asleep, and being tired. Which nursing diagnosis will thenurse
document in the plan of care?
a.
Anxiety
b.
Fatigue
c.
Insomnia
d. Sleep deprivation
A

ANS: C
Insomnia is experienced when the patient has chronic difficulty falling asleep, frequent awakenings
from sleep, and/or short sleep or nonrestorative sleep. It is the most common sleep-related complaint
and includes symptoms such as irritability, excessive daytime sleepiness, not being able to fall
asleep, and fatigue. Anxiety is a vague, uneasy feeling of discomfort or dread accompanied by an
autonomic response. Fatigue is an overwhelming sustained sense of exhaustion with decreased
capacity for physical and mental work at a usual level. Sleep deprivation is a condition caused by
dyssomnia and includes symptoms caused by illness, emotional distress, or medications.

How well did you know this?
1
Not at all
2
3
4
5
Perfectly
100
Q

The nurse is preparing an older-adult patient’s evening medications. Which treatment willthe
nurse recognize as relatively safe for difficulty sleeping in older adults?
a. Ramelteon (Rozerem)
b. Benzodiazepine
c. Antihistamine
d. Kava

A

ANS: A
Ramelteon (Rozerem), a melatonin receptor agonist, is well tolerated and appears to be effective in
improving sleep by improving the circadian rhythm and shortening time to sleep onset. It is safe for
long- and short-term use particularly in older adults. The use of benzodiazepines in older adults is
potentially dangerous because of the tendency of the drugs to remain active in the body for a longer
time. As a result, they also cause respiratory depression, next-day sedation, amnesia, rebound
insomnia, and impaired motor functioning and coordination, which leads to increased risk of falls.
Caution older adults about using over-the-counter antihistamines because their long duration of
action can cause confusion, constipation, and urinary retention. Kava promotes sleep in patients with
anxiety; it should be used cautiously because of its potential toxic effects on the liver.

How well did you know this?
1
Not at all
2
3
4
5
Perfectly
101
Q

The nurse is caring for a patient who has not been able to sleep well while in the hospital, leading
to a disrupted sleep-wake cycle. Which assessment findings will the nurse monitor for in thispatient?
(Select all that apply.)
a. Changes in physiological function such as temperature
b. Decreased appetite and weight loss
c. Anxiety, irritability, and restlessness
d. Shortness of breath and chest pain
e. Nausea, vomiting, and diarrhea
f. Impaired judgment

A

ANS: A, B, C, F
The biological rhythm of sleep frequently becomes synchronized with other body functions. Changes
in body temperature correlate with sleep pattern. When the sleep-wake cycle becomes disrupted,
changes in physiological function such as temperature can occur. Patients can experience decreased
appetite, loss of weight, anxiety, restlessness, irritability, and impaired judgment. Gastrointestinal
and respiratory/cardiovascular symptoms such as shortness of breath and chest pain are not
symptoms of a disrupted sleep cycle.

How well did you know this?
1
Not at all
2
3
4
5
Perfectly
102
Q

The nurse is caring for a patient in the intensive care unit who is having trouble sleeping. Thenurse
explains the purpose of sleep and its benefits. Which information will the nurse include in the
teaching session? (Select all that apply.)
a. NREM sleep contributes to body tissue restoration.
b. During NREM sleep, biological functions increase.
c. Restful sleep preserves cardiac function.
d. Sleep contributes to cognitive restoration.
e. REM sleep decreases cortical activity.

A

ANS: A, C, D
Sleep contributes to physiological and psychological restoration. NREM sleep contributes to body
tissue restoration. It allows the body to rest and conserve energy. This benefits the cardiac system by
allowing the heart to beat fewer times each minute. During stage 4, the body releases growth
hormone for renewal and repair of specialized cells such as the brain. During NREM sleep,
biological functions slow. REM sleep is necessary for brain tissue restoration and cognitive
restoration and is associated with a change in cerebral blood flow and increased cortical activity

How well did you know this?
1
Not at all
2
3
4
5
Perfectly
103
Q

The patient and the nurse discuss the need for sleep. After the discussion, the patient is able tostate
factors that hinder sleep. Which statements indicate the patient has a good understanding ofthe
teaching? (Select all that apply.)
a. “Drinking coffee at 7 PM could interrupt my sleep.”
b. “Staying up late for a party can interrupt sleep patterns.”
c. “Exercising 2 hours before bedtime can decrease relaxation.”
d. “Changing the time of day that I eat dinner can disrupt sleep.”
e. “Worrying about work can disrupt my sleep.”
f. “Taking an antacid can decrease sleep.”

A

ANS: A, B, D, E
Caffeine, alcohol, and nicotine consumed late in the evening produce insomnia. Worry over personal
problems or situations frequently disrupts sleep. Alterations in routines, including changing
mealtimes and staying up late at night for social activities, can disrupt sleep. Exercising 2 hours
before bedtime actually increases a sense of fatigue and promotes relaxation. Taking an antacid does
not decrease sleep.

How well did you know this?
1
Not at all
2
3
4
5
Perfectly
104
Q

A nurse is teaching a nutrition class about the different daily values. When teaching about the
referenced daily intakes (RDIs), which information should the nurse include?
a. Have values for protein, vitamins, and minerals
b. Are based on percentages of fat, cholesterol, and fiber
c. Have replaced recommended daily allowances (RDAs)
d. Are used to develop diets for chronic illnesses requiring 1800 cal/day

A

ANS: A
The RDIs are the first set, comprising protein, vitamins, and minerals based on the RDA. The daily
reference values (DRVs) make up the second set and consist of nutrients such as total fat, saturated
fat, cholesterol, carbohydrates, fiber, sodium, and potassium. Combined, both sets make up the daily
values used on food labels. Daily values did not replace RDAs but provided a separate, more
understandable format for the public. Daily values are based on percentages of a diet consisting of
2000 kcal/day for adults and children 4 years or older.

How well did you know this?
1
Not at all
2
3
4
5
Perfectly
105
Q

In teaching mothers-to-be about infant nutrition, which instruction should the nurse provide?

a. Supplement breast milk with corn syrup.
b. Give cow’s milk during the first year of life.
c. Add honey to infant formulas for increased energy.
d. Provide breast milk or formula for the first 4 to 6 months.

A

ANS: D
Breast milk or formula provides sufficient nutrition for the first 4 to 6 months of life. Infants should
not have regular cow’s milk during the first year of life. It is too concentrated for an infant’s kidneys
to manage, increases the risk of milk product allergies, and is a poor source of iron and vitamins C
and E. Furthermore, children under 1 year of age should never ingest honey and corn syrup products
because they are potential sources of the botulism toxin, which increases the risk of infant death.

How well did you know this?
1
Not at all
2
3
4
5
Perfectly
106
Q

When planning care for an adolescent who plays sports, which modification should thenurse
include in the care plan?
a. Increasing carbohydrates to 55% to 60% of total intake
b. Providing vitamin and mineral supplements
c. Decreasing protein intake to 0.75 g/kg/day
d. Limiting water before and after exercise

A

ANS: A
Sports and regular moderate to intense exercise necessitate dietary modification to meet increased
energy needs for adolescents. Carbohydrates, both simple and complex, are the main source of
energy, providing 55% to 60% of total daily kilocalories. Protein needs increase to 1 to 1.5 g/kg/day.
Fat needs do not increase. Adequate hydration is very important. Adolescents need to ingest water
before and after exercise to prevent dehydration, especially in hot, humid environments. Vitamin and
mineral supplements are not required, but intake of iron-rich foods is required to prevent anemia.

How well did you know this?
1
Not at all
2
3
4
5
Perfectly
107
Q

.In providing prenatal care to a pregnant patient, what does the nurse teach the expectantmother?

a. Calcium intake is especially important in the first trimester.
b. Protein intake needs to decrease to preserve kidney function.
c. Folic acid is needed to help prevent birth defects and anemia.
d. Extra vitamins and minerals should be taken as much as possible.

A

ANS: C
Folic acid intake is particularly important for DNA synthesis and growth of red blood cells.
Inadequate intake may lead to fetal neural tube defects, anencephaly, or maternal megaloblastic
anemia. Protein intake throughout pregnancy needs to increase to 60 grams daily. Calcium intake is
especially critical in the third trimester, when fetal bones mineralize. Prenatal care usually includes
vitamin and mineral supplementation to ensure daily intakes; however, pregnant women should not
take additional supplements beyond prescribed amounts.

How well did you know this?
1
Not at all
2
3
4
5
Perfectly
108
Q

The patient is an 80-year-old male who is visiting the clinic today for a routine physical
examination. The patient’s skin turgor is fair, but the patient reports fatigue and weakness. The skin
is warm and dry, pulse rate is 116 beats/min, and urinary sodium level is slightly elevated. Which
instruction should the nurse provide?
a. Drink more water to prevent further dehydration.
b. Drink more calorie-dense fluids to increase caloric intake.
c. Drink more milk and dairy products to decrease the risk of osteoporosis.
d. Drink more grapefruit juice to enhance vitamin C intake and medication absorption.

A

ANS: A
Thirst sensation diminishes, leading to inadequate fluid intake or dehydration; the patient should be
encouraged to drink more water/fluids. Symptoms of dehydration in older adults include confusion,
weakness, hot dry skin, furrowed tongue, and high urinary sodium. Milk continues to be an important
food for older woman and men, who need adequate calcium to protect against osteoporosis; the
patient’s problem is dehydration, not osteoporosis. Caution older adults to avoid grapefruit and
grapefruit juice because these will decrease absorption of many drugs. The patient needs fluids not
calories; drinking calorie-dense fluids is unnecessary

How well did you know this?
1
Not at all
2
3
4
5
Perfectly
109
Q

The nurse is providing home care for a patient diagnosed with acquired immunodeficiency
syndrome (AIDS). Which dietary intervention will the nurse add to the care plan?
a. Provide small, frequent nutrient-dense meals for maximizing kilocalories.
b. Prepare hot meals because they are more easily tolerated by the patient.
c. Avoid salty foods and limit liquids to preserve electrolytes.
d. Encourage intake of fatty foods to increase caloric intake.

A

ANS: A
Small, frequent, nutrient-dense meals that limit fatty foods and overly sweet foods are easier to
tolerate. Restorative care of malnutrition resulting from AIDS focuses on maximizing kilocalories
and nutrients. Patients benefit from eating cold foods and drier or saltier foods with fluid in between.

How well did you know this?
1
Not at all
2
3
4
5
Perfectly
110
Q

. When planning care for an adolescent who plays sports, which modification should the nurse
include in the care plan?
a. Increasing carbohydrates to 55% to 60% of total intake
b. Providing vitamin and mineral supplements
c. Decreasing protein intake to 0.75 g/kg/day
d. Limiting water before and after exercise

A

ANS: A
Sports and regular moderate to intense exercise necessitate dietary modification to meet increased
energy needs for adolescents. Carbohydrates, both simple and complex, are the main source of
energy, providing 55% to 60% of total daily kilocalories. Protein needs increase to 1 to 1.5 g/kg/day.
Fat needs do not increase. Adequate hydration is very important. Adolescents need to ingest water
before and after exercise to prevent dehydration, especially in hot, humid environments. Vitamin and
mineral supplements are not required, but intake of iron-rich foods is required to prevent anemia.

How well did you know this?
1
Not at all
2
3
4
5
Perfectly
111
Q
A nurse is preparing to administer an enteral feeding. In which order will the nurse implement the
steps, starting with the first one?
1. Elevate head of bed to at least 30 degrees.
2. Check for gastric residual volume.
3. Flush tubing with 30 mL of water.
4. Verify tube placement.
5. Initiate feeding.
a. 4, 2, 1, 5, 3
b. 2, 4, 1, 3, 5
c. 1, 4, 2, 3, 5
d. 2, 1, 4, 5, 3
A

ANS: C
The steps for an enteral feeding are as follows: Place patient in high-Fowler’s position or elevate
head of bed to at least 30 (preferably 45) degrees; verify tube placement; check for gastric residual
volume; flush tubing with 30 mL of water; and initiate feeding.

How well did you know this?
1
Not at all
2
3
4
5
Perfectly
112
Q

. Before giving the patient an intermittent gastric tube feeding, what should the nurse do?

a. Make sure that the tube is secured to the gown with a safety pin.
b. Inject air into the stomach via the tube and auscultate.
c. Have the tube feeding at room temperature.
d. Check to make sure pH is at least 5.

A

ANS: C
Be sure that the formula is at room temperature. Cold formula causes gastric cramping and
discomfort because the mouth and the esophagus do not warm the liquid. Do not use safety pins.
Safety pins can become unfastened and may cause harm to the patient. Auscultation is no longer
considered a reliable method for verification of tube placement because a tube inadvertently placed
in the lungs, pharynx, or esophagus transmits sound similar to that of air entering the stomach.
Gastric fluid of patient who has fasted for at least 4 hours usually has a pH of 1 to 4, especially when
the patient is not receiving gastric-acid inhibitor.

How well did you know this?
1
Not at all
2
3
4
5
Perfectly
113
Q
A small-bore feeding tube is placed. Which technique will the nurse use to best verify tube
placement?
a. X-ray
b. pH testing
c. Auscultation
d. Aspiration of contents
A

ANS: A
At present, the most reliable method for verification of placement of small-bore feeding tubes is xray examination. Aspiration of contents and pH testing are not infallible. The nurse would need a
more precise indicator to help differentiate the source of tube feeding aspirate. Auscultation is no
longer considered a reliable method for verification of tube placement because a tube inadvertently
placed in the lungs, pharynx, or esophagus transmits sound similar to that of air entering the stomach.

How well did you know this?
1
Not at all
2
3
4
5
Perfectly
114
Q

The nurse is concerned about pulmonary aspiration when providing the patient with an
intermittent tube feeding. Which action is the priority?
a. Observe the color of gastric contents.
b. Verify tube placement before feeding.
c. Add blue food coloring to the enteral formula.
d. Run the formula over 12 hours to decrease overload.

A

ANS: B
A major cause of pulmonary aspiration is regurgitation of formula. The nurse needs to verify tube
placement and elevate the head of the bed 30 to 45 degrees during feedings and for 2 hours
afterward. While observing the color of gastric contents is a component, it is not the priority
component; pH is the primary component. The addition of blue food coloring to enteral formula to
assist with detection of aspirate is no longer used. Do not hang formula longer than 4 to 8 hours.
Formula becomes a medium for bacterial growth after that length of time.

How well did you know this?
1
Not at all
2
3
4
5
Perfectly
115
Q
.A nurse is caring for a patient with a postsurgical wound. When planning care, which goal will be
 the priority?
a. Reduce dependent nitrogen balance.
b. Maintain negative nitrogen balance.
c. Promote positive nitrogen balance.
d. Facilitate neutral nitrogen balance.
A

ANS: C
When intake of nitrogen is greater than output, the body is in positive nitrogen balance. Positive
nitrogen balance is required for growth, normal pregnancy, maintenance of lean muscle mass and
vital organs, and wound healing. Negative nitrogen balance occurs when the body loses more
nitrogen than the body gains. Neutral nitrogen balance occurs when gain equals loss and is not
optimal for tissue healing. There is no such term as dependent nitrogen balance.

How well did you know this?
1
Not at all
2
3
4
5
Perfectly
116
Q

. The nurse is working on a medical-surgical unit that has been participating in a research project
associated with pressure ulcers. Which risk factor will the nurse assess for that predisposes apatient
to pressure ulcer development?
a. Decreased level of consciousness
b. Adequate dietary intake
c. Shortness of breath
d. Muscular pain

A

ANS: A
Patients who are confused or disoriented or who have changing levels of consciousness are unable to
protect themselves. The patient may feel the pressure but may not understand what to do to relieve
the discomfort or to communicate that he or she is feeling discomfort. Impaired sensory perception,
impaired mobility, shear, friction, and moisture are other predisposing factors. Shortness of breath,
muscular pain, and an adequate dietary intake are not included among the predisposing factors.

How well did you know this?
1
Not at all
2
3
4
5
Perfectly
117
Q

Which nursing observation will indicate the patient is at risk for pressure ulcerformation?

a. The patient has fecal incontinence.
b. The patient ate two thirds of breakfast.
c. The patient has a raised red rash on the right shin.
d. The patient’s capillary refill is less than 2 seconds.

A

ANS: A
The presence and duration of moisture on the skin increase the risk of ulcer formation by making it
susceptible to injury. Moisture can originate from wound drainage, excessive perspiration, and fecal
or urinary incontinence. Bacteria and enzymes in the stool can enhance the opportunity for skin
breakdown because the skin is moistened and softened, causing maceration. Eating a balanced diet is
important for nutrition, but eating just two thirds of the meal does not indicate that the individual is at
risk. A raised red rash on the leg again is a concern and can affect the integrity of the skin, but it is
located on the shin, which is not a high-risk area for skin breakdown. Pressure can influence capillary
refill, leading to skin breakdown, but this capillary response is within normal limits.

How well did you know this?
1
Not at all
2
3
4
5
Perfectly
118
Q

The nurse is admitting an older patient from a nursing home. During the assessment, the nurse
notes a shallow open reddish, pink ulcer without slough on the right heel of the patient. How will the
nurse stage this pressure ulcer?
a. Stage I
b. Stage II
c. Stage III
d. Stage IV

A

ANS: B
This would be a Stage II pressure ulcer because it presents as partial-thickness skin loss involving
epidermis and dermis. The ulcer presents clinically as an abrasion, blister, or shallow crater. Stage I
is intact skin with nonblanchable redness over a bony prominence. With a Stage III pressure ulcer,
subcutaneous fat may be visible, but bone, tendon, and muscles are not exposed. Stage IV involves
full-thickness tissue loss with exposed bone, tendon, or muscle.

How well did you know this?
1
Not at all
2
3
4
5
Perfectly
119
Q
A patient has developed a pressure ulcer. Which laboratory data will be important for the nurse to
 check?
a. Vitamin E
b. Potassium
c. Albumin
d. Sodium
A

ANS: C
Normal wound healing requires proper nutrition. Serum proteins are biochemical indicators of
malnutrition, and serum albumin is probably the most frequently measured of these parameters. The
best measurement of nutritional status is prealbumin because it reflects not only what the patient has
ingested but also what the body has absorbed, digested, and metabolized. Zinc and copper are the
minerals important for wound healing, not potassium and sodium. Vitamins A and C are important
for wound healing, not vitamin E.

How well did you know this?
1
Not at all
2
3
4
5
Perfectly
120
Q

A nurse is caring for a patient with a wound. Which assessment data will be most important for
the nurse to gather with regard to wound healing?
a. Muscular strength assessment
b. Pulse oximetry assessment
c. Sensation assessment
d. Sleep assessment

A

ANS: B
Oxygen fuels the cellular functions essential to the healing process; the ability to perfuse tissues with
adequate amounts of oxygenated blood is critical in wound healing. Pulse oximetry measures the
oxygen saturation of blood. Assessment of muscular strength and sensation, although useful for
fitness and mobility testing, does not provide any data with regard to wound healing. Sleep, although
important for rest and for integration of learning and restoration of cognitive function, does not
provide any data with regard to wound healing.

How well did you know this?
1
Not at all
2
3
4
5
Perfectly
121
Q
. The nurse is completing an assessment of the patient’s skin’s integrity. Which assessment is
 the priority?
a. Pressure points
b. Breath sounds
c. Bowel sounds
d. Pulse points
A

ANS: A
Observe pressure points such as bony prominences. The nurse continually assesses the skin for signs
of ulcer development. Assessment for tissue pressure damage includes visual and tactile inspectionof
the skin. Assessment of pulses, breath sounds, and bowel sounds is part of a head-to-toe assessment
and could influence the function of the body and ultimately skin integrity; however, this assessment
is not a specific part or priority of a skin assessment.

How well did you know this?
1
Not at all
2
3
4
5
Perfectly
122
Q

The nurse is caring for a patient who is immobile. The nurse wants to decrease the formation of
pressure ulcers. Which action will the nurse take first?
a. Offer favorite fluids.
b. Turn the patient every 2 hours.
c. Determine the patient’s risk factors.
d. Encourage increased quantities of carbohydrates and fats.

A

ANS: C
The first step in prevention is to assess the patient’s risk factors for pressure ulcer development.
When a patient is immobile, the major risk to the skin is the formation of pressure ulcers. Nursing
interventions focus on prevention. Offering favorite fluids, turning, and increasing carbohydrates and
fats are not the first steps. Determining risk factors is first so interventions can be implemented to
reduce or eliminate those risk factors.

How well did you know this?
1
Not at all
2
3
4
5
Perfectly
123
Q

The nurse is caring for a patient who has suffered a stroke and has residual mobility problems.
The patient is at risk for skin impairment. Which initial actions should the nurse take to decrease this
risk?
a. Use gentle cleansers, and thoroughly dry the skin.
b. Use therapeutic bed and mattress.
c. Use absorbent pads and garments.
d. Use products that hold moisture to the skin.

A

ANS: A
Use cleansers with nonionic surfactants that are gentle to the skin. After you clean the skin, make
sure that it is completely dry. Absorbent pads and garments are controversial and should be
considered only when other alternatives have been exhausted. Depending on the needs of the patient,
a specialty bed may be needed, but again, this does not provide the initial defense for skin
breakdown. Use only products that wick moisture away from the patient’s skin.

How well did you know this?
1
Not at all
2
3
4
5
Perfectly
124
Q

The nurse is caring for a patient who is at risk for skin impairment. The patient is able to sit upin
a chair. The nurse includes this intervention in the plan of care. How long should the nurse schedule
the patient to sit in the chair?
a. At least 3 hours
b. Less than 2 hours
c. No longer than 30 minutes
d. As long as the patient remains comfortable

A

ANS: B
When patients are able to sit up in a chair, make sure to limit the amount of time to 2 hours or less.
The chair sitting time should be individualized. In the sitting position, pressure on the ischial
tuberosities is greater than in a supine position. Utilize foam, gel, or an air cushion to distribute
weight. Sitting for longer than 2 hours can increase the chance of ischemia

How well did you know this?
1
Not at all
2
3
4
5
Perfectly
125
Q

The patient has a risk for skin impairment and has a 15 on the Braden Scale upon admission. The
nurse has implemented interventions. Upon reassessment, which Braden score will be the best sign
that the risk for skin breakdown is removed?
a. 12
b. 13
c. 20
d. 23

A

ANS: D
The best sign is a perfect score of 23. The Braden Scale is composed of six subscales: sensory
perception, moisture, activity, mobility, nutrition, and friction and shear. The total score ranges from
6 to 23, and a lower total score indicates a higher risk for pressure ulcer development. The cutoff
score for onset of pressure ulcer risk with the Braden Scale in the general adult population is 18.

How well did you know this?
1
Not at all
2
3
4
5
Perfectly
126
Q

The nurse is caring for a patient with potential skin breakdown. Which components will the nurse
include in the skin assessment? (Select all that apply.)
a. Vision
b. Hyperemia
c. Induration
d. Blanching
e. Temperature of skin

A

ANS: B, C, D, E
Assessment of the skin includes both visual and tactile inspection. Assess for hyperemia and palpate
for blanching or nonblaching. Early signs of skin damage include induration, bogginess (less-thannormal stiffness), and increased warmth at the injury site compared to nearby areas. Changes in
temperature can indicate changes in blood flow to that area of the skin. Vision is not included in the
skin assessment.

How well did you know this?
1
Not at all
2
3
4
5
Perfectly
127
Q

The nurse is caring for a patient with potential skin breakdown. Which components will the nurse
include in the skin assessment? (Select all that apply.)
a. Vision
b. Hyperemia
c. Induration
d. Blanching
e. Temperature of skin

A

ANS: B, C, D, E
Assessment of the skin includes both visual and tactile inspection. Assess for hyperemia and palpate
for blanching or nonblaching. Early signs of skin damage include induration, bogginess (less-thannormal stiffness), and increased warmth at the injury site compared to nearby areas. Changes in
temperature can indicate changes in blood flow to that area of the skin. Vision is not included in the
skin assessment.

How well did you know this?
1
Not at all
2
3
4
5
Perfectly
128
Q

A patient with a hip fracture is having difficulty defecating into a bedpan while lying in bed.
Which action by the nurse will assist the patient in having a successful bowelmovement?
a. Preparing to administer a barium enema
b. Withholding narcotic pain medication
c. Administering laxatives to the patient
d. Raising the head of the bed

A

ANS: D
Lying in bed is an unnatural position; raising the head of the bed assists the patient into a more
normal position that allows proper contraction of muscles for elimination. Laxatives would not give
the patient control over bowel movements. A barium enema is a diagnostic test, not an intervention
to promote defecation. Pain relief measures should be given; however, preventative action should be
taken to prevent constipation.

129
Q

Which nursing intervention is most effective in promoting normal defecation for a patient who
has muscle weakness in the legs?
a. Administer a soapsuds enema every 2 hours.
b. Use a mobility device to place the patient on a bedside commode.
c. Give the patient a pillow to brace against the abdomen while bearing down.
d. Elevate the head of the bed 20 degrees 60 minutes after breakfast while on bedpan.

A

ANS: B
The best way to promote normal defecation is to assist the patient into a posture that is as normal as
possible for defecation. Using a mobility device promotes nurse and patient safety. Elevating the
head of the bed is appropriate but is not the most effective; closer to 30 to 45 degrees is the proper
position for the patient on a bedpan, and the patient is not on bed rest so a bedside commode is the
best choice. Giving the patient a pillow may reduce discomfort, but this is not the best way to
promote defecation. A soapsuds enema is indicated for a patient who needs assistance to stimulate
peristalsis. It promotes non-natural defecation.

130
Q

The nurse will anticipate which diagnostic examination for a patient with black tarry stools?

a. Ultrasound
b. Barium enema
c. Endoscopy
d. Anorectal manometry

A

ANS: C
Black tarry stools are an indication of bleeding in the GI tract; endoscopy would allow visualization
of the bleeding. No other option (ultrasound, barium enema, and anorectal manometry) would allow
GI visualization.

131
Q

The nurse has attempted to administer a tap water enema for a patient with fecal impaction with
no success. The fecal mass is too large for the patient to pass voluntarily. Which is the next priority
nursing action?
a. Preparing the patient for a second tap water enema
b. Obtaining an order for digital removal of stool
c. Positioning the patient on the left side
d. Inserting a rectal tube

A

ANS: B
When enemas are not successful, digital removal of the stool may be necessary to break up pieces of
the stool or to stimulate the anus to defecate. Tap water enemas should not be repeated because of
risk of fluid imbalance. Positioning the patient on the left side does not promote defecation. A rectal
tube is indicated for a patient with liquid stool incontinence or flatus but would not be applicable or
effective for this patient.

132
Q

. A nurse is checking orders. Which order should the nurse question?

a. A normal saline enema to be repeated every 4 hours until stool is produced
b. A hypertonic solution enema for a patient with fluid volume excess
c. A Kayexalate enema for a patient with severe hypokalemia
d. An oil retention enema for a patient with constipation

A

ANS: C
Kayexalate binds to and helps excrete potassium, so it would be contraindicated in patients who are
hypokalemic (have low potassium). Normal saline enemas can be repeated without risk of fluid or
electrolyte imbalance. Hypertonic solutions are intended for patients who cannot handle large fluid
volume and are contraindicated for dehydrated patients. Oil retention enemas lubricate the feces in
the rectum and colon and are used for constipation.

133
Q

The nurse has attempted to administer a tap water enema for a patient with fecal impaction with
no success. The fecal mass is too large for the patient to pass voluntarily. Which is the next priority
nursing action?
a. Preparing the patient for a second tap water enema
b. Obtaining an order for digital removal of stool
c. Positioning the patient on the left side
d. Inserting a rectal tube

A

ANS: B
When enemas are not successful, digital removal of the stool may be necessary to break up pieces of
the stool or to stimulate the anus to defecate. Tap water enemas should not be repeated because of
risk of fluid imbalance. Positioning the patient on the left side does not promote defecation. A rectal
tube is indicated for a patient with liquid stool incontinence or flatus but would not be applicable or
effective for this patient.

134
Q

. Before administering a cleansing enema to an 80-year-old patient, the patient says “I don’t think I
will be able to hold the enema.” Which is the next priority nursing action?
a. Rolling the patient into right-lying Sims’ position
b. Positioning the patient in the dorsal recumbent position on a bedpan
c. Inserting a rectal plug to contain the enema solution after administering
d. Assisting the patient to the bedside commode and administering the enema

A

ANS: B
If you suspect the patient of having poor sphincter control, position on bedpan in a comfortable
dorsal recumbent position. Patients with poor sphincter control are unable to retain all of the enema
solution. Administering an enema with the patient sitting on the toilet is unsafe because it is
impossible to safely guide the tubing into the rectum, and it will be difficult for the patient to retain
the fluid as he or she is in the position used for emptying the bowel. Rolling the patient into rightlying Sims’ position will not help the patient retain the enema. Use of a rectal plug to contain the
solution is inappropriate and unsafe.

135
Q

A nurse is providing care to a group of patients. Which patient will the nurse see first?

a. A child about to receive a normal saline enema
b. A teenager about to receive loperamide for diarrhea
c. An older patient with glaucoma about to receive an enema
d. A middle-aged patient with myocardial infarction about to receive docusate sodium

A

ANS: C
An enema is contradicted in a patient with glaucoma; this patient should be seen first. All the rest are
expected. A child can receive normal saline enemas since they are isotonic. Loperamide, an
antidiarrheal, is given for diarrhea. Docusate sodium is given to soften stool for patients with
myocardial infarction to prevent straining.

136
Q
The nurse administers a cathartic to a patient. Which finding helps the nurse determine that the
 cathartic has a therapeutic effect?
a. Reports decreased diarrhea.
b. Experiences pain relief.
c. Has a bowel movement.
d. Passes flatulence.
A

ANS: C
A cathartic is a laxative that stimulates a bowel movement. It would be effective if the patient
experiences a bowel movement. The other options are not outcomes of administration of a cathartic.
An antidiarrheal will provide relief from diarrhea. Pain medications will provide pain relief.
Carminative enemas provide relief from gaseous distention (flatulence)

137
Q
A patient is receiving a neomycin solution enema. Which primary goal is the nurse trying to
 achieve?
a. Prevent gaseous distention
b. Prevent constipation
c. Prevent colon infection
d. Prevent lower bowel inflammation
A

ANS: C
A medicated enema is a neomycin solution, i.e., an antibiotic used to reduce bacteria in the colon
before bowel surgery. Carminative enemas provide relief from gaseous distention. Bulk forming,
emollient (wetting), and osmotic laxatives and cathartics help prevent constipation or treat
constipation. An enema containing steroid medication may be used for acute inflammation in the
lower colon.

138
Q

Which patient will the nurse assess most closely for an ileus?

a. A patient with a fecal impaction
b. A patient with chronic cathartic abuse
c. A patient with surgery for bowel disease and anesthesia
d. A patient with suppression of hydrochloric acid from medication

A

ANS: C
Any surgery that involves direct manipulation of the bowel temporarily stops peristalsis. Anesthesia
can also cause cessation of peristalsis. This condition, called an ileus, usually lasts about 24 to 48
hours. Fecal impaction, cathartic abuse, and medication to suppress hydrochloric acid will have
bowel sounds, but they may be hypoactive or hyperactive.

139
Q
The nurse is caring for patients with ostomies. In which ostomy location will the nurse expect very
liquid stool to be present?
a. Sigmoid
b. Transverse
c. Ascending
d. Descending
A

ANS: C
The path of digestion goes from the ascending, across the transverse, to the descending and finally
passing into the sigmoid; therefore, the least formed stool (very liquid) would be in the ascending.

140
Q

The nurse is devising a plan of care for a patient with the nursing diagnosis of Constipation
related to opioid use.Which outcome will the nurse evaluate as successful for the patient toestablish
normal defecation?
a. The patient reports eliminating a soft, formed stool.
b. The patient has quit taking opioid pain medication.
c. The patient’s lower left quadrant is tender to the touch.
d. The nurse hears bowel sounds in all four quadrants.

A

ANS: A
The nurse’s goal is for the patient to take opioid medication and to have normal bowel elimination.
Normal stools are soft and formed. Ceasing pain medication is not a desired outcome for the patient.
Tenderness in the left lower quadrant indicates constipation and does not indicate success. Bowel
sounds indicate that the bowels are moving; however, they are not an indication of defecation

141
Q

A nurse is preparing a bowel training program for a patient. Which actions will the nurse take?
(Select all that apply.)
a. Record times when the patient is incontinent.
b. Help the patient to the toilet at the designated time.
c. Lean backward on the hips while sitting on the toilet.
d. Maintain normal exercise within the patient’s physical ability.
e. Apply pressure with hands over the abdomen, and strain while pushing.
f. Choose a time based on the patient’s pattern to initiate defecation-control measures.

A

ANS: A, B, D, F
A successful program includes the following: Assessing the normal elimination pattern and recording
times when the patient is incontinent. Choosing a time based on the patient’s pattern to initiate
defecation-control measures. Maintaining normal exercise within the patient’s physical ability.
Helping the patient to the toilet at the designated time. Offering a hot drink (hot tea) or fruit juice
(prune juice) (or whatever fluids normally stimulate peristalsis for the patient) before the defecation
time. Instructing the patient to lean forward at the hips while sitting on the toilet, apply manual
pressure with the hands over the abdomen, and bear down but do not strain to stimulate colon
emptying.

142
Q

A patient is undergoing chemotherapy. When giving
antiemetics, the nurse will remember that these
drugs are most effective against nausea when given
at which time?
a. Before meals
b. At bedtime
c. Before the chemotherapy begins
d. Just after the chemotherapy begins

A

C. before the chemotherapy begins

143
Q

A 33-year-old patient is in the outpatient cancer center for his first round of chemotherapy. The nurse knows that which schedule is the most appropriate timing for the intravenous antiemetic drug?

a Four hours before the chemotherapy begins
b Thirty minutes before the chemotherapy begins
c At the same time as the chemotherapy drugs
d At the first sign of nausea

A

b. thirty minutes before the chemotherapy begins

144
Q

When planning administration of antiemetic medications to a patient, the nurse is aware that combination therapy is preferred because

A. the risk of constipation is decreased.
B. lower doses of medication are cost-effective.
C. different vomiting pathways are blocked.
D. increased sedation is achieved by higher doses of medication.

A

C. different vomiting pathways are blocked.

Combining antiemetic drugs from various categories allows the blocking of the vomiting center and chemoreceptor trigger zone through different pathways, thus enhancing the antiemetic effect.

145
Q

In developing a plan of care for a patient receiving an antihistamine antiemetic drug, which nursing diagnosis would be of highest priority?
A. Deficient knowledge regarding medication administration
B. Deficient fluid volume related to nausea and vomiting
C. Risk for injury related to side effects of medication
D. Anxiety related to nausea and vomiting

A

B. Deficient fluid volume related to nausea and vomiting

Although all of the options are appropriate nursing diagnoses, fluid volume deficit is the highest priority because it has the highest associated mortality rate. Although a fall or injury could also prove fatal, this diagnosis is a risk; actual nursing diagnoses have priority over potential diagnoses.

146
Q
The nurse would expect to teach a patient about which antiemetic commonly used to prevent motion sickness?
A. scopolamine (Transderm-Scōp)
B. droperidol (Inapsine)
C. prochlorperazine (Compazine)
D. metoclopramide (Reglan)
A

A. scopolamine (Transderm-Scōp)

Scopolamine blocks the binding of acetylcholine with cholinergic receptors in the inner ear, an imbalance of which is a common cause of motion sickness.

147
Q

Which statement regarding antiemetic medications, if made by the patient, indicates the need for further patient teaching?
A. “I will not drive while I am taking these medications because they may cause drowsiness.”
B. “I may take Tylenol to treat the headache caused by ondansetron.”
C. “I will apply the scopolamine patches to rotating sites on my arms.”
D. “I should take my prescribed antiemetic before receiving my chemotherapy dose.”

A

C. “I will apply the scopolamine patches to rotating sites on my arms.”

Transdermal scopolamine patches should be applied to nonirritated areas behind the ear, not on the arms.

148
Q

he nurse is giving medication to reduce nausea.
Which antiemetic drug class is known to cause drying
of secretions and drowsiness when given? (Select
all that apply.)
a. Antihistamines
b. Antidopaminergic drugs
c. Serotonin blockers
d. Tetrahydrocannabinoids
e. Anticholinergics

A

a. Antihistamines
b. Antidopaminergic drugs
d. Tetrahydrocannabinoids
e. Anticholinergics

149
Q

The nurse is preparing to apply an external catheter. Which action will the nurse take?

a. Allow 1 to 2 inches of space between the tip of the penis and the end of the catheter.
b. Spiral wrap the penile shaft using adhesive tape to secure the catheter.
c. Twist the catheter before applying drainage tubing to the end of the catheter.
d. Shave the pubic area before applying the catheter.

A

ANS: A
When applying an external catheter, allow 2.5 to 5 cm (1 to 2 inches) of space between the tip of the
penis and the end of the catheter. Spiral wrap the penile shaft with supplied elastic adhesive. The
strip should not overlap. The elastic strip should be snug but not tight. NOTE: Never use adhesive
tape. Connect drainage tubing to the end of the condom catheter. Be sure the condom is not twisted.
Connect the catheter to a large-volume drainage bag or leg. Clip hair at the base of the penile shaft,
as necessary. Do not shave the pubic area.

150
Q

. A patient is experiencing oliguria. Which action should the nurse perform first?

a. Assess for bladder distention.
b. Request an order for diuretics.
c. Increase the patient’s intravenous fluid rate.
d. Encourage the patient to drink caffeinated beverages.

A

ANS: A
Oliguria is diminished urinary output in relation to fluid intake. The nurse first should gather all
assessment data to determine the potential cause of oliguria. It could be that the patient does not have
adequate intake, or it could be that the bladder sphincter is not functioning and the patient is retaining
water. Increasing fluids is effective if the patient does not have adequate intake or if dehydration
occurs. Caffeine can work as a diuretic but is not helpful if an underlying pathology is present. An
order for diuretics can be obtained if the patient was retaining water, but this should not be the first
action

151
Q

The nurse anticipates a suprapubic catheter for which patient?

a. A patient with recent prostatectomy
b. A patient with a urethral stricture
c. A patient with an appendectomy
d. A patient with menopause

A

ANS: B
A patient with a urethral stricture is most likely to have a suprapubic catheter. Suprapubic catheters
are placed when there is blockage of the urethra (e.g., enlarged prostate, urethral stricture, after
urological surgery). A patient with a recent prostatectomy indicates the enlarged prostate was
removed and would not need a suprapubic catheter; however, continuous bladder irrigation may be
needed. Appendectomies and menopause do not require a suprapubic catheter.

152
Q

nurse administers an antimuscarinic to a patient. Which findings indicate the patient is having
therapeutic effects from this medication? (Select all that apply.)
a. Decrease in dysuria
b. Decrease in urgency
c. Decrease in frequency
d. Decrease in prostate size
e. Decrease in bladder infection

A

ANS: B, C
When newly started on an antimuscarinic, you should monitor the patient for effectiveness, watching
for a decrease in symptoms such as urgency, frequency, and urgency urinary incontinence episodes.
Patients with painful urination are sometimes prescribed urinary analgesics that act on the urethral
and bladder mucosa (e.g., phenazopyridine). Antibiotics are used to treat bladder infections. Agents
that shrink the prostate include finasteride and dutasteride.

153
Q
An 86-year-old patient is experiencing uncontrollable leakage of urine with a strong desire tovoid
and even leaks on the way to the toilet. Which priority nursing diagnosis will the nurse include in
 the patient’s plan of care?
a.
Functional urinary incontinence
b.
Urge urinary incontinence
c.
Impaired skin integrity
d.
Urinary retention
A

ANS: B
Urge urinary incontinence is the leakage of urine associated with a strong urge to void. Patients leak
urine on the way to or at the toilet and rush or hurry to the toilet. Urinary retention is the inability to
empty the bladder. Functional urinary incontinence is incontinence due to causes outside the urinary
tract, such as mobility or cognitive impairments. While Impaired skin integrity can occur, it is not the
priority at this time, and there is no data to support this diagnosis.

154
Q

A patient who is dribbling small amounts on the way to the bathroom and has a diagnosis of urge incontinency

b. A patient with reflex incontinence with elevated blood pressure and pulse rate
c. A patient with an indwelling catheter that has stool on the catheter tubing
d. A patient who has just voided and needs a postvoid residual test

A

ANS: B
The nurse should see the patient with reflex incontinence first. Patients with reflex incontinence are
at risk for developing autonomic dysreflexia, a life-threatening condition that causes severe elevation
of blood pressure and pulse rate and diaphoresis. This is a medical emergency requiring immediate intervention; notify the health care provider immediately. A patient with urge incontinence will
dribble, and this is expected. While a patient with a catheter and stool on the tubing does need to be
cleaned, it is not life threatening. The nurse has 10 minutes before checking on the patient who has a
postvoid residual test.

155
Q

The nurse suspects cystitis related to a lower urinary tract infection. Which clinical manifestation
does the nurse expect the patient to report?
a. Dysuria
b. Flank pain
c. Frequency
d. Fever

A

ANS: C
Cystitis is inflammation of the bladder; associated symptoms include hematuria, foul-smelling
cloudy urine, and urgency/frequency. Dysuria is a common symptom of a lower urinary tract
infection (bladder). Flank pain, fever, and chills are all signs of pyelonephritis (upper urinary tract).

156
Q

The nurse is caring for a patient in the hospital. The nurse observes the nursing assistive
personnel (NAP) turning off the handle faucet with bare hands. Which professional practiceprinciple
supports the need for follow-up with the NAP?
a. The nurse is responsible for providing a safe environment for the patient.
b. Different scopes of practice allow modification of procedures.
c. Allowing the water to run is a waste of resources and money.
d. This is a key step in the procedure for washing hands.

A

ANS: A
The nurse is responsible for providing a safe environment for the patient. The effectiveness of
infection control practices depends on conscientiousness and consistency in using effective aseptic
technique by all health care providers. After washing hands, turn off a handle faucet with a dry paper
towel, and avoid touching the handles with your hands to assist in preventing the transfer of
microorganisms. Wet towels and hands allow the transfer of pathogens from faucet to hands. The
principles and procedures for washing hands are universal and apply to all members of health care
teams. Being resourceful and aware of the cost of health care is important, but taking shortcuts that
may endanger an individual’s health is not a prudent practice.

157
Q

The nurse is performing hand hygiene before assisting a health care provider with insertion of a
chest tube. While washing hands, the nurse touches the sink. Which action will the nurse take next?
a. Inform the health care provider and recruit another nurse to assist.
b. Rinse and dry hands, and begin assisting the health care provider.
c. Extend the handwashing procedure to 5 minutes.
d. Repeat handwashing using antiseptic soap.

A

ANS: D
The inside of the sink and the edges of the sink, faucet, and handles are considered contaminated
areas. If the hands touch any of these areas during handwashing, repeat the handwashing procedure
utilizing antiseptic soap. There is no need to inform the health care provider or be relieved of this
assignment. If the hands are contaminated when touching the sink, drying hands and proceeding with
the procedure could possibly contaminate and contribute to increased microbial counts during the
procedure, resulting in infection for the patient. Extending the time for washing the hands (although
this is what will happen when the procedure is repeated) is not the focus. The focus is to repeat the
whole hand hygiene procedure utilizing antiseptic soap.

158
Q

The home health nurse is teaching a patient and family about hand hygiene in the home. Which
situation will cause the nurse to emphasize washing hands before and after?
a. Shaking hands
b. Performing treatments
c. Opening the refrigerator
d. Working on a computer

A

ANS: B
Patients and family members should perform hand hygiene before and after treatments and when
coming in contact with body fluids. Shaking hands does not require washing of hands before and
after. Washing hands before and after opening the refrigerator and using the computer is not required.

159
Q

The patient and the nurse are discussing Rickettsia rickettsii—Rocky Mountain spotted fever.
Which patient statement to the nurse indicates understanding regarding the mode of transmission for
this disease?
a. “When camping, I will use sunscreen.”
b. “When camping, I will drink bottled water.”
c. “When camping, I will wear insect repellent.”
d. “When camping, I will wash my hands with hand gel.”

A

ANS: C
Rocky Mountain spotted fever is caused by bacteria transmitted by the bite of ticks. Wearing a
repellent that is designed for repelling ticks, mosquitoes, and other insects can help in preventing
transmission of this disease. Drinking plenty of uncontaminated water, wearing sunscreen, and using
alcohol-based hand gels for cleaning hands are all important activities to participate in while
camping, but they do not contribute to or prevent transmission of this disease.

160
Q

The nurse is providing an educational session for a group of preschool workers. The nurse reminds
the group about the most important thing to do to prevent the spread of infection. Which information
did the nurse share with the preschool workers?
a. Encourage preschool children to eat a nutritious diet.
b. Suggest that parents provide a multivitamin to the children.
c. Clean the toys every afternoon before putting them away.
d. Wash their hands between each interaction with children.

A

ANS: D
The single most important thing that individuals can do to prevent the spread of infection is to wash
their hands before and after eating, going to the bathroom, changing a diaper, and wiping a nose and
between touching each individual child. It is important for preschool children to have a nutritious
diet; a healthy individual can fight infection more effectively. A health care provider, along with the
parent, makes decisions about dietary supplements. Cleaning the toys can decrease the number of
pathogens but is not the most important thing to do in this scenario.

161
Q

The nurse is admitting a patient with an infectious disease process. Which question will
be most appropriate for a nurse to ask about the patient’s susceptibility to this infectiousprocess?
a. “Do you have a spouse?”
b. “Do you have a chronic disease?”
c. “Do you have any children living in the home?”
d. “Do you have any religious beliefs that will influence your care?”

A

ANS: B
Multiple factors influence a patient’s susceptibility to infection. Patients with chronic diseases such
as diabetes mellitus and multiple sclerosis are also more susceptible to infection because of general
debilitation and nutritional impairment. Other factors include age, nutritional status, trauma, and
smoking. The other questions are part of an admission assessment process but are not pertinent to the
infectious disease process

162
Q

The nurse is caring for a group of medical-surgical patients. Which patient is most at risk for
developing an infection?
a. A patient who is in observation for chest pain
b. A patient who has been admitted with dehydration
c. A patient who is recovering from a right total hip surgery
d. A patient who has been admitted for stabilization of heart problems

A

ANS: C
The patient who is recovering from a right total hip surgery has a large incision from the surgery.
This break in the skin increases the likelihood of infection. Any break in the integrity of the skin and
mucous membranes allows pathogens to enter and exit the body. The patient has had anesthesia,
which depresses the respiratory system and has the potential to decrease the expansion of alveoli and
to increase the chance of infection in the respiratory system. A patient who is having chest pain,
experiencing dehydration, or being admitted with heart problems does not have open incisions that
break the skin; therefore, his or her infection risk is lower.

163
Q

The nurse is caring for a patient who is susceptible to infection. Which instruction will the nurse
include in an educational session to decrease the risk ofinfection?
a. Teaching the patient about fall prevention
b. Teaching the patient to take a temperature
c. Teaching the patient to select nutritious foods
d. Teaching the patient about the effects of alcoho

A

ANS: C
A patient’s nutritional health directly influences susceptibility to infection. A reduction in the intake
of protein and other nutrients such as carbohydrates and fats reduces body defenses against infection
and impairs wound healing. This is the only teaching point that directly influences risk. Teaching the
patient how to take a temperature can help the patient assess if there is a fever, but it is not related to
decreasing the individual’s risk for infection. Teaching the patient about fall prevention or about the
effects of alcohol does not decrease the risk of infection.

164
Q

The infection control nurse is reviewing data for the medical-surgical unit. The nurse notices an
increase in postoperative infections from Aspergillus. Which type of health care–associated infection
will the nurse report?
a. Vector
b. Exogenous
c. Endogenous
d. Suprainfection

A

ANS: B
An exogenous infection comes from microorganisms found outside the individual such
as Salmonella, Clostridiumtetani, and Aspergillus. They do not exist as normal floras. A vector
transmits microorganisms and is usually a type of insect or organism. Endogenous infection occurs
when part of the patient’s flora becomes altered and an overgrowth results (e.g., staphylococci,
enterococci, yeasts, and streptococci). This often happens when a patient receives broad-spectrum
antibiotics that alter the normal floras. A suprainfection develops when broad-spectrum antibiotics
eliminate a wide range of normal flora organisms, not just those causing infection.

165
Q

The patient has contracted a urinary tract infection (UTI) while in the hospital. Which action
will most likely increase the risk of a patient contracting aUTI?
a. Reusing the patient’s graduated receptacle to empty the drainage bag.
b. Allowing the drainage bag port to touch the graduated receptacle.
c. Emptying the urinary drainage bag at least once a shift.
d. Irrigating the catheter infrequently.

A

ANS: B
Allowing the urinary drainage bag port to touch contaminated items (graduated receptacle) may
introduce bacteria into the urinary system and contribute to a urinary tract infection. The urinary
drainage bag should be emptied at least once a shift. Patients should have their own receptacle for
measurement to prevent cross-contamination. Repeated catheter irrigations increase the chance so
irrigating infrequently will be beneficial in reducing the risk.

166
Q

Which nursing action will most likely increase a patient’s risk for developing a health care–
associated infection?
a. Uses surgical aseptic technique to suction an airway
b. Uses a clean technique for inserting a urinary catheter
c. Uses a cleaning stroke from the urinary meatus toward the rectum
d. Uses a sterile bottled solution more than once within a 24-hour period

A

ANS: B
Using clean technique (medical asepsis) to insert a urinary catheter would place the patient at risk for
a health care–associated infection. Urinary catheters need to be inserted using sterile technique,
which is also referred to as surgical asepsis. Surgical aseptic technique (also called sterile technique)
should be used when suctioning an airway because it is considered a sterile body cavity. Washing
from clean to dirty (urinary meatus toward rectum) is correct for decreasing infection risk. Bottled
solutions may be used repeatedly during a 24-hour period; however, special care is needed to ensure
that the solution in the bottle remains sterile. After 24 hours, the solution should be discarded.

167
Q

. The nurse is dressed and is preparing to care for a patient in the perioperative area. The nursehas
scrubbed hands and has donned a sterile gown and gloves. Which action will indicate a break in
sterile technique?
a. Touching clean protective eyewear
b. Standing with hands above waist area
c. Accepting sterile supplies from the surgeon
d. Staying with the sterile table once it is open

A

ANS: A
Touching nonsterile (clean) protective eyewear once gowned and gloved with sterile gown and
gloves would indicate a break in sterile technique. Sterile objects remain sterile only when touched
by another sterile object. Standing with hands folded on the chest is common practice and prevents
arms and hands from touching unsterile objects. Accepting sterile supplies from the surgeon who has
opened them with the appropriate technique is acceptable. Staying with a sterile table once opened is
a common practice to ascertain that no one or nothing has contaminated the table.

168
Q

The nurse is caring for a patient with an incision. Which actions will best indicate an
understanding of medical and surgical asepsis for a sterile dressingchange?
a. Donning clean goggles, gown, and gloves to dress the wound
b. Donning sterile gown and gloves to remove the wound dressing
c. Utilizing clean gloves to remove the dressing and sterile supplies for the new dressing
d. Utilizing clean gloves to remove the dressing and clean supplies for the new dressing

A

ANS: C
Utilize clean gloves (medical asepsis) to remove contaminated dressings and sterile supplies,
including gloves and dressings (surgical asepsis–sterile technique) to reapply sterile dressings.
Wearing sterile gowns and gloves is not necessary when removing soiled dressings. Donning clean
gloves to dress a sterile wound would contaminate the sterile supplies. Utilizing clean supplies for a
sterile dressing would not help in decreasing the number of microbes at the incision site.

169
Q

The nurse is caring for a patient who is at risk for infection. Which action by the nurse indicates
correct understanding about standard precautions?
a. Teaches the patient about good nutrition
b. Dons gloves when wearing artificial nails
c. Disposes an uncapped needle in the designated container
d. Wears eyewear when emptying the urinary drainage bag

A

ANS: D
Standard precautions include the wearing of eyewear whenever there is a possibility of a splash or
splatter, like when emptying the urinary drainage bag. Teaching the patient about good nutrition is
positive but does not apply to standard precautions. Standard precautions apply to contact with blood,
body fluid (except sweat), nonintact skin, and mucous membranes from all patients. Artificial nails
are not worn when using standard precautions. Any needles should be disposed of uncapped, or a
mechanical safety device is activated for recapping.

170
Q

The nurse is caring for a patient who has just delivered a neonate. The nurse is checking the
patient for excessive vaginal drainage. Which precaution will the nurse use?
a. Contact
b. Droplet
c. Standard
d. Protective environment

A

ANS: C
Standard precautions apply to contact with blood, body fluid, nonintact skin, and mucous membranes
of all patients. Contact precautions apply to individuals with infections that can be transmitted by
direct or indirect contact. Protective environment precautions apply to individuals who have
undergone transplantations and gene therapy. Droplet precautions focus on diseases that are
transmitted by large droplets.

171
Q

The nurse is caring for a patient who becomes nauseated and vomits without warning. The nurse
has contaminated hands. Which action is best for the nurse to take next?
a. Wash hands with an antimicrobial soap and water.
b. Clean hands with wipes from the bedside table.
c. Use an alcohol-based waterless hand gel.
d. Wipe hands with a dry paper towel.

A

ANS: A
The Centers for Disease Control and Prevention (CDC) recommends that when hands are visibly
soiled, one should wash with a non-antimicrobial soap or with antimicrobial soap. Cleaning hands
with wipes or using waterless hand gel does not meet this standard. If hands are not visibly soiled,
use an alcohol-based waterless antiseptic agent for routinely decontaminating hands. Wiping hands
with a dry paper towel will occur after the nurse has washed both hands.

172
Q

The nurse on the surgical team and the surgeon have completed a surgery. After donninggloves,
gathering instruments, and placing in the transport carrier, what is the next step in handling the
instruments used during the procedure?
a. Sending to central sterile for cleaning and sterilization
b. Sending to central sterile for cleaning and disinfection
c. Sending to central sterile for cleaning and boiling
d. Sending to central sterile for cleaning

A

ANS: A
Surgical instruments need to be cleaned and sterilized. Disinfecting, boiling, or cleaning is not
utilized on critical items that will be reused on patients in the hospital environment. Items that are
used on sterile tissue or in the vascular system present a high risk of infection if they become
contaminated with bacteria.

173
Q

The nurse is observing a family member changing a dressing for a patient in the home health
environment. Which observation indicates the family member has a correct understanding of how to
manage contaminated dressings?
a. The family member places the used dressings in a plastic bag.
b. The family member saves part of the dressing because it is clean.
c. The family member removes gloves and gathers items for disposal.
d. The family member wraps the used dressing in toilet tissue before placing in trash.

A

ANS: A
Contaminated dressings and other infectious, disposable items should be placed in impervious plastic
or brown paper bags and then disposed of properly in garbage containers. Gloves should be worn
during this process. Parts of the dressing should not be saved, even though they may seem clean,
because microbes may be present.

174
Q

The home health nurse is teaching a patient and family about hand hygiene in the home. Which
situation will cause the nurse to emphasize washing hands before and after?
a. Shaking hands
b. Performing treatments
c. Opening the refrigerator
d. Working on a computer

A

ANS: B
Patients and family members should perform hand hygiene before and after treatments and when
coming in contact with body fluids. Shaking hands does not require washing of hands before and
after. Washing hands before and after opening the refrigerator and using the computer is not required.

175
Q

The nurse is caring for a patient on contact precautions. Which action will be most appropriate to
prevent the spread of disease?
a. Place the patient in a room with negative airflow.
b. Wear a gown, gloves, face mask, and goggles for interactions with the patient.
c. Transport the patient safely and quickly when going to the radiology department.
d. Use a dedicated blood pressure cuff that stays in the room and is used for that patient only

A

ANS: D
Contact precautions are a type of isolation precaution used for patients with illness that can be
transmitted through direct or indirect contact. Patients who are on contact precautions should have
dedicated equipment wherever possible. This would mean, for example, that one blood pressure cuff
and one stethoscope would stay in the room with the patient and would be used for that patient only.
A gown and gloves may be required for interactions with a patient who is on contact precautions. A
face mask and goggles are not part of contact precautions. A room with negative airflow is needed
for patients placed on airborne precautions; it is not necessary for a patient on contact precautions.
When a patient on contact precautions needs to be transported, the patient should wear clean gown,
and hands cleaned, and the infectious material is contained or covered.

176
Q

The nurse is caring for a patient who has a bloodborne pathogen. The nurse splashes blood above
the glove to intact skin while discontinuing an intravenous (IV) infusion. Which step(s) will thenurse
take next?
a. Obtain an alcohol swab, remove the blood with an alcohol swab, and continue care.
b. Immediately wash the site with soap and running water, and seek guidance from the manager.
c. Do nothing; accidentally getting splashed with blood happens frequently and is part of the job.
d. Delay washing of the site until the nurse is finished providing care to the patient.

A

ANS: B
After getting splashed with blood from a patient who has a known bloodborne pathogen, it is
important to cleanse the site immediately and thoroughly with soap and running water and notify the
manager for guidance on next steps in the process. Removing the blood with an alcohol swab,
delaying washing, and doing nothing because the splash was to intact skin could possibly spread the
blood within the room and could spread the infection. Contain contamination immediately to prevent
contact spread.

177
Q

The nurse is caring for a patient in protective environment. Which actions will the nurse take?
(Select all that apply.)
a. Wear an N95 respirator when entering the patient’s room.
b. Maintain airflow rate greater than 12 air exchanges/hr.
c. Place in special room with negative-pressure airflow.
d. Open drapes during the daytime.
e. Listen to the patient’s interests.
f. Place dried flowers in a plastic vase.

A

ANS: B, D, E
This form of isolation requires a specialized room with positive airflow. The airflow rate is set at
greater than 12 air exchanges/hr, and all air is filtered through a HEPA filter. Isolation disrupts
normal social relationships with visitors and caregivers. Take the opportunity to listen to a patient’s
concerns or interests. Open drapes or shades and remove excess supplies and equipment. Patients are
not allowed to have dried or fresh flowers or potted plants in these rooms. All health care personnel
wear an N95 respirator every time they enter the room for patients, and a private room with negative
airflow is required for patients on airborne precautions.

178
Q

The nurse is assessing a new patient admitted to home health. Which questions will
be most appropriate for the nurse to ask to determine the risk of infection? (Select all that apply.)
a. “Can you explain the risk for infection in your home?”
b. “Have you traveled outside of the United States?”
c. “Will you demonstrate how to wash your hands?”
d. “What are the signs and symptoms of infection?”
e. “Are you able to walk to the mailbox?”
f. “Who runs errands for you?”

A

ANS: A, B, C, D
In the home setting, the objective is that the patient and/or family will utilize proper infection control
techniques. Asking the patient and family about handwashing, risk of infection, recent travel, and
signs and symptoms of infection is important in evaluating the patient’s knowledge based on
infection control strategies. Activity assessment is important for evaluation of the overall status of the
patient, and knowing who runs errands gives you information on who is helping to meet the needs of
the patient, but neither of these relates to decreasing the risk of infection.

179
Q

A home health nurse is performing a home assessment for safety. Which comment by the patient
will cause the nurse to follow up?
a. “Every December is the time to change batteries on the carbon monoxide detector.”
b. “I will schedule an appointment with a chimney inspector next week.”
c. “If I feel dizzy when using the heater, I need to have it inspected.”
d. “When it is cold outside in the winter, I will use a nonvented furnace.”

A

ANS: D
Using a nonvented heater introduces carbon monoxide into the environment and decreases the
available oxygen for human consumption and the nurse should follow up to correct this behavior.
Checking the chimney and heater,changing the batteries on the detector, and following up on
symptoms such as dizziness, nausea, and fatigue are all statements that are safe and appropriate and
need no follow-up.

180
Q

The nurse is caring for a hospitalized patient. Which behavior alerts the nurse to consider the
need for a restraint?
a. The patient refuses to call for help to go to the bathroom.
b. The patient continues to remove the nasogastric tube.
c. The patient gets confused regarding the time at night.
d. The patient does not sleep and continues to ask for items.

A

ANS: B
Patients who are confused, disoriented, and wander or repeatedly fall or try to remove medical
devices (e.g., oxygen equipment, IV lines, or dressings) often require the temporary use of restraints
to keep them safe. Restraints can be used to prevent interruption of therapy such as traction, IV
infusions, NG tube feeding, or Foley catheterization. Refusing to call for help, although unsafe, is not
a reason for restraint. Getting confused at night regarding the time or not sleeping and bothering the
staff to ask for items is not a reason for restraint.

181
Q

The nurse is caring for a patient who suddenly becomes confused and tries to remove an
intravenous (IV) infusion. Which priority action will the nurse take?
a. Assess the patient.
b. Gather restraint supplies.
c. Try alternatives to restraint.
d. Call the health care provider for a restraint order.

A

ANS: A
When a patient becomes suddenly confused, the priority is to assess the patient, to identify the reason
for change in behavior, and to try to eliminate the cause. If interventions and alternatives are
exhausted, the nurse working with the health care provider may determine the need for restraints.

182
Q

The patient is confused, is trying to get out of bed, and is pulling at the intravenousinfusion

tubing. Which nursing diagnosis will the nurse add to the care plan?
a. Impaired home maintenance
b. Deficient knowledge
c. Risk for poisoning
d. Risk for injury

A

ANS: D
The patient’s behaviors support the nursing diagnosis of Risk for injury. The patient is confused, is
pulling at the intravenous line, and is trying to climb out of bed. Injury could result if the patient falls
out of bed or begins to bleed from a pulled line. Nothing in the scenario indicates that this patient
lacks knowledge or is at risk for poisoning. Nothing in the scenario refers to the patient’s home
maintenance.

183
Q

A confused patient is restless and continues to try to remove the oxygen cannula and urinary

catheter. What is the priority nursing diagnosis and intervention to implement for thispatient?
a. Risk for injury: Check on patient every 15 minutes.
b. Risk for suffocation: Place “Oxygen in Use” sign on door.
c. Disturbed body image: Encourage patient to express concerns about body.
d. Deficient knowledge: Explain the purpose of oxygen therapy and the urinary catheter.

A

ANS: A
The priority nursing diagnosis is Risk for injury. This patient could cause harm to self by interrupting
the oxygen therapy or by damaging the urethra by pulling the urinary catheter out. Before restraining
a patient, it is important to implement and exhaust alternatives to restraint. Alternatives can include
more frequent observations. This patient may have deficient knowledge; educating the patient about
treatments could be considered as an alternative to restraints. However, the nursing diagnosis of
highest priority is risk for injury. This scenario does not indicate that the patient has a disturbed body
image or that the patient is at risk for suffocation.

184
Q

The nurse is caring for a hospitalized patient. Which behavior alerts the nurse to consider the
need for a restraint?
a. The patient refuses to call for help to go to the bathroom.
b. The patient continues to remove the nasogastric tube.
c. The patient gets confused regarding the time at night.
d. The patient does not sleep and continues to ask for items.

A

ANS: B
Patients who are confused, disoriented, and wander or repeatedly fall or try to remove medical
devices (e.g., oxygen equipment, IV lines, or dressings) often require the temporary use of restraints
to keep them safe. Restraints can be used to prevent interruption of therapy such as traction, IV
infusions, NG tube feeding, or Foley catheterization. Refusing to call for help, although unsafe, is not
a reason for restraint. Getting confused at night regarding the time or not sleeping and bothering the
staff to ask for items is not a reason for restraint.

185
Q

The nurse is trying to use alternatives rather than restrain a patient. Which finding will cause the
nurse to determine the alternative is working?
a. The patient continues to get up from the chair at the nurses’ station.
b. The patient gets restless when the sitter leaves for lunch.
c. The patient folds three washcloths over and over.
d. The patient apologizes for being “such a bother.”

A

ANS: C
Restraint alternatives include more frequent observations, social interaction such as involvement of
family during visitation, frequent reorientation, regular exercise, and the introduction of familiar and
meaningful stimuli (e.g., involve in hobbies such as knitting or crocheting or looking at family
photos) within the environment or folding washcloths. Getting up constantly can be cause for
concern. Apologizing is not an alternative to restraints. Getting restless when the sitter leaves
indicates the alternative is not working

186
Q

The nurse is caring for a patient who suddenly becomes confused and tries to remove an
intravenous (IV) infusion. Which priority action will the nurse take?
a. Assess the patient.
b. Gather restraint supplies.
c. Try alternatives to restraint.
d. Call the health care provider for a restraint order.

A

ANS: A
When a patient becomes suddenly confused, the priority is to assess the patient, to identify the reason
for change in behavior, and to try to eliminate the cause. If interventions and alternatives are
exhausted, the nurse working with the health care provider may determine the need for restraints

187
Q

A patient may need restraints. Which task can the nurse delegate to a nursing assistivepersonnel?

a. Determining the need for restraints
b. Assessing the patient’s orientation
c. Obtaining an order for a restraint
d. Applying the restraint

A

ANS: D
The application and routine checking of a restraint can be delegated to nursing assistive personnel.
The skill of assessing a patient’s behavior, orientation to the environment, need for restraints, and
appropriate use cannot be delegated. A nurse must obtain an order from a health care provider.

188
Q

A patient has an ankle restraint applied. Upon assessment the nurse finds the toes a lightblue

color. Which action will the nurse take next?
a. Remove the restraint.
b. Place a blanket over the feet.
c. Immediately do a complete head-to-toe neurologic assessment.
d. Take the patient’s blood pressure, pulse, temperature, and respiratory rate

A

ANS: A
If the patient has altered neurovascular status of an extremity such as cyanosis, pallor, and coldness
of skin or complains of tingling, pain, or numbness, remove the restraint immediately and notify the
health care provider. Light blue is cyanosis, indicating the restraints are too tight, not that the patient
is cold and needs a blanket. A complete head-to-toe neurological assessment is not needed at this
time. The nurse can take vital signs after the restraint is removed.

189
Q

A patient requires restraints after alternatives are not successful. The nurse is reviewing the orders.
Which findings indicate to the nurse the order is legal and appropriate for safe care? (Select allthat
apply.)
a. Health care provider orders restraints prn (as needed).
b. Health care provider writes the type and location of the restraint.
c. Health care provider renews orders for restraints every 24 hours.
d. Health care provider performs a face-to-face assessment prior to the order.
e. Health care provider specifies the duration and circumstances under which the restraint will be used.

A

ANS: B, D, E
A physician’s/health care provider’s order is required, based on a face-to-face assessment of the
patient. The order must be current, state the type and location of restraint, and specify the duration
and circumstances under which it will be used. These orders need to be renewed within a specific
time frame according to the policy of the agency. In hospital settings each original restraint order and
renewal is limited to 8 hours for adults, 2 hours for ages 9 through 17, and 1 hour for children under
age 9. Restraints are not to be ordered prn (as needed).

190
Q

The nurse is caring for a patient in restraints. Which essential information will the nurse document
in the patient’s medical record to provide safe care? (Select all that apply.)
a. One family member has gone to lunch.
b. Patient is placed in bilateral wrist restraints at 0815.
c. Bilateral radial pulses present, 2+, hands warm to touch
d. Straps with quick-release buckle attached to bed side rails
e. Attempts to distract the patient with television are unsuccessful.
f. Released from restraints, active range-of-motion exercises completed

A

ANS: B, C, E, F
Proper documentation, including the behaviors that necessitated the
application of restraints, the procedure used in restraining, the condition
of the body part restrained (e.g., circulation to hand), and the evaluation
of the patient response, is essential. Record nursing interventions,
including restraint alternatives tried, in nurses’ notes. Record purpose for
restraint, type and location of restraint used, time applied and
discontinued, times restraint was released, and routine observations (e.g.,
skin color, pulses, sensation, vital signs, and behavior) in nurses’ notes
and flow sheets. Straps are not attached to side rails. Comments about the
activities of one family member are not necessarily required in nursing
documentation of restraints.

191
Q

. The nurse is monitoring for the four categories of risk that have been identified in the health care
environment. Which examples will alert the nurse that these safety risks areoccurring?
a. Tile floors, cold food, scratchy linen, and noisy alarms
b. Dirty floors, hallways blocked, medication room locked, and alarms set
c. Carpeted floors, ice machine empty, unlocked supply cabinet, and call light in reach
d.
Wet floors unmarked, patient pinching fingers in door, failure to use lift for patient, and alarms not functi
properly

A

ANS: D
Specific risks to a patient’s safety within the health care environment include falls, patient-inherent
accidents, procedure-related accidents, and equipment-related accidents. Wet floors contribute to
falls, pinching finger in door is patient inherent, failure to use the lift is procedure related, and an
alarm not functioning properly is equipment related. Tile floors and carpeted or dirty floors do not
necessarily contribute to falls. Cold food, ice machine empty, and hallways blocked are not patientinherent issues in the hospital setting but are more of patient satisfaction, infection control, or fire
safety issues. Scratchy linen, unlocked supply cabinet, and medication room locked are not
procedure-related accidents. These are patient satisfaction issues and control of supply issues and are
examples of actually following a procedure correctly. Noisy alarms, call light within reach, and
alarms set are not equipment-related accidents but are examples of following a procedure correctly.

192
Q

Which activity will cause the nurse to monitor for equipment-related accidents?

a. Uses a patient-controlled analgesic pump
b. Uses a computer-based documentation record
c. Uses a measuring device that measures urine
d. Uses a manual medication-dispensing device

A

ANS: A
Accidents that are equipment related result from the malfunction, disrepair, or misuse of equipment
or from an electrical hazard. To avoid rapid infusion of IV fluids, all general-use and patient controlled analgesic pumps need to have free-flow protection devices. Measuring devices used by the
nurse to measure urine, computer documentation, and manual dispensing devices can break or
malfunction but are not used directly on a patient and are considered procedure-related accidents.

193
Q

A home health nurse is teaching a family to prevent electrical shock. Which information will the
nurse include in the teaching session?
a. Run wires under the carpet.
b. Disconnect items before cleaning.
c. Grasp the cord when unplugging items.
d. Use masking tape to secure cords to the floor.

A

ANS: B
A guideline to prevent electrical shock is to disconnect items before cleaning. Do not run wires under
carpeting. Grasp the plug, not the cord, when unplugging items. Use electrical tape to secure the cord
to the floor, preferably against baseboards.

194
Q

A home health nurse is assessing the home for fire safety. Which information from the family will
cause the nurse to intervene? (Select all that apply.)
a. Smoking in bed helps me relax and fall asleep.
b. We never leave candles burning when we are gone.
c. We use the same space heater my grandparents used.
d. We use the RACE method when using the fire extinguisher.
e. There is a fire extinguisher in the kitchen and garage workshop.

A

ANS: A, C, D
Incorrect information will cause the nurse to intervene. Accidental home fires typically result from
smoking in bed. Advise families to only purchase newer model space heaters that have all of the
current safety features. The PASS method is used for fire extinguishers. All the rest are correct and
do not require follow-up. Candles should not be left burning when no one is home. Keep a fire
extinguisher in the kitchen, near the furnace, and in the garage.

195
Q

formation about the alveoli’s function will the nurse share with the patient?

a. Carries out gas exchange
b. Regulates tidal volume
c. Produces hemoglobin
d. Stores oxygen

A

ANS: A
The alveolus is a capillary membrane that allows gas exchange of oxygen and carbon dioxide during
respiration. The alveoli do not store oxygen, regulate tidal volume, or produce hemoglobin.

196
Q

The nurse is teaching about the process of exchanging gases through the alveolar capillary

membrane. Which term will the nurse use to describe this process?
a. Ventilation
b. Surfactant
c. Perfusion
d. Diffusion

A

ANS: D
Diffusion is the process of gases exchanging across the alveoli and capillaries of body
tissues. Ventilation is the process of moving gases into and out of the lungs. Surfactant is a chemical
produced in the lungs to maintain the surface tension of the alveoli and keep them from collapsing.
Perfusion is the ability of the cardiovascular system to carry oxygenated blood to tissues and return
deoxygenated blood to the heart.

197
Q

While performing an assessment, the nurse hears crackles in the patient’s lung fields. The nurse
also learns that the patient is sleeping on three pillows to help with the difficulty breathing duringthe
night. Which condition will the nurse most likely observe written in the patient’s medicalrecord?
a. Atrial fibrillation
b. Myocardial ischemia
c. Left-sided heart failure
d. Right-sided heart failure

A

ANS: C
Left-sided heart failure results in pulmonary congestion, the signs and symptoms of which include
shortness of breath, cough, crackles, and paroxysmal nocturnal dyspnea (difficulty breathing when
lying flat). Right-sided heart failure is systemic and results in peripheral edema, weight gain, and
distended neck veins. Atrial fibrillation is often described as an irregularly irregular rhythm; rhythm
is irregular because of the multiple pacemaker sites. Myocardial ischemia results when the supply of
blood to the myocardium from the coronary arteries is insufficient to meet myocardial oxygen
demands, producing angina or myocardial infarction.

198
Q

A nurse is teaching a health class about the heart. Which information from the class members
indicates teaching by the nurse is successful for the flow of blood through the heart, starting inthe
right atrium?
a. Right ventricle, left ventricle, left atrium
b. Left atrium, right ventricle, left ventricle
c. Right ventricle, left atrium, left ventricle
d. Left atrium, left ventricle, right ventricle

A

ANS: C
Unoxygenated blood flows through the venae cavae into the right atrium, where it is pumped down to
the right ventricle; the blood is then pumped out the pulmonary artery and is returned oxygenated via
the pulmonary vein to the left atrium, where it flows to the left ventricle and is pumped out to the rest
of the body via the aorta.

199
Q
The patient has right-sided heart failure. Which finding will the nurse expect when performing an
 assessment?
a. Peripheral edema
b. Basilar crackles
c. Chest pain
d. Cyanosis
A

ANS: A
Right-sided heart failure results from inability of the right side of the heart to pump effectively,
leading to a systemic backup. Peripheral edema, distended neck veins, and weight gain are signs of
right-sided failure. Basilar crackles can indicate pulmonary congestion from left-sided heart failure.
Cyanosis and chest pain result from inadequate tissue perfusion.

200
Q
The nurse is caring for a patient with fluid volume overload. Which physiological effect doesthe
 nurse most likely expect?
a. Increased preload
b. Increased heart rate
c. Decreased afterload
d. Decreased tissue perfusion
A

ANS: A
Preload refers to the amount of blood in the left ventricle at the end of diastole; an increase in
circulating volume would increase the preload of the heart. Afterload refers to resistance; increased
pressure would lead to increased resistance, and afterload would increase. A decrease in tissue
perfusion would be seen with hypovolemia. A decrease in fluid volume would cause an increase in
heart rate as the body is attempting to increase cardiac output.

201
Q
A nurse is teaching about risk factors for cardiopulmonary disease. Which risk factor should the
 nurse describe as modifiable?
a. Stress
b. Allergies
c. Family history
d. Gender
A

ANS: A
Young and middle-age adults are exposed to multiple cardiopulmonary risk factors: an unhealthy
diet, lack of exercise, stress, over-the-counter and prescription drugs not used as intended, illegal
substances, and smoking. Reducing these modifiable factors decreases a patient’s risk for cardiac or
pulmonary diseases. A nonmodifiable risk factor is family history; determine familial risk factors
such as a family history of lung cancer or cardiovascular disease. Other nonmodifiable risk factors
include allergies and gender.

202
Q

The home health nurse recommends that a patient with respiratory problems install a carbon
monoxide detector in the home. What is the rationale for the nurse’s action?
a. Carbon monoxide detectors are required by law in the home.
b. Carbon monoxide tightly binds to hemoglobin, causing hypoxia.
c. Carbon monoxide signals the cerebral cortex to cease ventilations.
d. Carbon monoxide combines with oxygen in the body and produces a deadly toxin.

A

ANS: B
Carbon monoxide binds tightly to hemoglobin; therefore, oxygen is not able to bind to hemoglobin
and be transported to tissues, causing hypoxia. A carbon monoxide detector is not required by law,
does not signal the cerebral cortex to cease ventilations, and does not combine with oxygen but with
hemoglobin to produce a toxin.

203
Q
The nurse is caring for a patient with respiratory problems. Which assessment finding indicates a
 late sign of hypoxia?
a. Elevated blood pressure
b. Increased pulse rate
c. Restlessness
d. Cyanosis
A

ANS: D
Cyanosis, blue discoloration of the skin and mucous membranes caused by the presence of
desaturated hemoglobin in capillaries, is a late sign of hypoxia. Elevated blood pressure, increased
pulse rate, and restlessness are early signs of hypoxia.

204
Q

A nurse teaches a patient about atelectasis. Which statement by the patient indicates an
understanding of atelectasis?
a. “Atelectasis affects only those with chronic conditions such as emphysema.”
b. “It is important to do breathing exercises every hour to prevent atelectasis.”
c. “If I develop atelectasis, I will need a chest tube to drain excess fluid.”
d. “Hyperventilation will open up my alveoli, preventing atelectasis.”

A

ANS: B
Atelectasis develops when alveoli do not expand. Breathing exercises, especially deep breathing and
incentive spirometry, increase lung volume and open the airways, preventing atelectasis. Deep
breathing also opens the pores of Kohn between alveoli to allow sharing of oxygen between alveoli.
Atelectasis can affect anyone who does not deep breathe. A chest tube is for pneumothorax or
hemothorax. It is deep breathing, not hyperventilation, that prevents atelectasis.

205
Q

A nurse is caring for a 5-year-old patient whose temperature is 101.2° F. The nurse expects this
patient to hyperventilate. Which factor does the nurse remember when planning care for this typeof
hyperventilation?
a. Anxiety over illness
b. Decreased drive to breathe
c. Increased metabolic demands
d. Infection destroying lung tissues

A

ANS: C
Increased body temperature (fever) increases the metabolic rate, thereby increasing carbon dioxide
production. The increased carbon dioxide level stimulates an increase in the patient’s rate and depth
of respiration, causing hyperventilation. Anxiety can cause hyperventilation, but this is not the direct
cause from a fever. Sleep causes a decreased respiratory drive; hyperventilation speeds up breathing.
The cause of the fever in this question is unknown.

206
Q

A patient with chronic obstructive pulmonary disease (COPD) asks the nurse why clubbing

occurs. Which response by the nurse is most therapeutic?
a. “Your disease doesn’t send enough oxygen to your fingers.”
b. “Your disease affects both your lungs and your heart, and not enough blood is being pumped.”
c. “Your disease will be helped if you pursed-lip breathe.”
d. “Your disease often makes patients lose mental status.”

A

ANS: A
Clubbing of the nail bed can occur with COPD and other diseases that cause prolonged oxygen
deficiency or chronic hypoxemia. Pursed-lipped breathing helps the alveoli stay open but is not the
cause of clubbing. Loss of mental status is not a normal finding with COPD and will not result in
clubbing. Low oxygen and not low circulating blood volume is the problem in COPD that results in
clubbing.

207
Q

A nurse is caring for a patient who has poor tissue perfusion as the result of hypertension. When
the patient asks what to eat for breakfast, which meal should the nurse suggest?
a. A cup of nonfat yogurt with granola and a handful of dried apricots
b. Whole wheat toast with butter and a side of bacon
c. A bowl of cereal with whole milk and a banana
d. Omelet with sausage, cheese, and onions

A

ANS: A
A 2000-calorie diet of fruits, vegetables, and low-fat dairy foods that are high in fiber, potassium,
calcium, and magnesium and low in saturated and total fat helps prevent and reduce the effects of
hypertension. Nonfat yogurt with granola is a good source of calcium, fiber, and potassium; dried
apricots add a second source of potassium. Although cereal and a banana provide fiber and
potassium, skim milk should be substituted for whole milk to decrease fat. An omelet with sausage
and cheese is high in fat. Butter and bacon are high in fat.

208
Q
A nurse is caring for a patient with left-sided hemiparesis who has developed bronchitis and has a
heart rate of 105 beats/min, blood pressure of 156/90 mm Hg, and respiration rate of 30 breaths/min.
 Which nursing diagnosis is apriority?
a.
Risk for skin breakdown
b.
Impaired gas exchange
c.
Activity intolerance
d. Risk for infection
A

ANS: B
The most important nursing intervention is to maintain airway and circulation for this patient;
therefore, Impaired gas exchange is the first nursing priority. Activity intolerance is a concern but is
not the priority in this case. Risk for skin breakdown and Risk for infection are also important but do
not address an immediate impairment with physiologic integrity

209
Q
A nurse is caring for a patient with left-sided hemiparesis who has developed bronchitis and has a
heart rate of 105 beats/min, blood pressure of 156/90 mm Hg, and respiration rate of 30 breaths/min.
 Which nursing diagnosis is apriority?
a.
Risk for skin breakdown
b.
Impaired gas exchange
c.
Activity intolerance
d. Risk for infection
A

ANS: B
The most important nursing intervention is to maintain airway and circulation for this patient;
therefore, Impaired gas exchange is the first nursing priority. Activity intolerance is a concern but is
not the priority in this case. Risk for skin breakdown and Risk for infection are also important but do
not address an immediate impairment with physiologic integrity

210
Q

The health care provider has ordered a hypotonic intravenous (IV) solution to be administered.
Which IV bag will the nurse prepare?
a. 0.45% sodium chloride (1/2 NS)
b. 0.9% sodium chloride (NS)
c. Lactated Ringer’s (LR)
d. Dextrose 5% in Lactated Ringer’s (D5LR)

A

ANS: A

0.45% sodium chloride is a hypotonic solution. NS and LR are isotonic. D5LR is hypertonic.

211
Q
The nurse is assessing a patient and finds crackles in the lung bases and neck vein distention.
 Which action will the nurse take first?
a. Offer calcium-rich foods.
b. Administer diuretic.
c. Raise head of bed.
d. Increase fluids.
A

ANS: C
The patient is in fluid overload. Raising the head of the bed to ease breathing is the first action.
Offering calcium-rich foods is for hypocalcemia, not fluid overload. Administering a diuretic is
the second action. Increasing fluids is contraindicated and would make the situation worse.

212
Q

A nurse is administering a diuretic to a patient and teaching the patient about foods to increase.
Which food choices by the patient will best indicate successful teaching?
a. Milk and cheese
b. Potatoes and fresh fruit
c. Canned soups and vegetables
d. Whole grains and dark green leafy vegetables

A

ANS: B
Potatoes and fruits are high in potassium. Milk and cheese are high in calcium. Canned soups and
vegetables are high in sodium. Whole grains and dark green leafy vegetables are high in magnesium.

213
Q

A nurse is caring for a diabetic patient with a bowel obstruction and has orders to ensure that the
volume of intake matches the output. In the past 4 hours, the patient received dextrose 5% with 0.9%
sodium chloride through a 22-gauge catheter infusing at 150 mL/hr and has eaten 200 mL of ice
chips. The patient also has an NG suction tube set to low continuous suction that had 300-mL output.
The patient has voided 400 mL of urine. After reporting these values to the health care provider,
which order does the nurse anticipate?
a. Add a potassium supplement to replace loss from output.
b. Decrease the rate of intravenous fluids to 100 mL/hr.
c. Administer a diuretic to prevent fluid volume excess.
d. Discontinue the nasogastric suctioning

A

ANS: A
The total fluid intake and output equals 700 mL, which meets the provider goals. Patients with
nasogastric suctioning are at risk for potassium deficit, so the nurse would anticipate a potassium
supplement to correct this condition. Remember to record half the volume of ice chips when
calculating intake. The other measures would be unnecessary because the net fluid volume is equal.

214
Q
The nurse administers an intravenous (IV) hypertonic solution to a patient. In which direction will
the fluid shift?
a. From intracellular to extracellular
b. From extracellular to intracellular
c. From intravascular to intracellular
d. From intravascular to interstitial
A

ANS: A
Hypertonic solutions will move fluid from the intracellular to the extracellular (intravascular). A
hypertonic solution has a concentration greater than normal body fluids, so water will shift out of
cells because of the osmotic pull of the extra particles. Movement of water from the extracellular
(intravascular) into cells (intracellular) occurs when hypotonic fluids are administered. Distribution
of fluid between intravascular and interstitial spaces occurs by filtration, the net sum of hydrostatic
and osmotic pressures.

215
Q
The nurse receives the patient’s most recent blood work results. Which laboratory value isof
greatest concern?
a. Sodium of 145 mEq/L
b. Calcium of 15.5 mg/dL
c. Potassium of 3.5 mEq/L
d. Chloride of 100 mEq/L
A

ANS: B
Normal calcium range is 8.4 to 10.5 mg/dL; therefore, a value of 15.5 mg/dL is abnormally high and
of concern. The rest of the laboratory values are within their normal ranges: sodium 136 to 145
mEq/L; potassium 3.5 to 5.0 mEq/L; and chloride 98 to 106 mEq/L.

216
Q

.A 2-year-old child is brought into the emergency department after ingesting a medication that
causes respiratory depression. For which acid-base imbalance will the nurse most closely monitor
this child?
a. Respiratory alkalosis
b. Respiratory acidosis
c. Metabolic acidosis
d. Metabolic alkalosis

A

ANS: B
Respiratory depression leads to hypoventilation. Hypoventilation results in retention of CO2 and
respiratory acidosis. Respiratory alkalosis would result from hyperventilation, causing a decrease in
CO2 levels. Metabolic acid-base imbalance would be a result of kidney dysfunction, vomiting,
diarrhea, or other conditions that affect metabolic acids.

217
Q
A patient is experiencing respiratory acidosis. Which organ system is responsible for
compensation in this patient?
a. Renal
b. Endocrine
c. Respiratory
d. Gastrointestinal
A

ANS: A
The kidneys (renal) are responsible for respiratory acidosis compensation. A problem with the
respiratory system causes respiratory acidosis, so another organ system (renal) needs to compensate.
Problems with the gastrointestinal and endocrine systems can cause acid-base imbalances, but these
systems cannot compensate for an existing imbalance.

218
Q

The nurse is caring for a diabetic patient in renal failure who is in metabolic acidosis. Which
laboratory findings are consistent with metabolic acidosis?

a. pH 7.3, PaCO2 36 mm Hg, HCO – 19 mEq/L
b. pH 7.5, PaCO2 35 mm Hg, HCO – 35 mEq/L
c. pH 7.32, PaCO2 47 mm Hg, HCO – 23 mEq/L
d. pH 7.35, PaCO2 40 mm Hg, HCO – 25 mEq/L

A

ANS: A
The laboratory values that reflect metabolic acidosis are pH 7.3, PaCO2 36 mm Hg, HCO – 19 mEq/L.
A laboratory finding of pH 7.5, PaCO2 35 mm Hg, HCO – 35 mEq/L is metabolic alkalosis. pH 7.32,
PaCO2 47 mm Hg, HCO – 23 mEq/L is respiratory acidosis. pH 7.35, PaCO2 40 mm Hg, HCO – 25
mEq/L values are within normal range.

NORMAL RANGES:
ph 7.38-7.42
HCO3 22-28
PaCO2 38-42
SaO2 94-100
219
Q

A chemotherapy patient has gained 5 pounds in 2 days. Which assessment question by the nurse
is mostappropriate?
a. “Are you following any weight loss program?”
b. “How many calories a day do you consume?”
c. “Do you have dry mouth or feel thirsty?”
d. “How many times a day do you urinate?”

A

ANS: D
A rapid gain in weight usually indicates extracellular volume (ECV) excess if the person began with
normal ECV. Asking the patient about urination habits will help determine whether the body is trying
to excrete the excess fluid or if renal dysfunction is contributing to ECV excess. This is too rapid a
weight gain to be dietary; it is fluid retention. Asking about following a weight loss program will not
help determine the cause of the problem. Caloric intake does not account for rapid weight changes.
Dry mouth and thirst accompany ECV deficit, which would be associated with rapid weight loss.

220
Q

A patient presents to the emergency department with reports of vomiting and diarrhea for the past
48 hours. The health care provider orders isotonic intravenous (IV) therapy. Which IV will the nurse
prepare?
a. 0.225% sodium chloride (1/4 NS)
b. 0.45% sodium chloride (1/2 NS)
c. 0.9% sodium chloride (NS)
d. 3% sodium chloride (3% NaCl)

A

ANS: C
Patients with prolonged vomiting and diarrhea become hypovolemic. A solution to replace
extracellular volume is 0.9% sodium chloride, which is an isotonic solution. 0.225% and 0.45%
sodium chloride are hypotonic. 3% sodium chloride is hypertonic.

221
Q

A nurse is caring for a patient who is receiving peripheral intravenous (IV) therapy. When the
nurse is flushing the patient’s peripheral IV, the patient reports pain. Upon assessment, the nurse
notices a red streak that is warm to the touch. What is the nurse’s initial action?
a. Record a phlebitis grade of 4.
b. Assign an infiltration grade.
c. Apply moist compress.
d. Discontinue the IV.

A

ANS: D
The IV site has phlebitis. The nurse should discontinue the IV. The phlebitis score is 3. The site has
phlebitis, not infiltration. A moist compress may be needed after the IV is discontinued.

222
Q

A nurse is caring for a patient who is receiving peripheral intravenous (IV) therapy. When the
nurse is flushing the patient’s peripheral IV, the patient reports pain. Upon assessment, the nurse
notices a red streak that is warm to the touch. What is the nurse’s initial action?
a. Record a phlebitis grade of 4.
b. Assign an infiltration grade.
c. Apply moist compress.
d. Discontinue the IV.

A

ANS: D
The IV site has phlebitis. The nurse should discontinue the IV. The phlebitis score is 3. The site has
phlebitis, not infiltration. A moist compress may be needed after the IV is discontinued.

223
Q

A nurse attends a seminar on nursing theories for caring. Which information from the nurse
indicates a correct understanding of these theories?
a. Benner identifies caring as highly connected involving patient and nurse.
b. Swanson develops four caring processes to convey caring in nursing.
c. Watson’s transcultural caring views inclusion of culture as caring.
d. Leininger’s theory places care before cure and is transformative.

A

ANS: A
Benner believes caring is highly connected involving each nurse-patient encounter. Swanson
developed five caring processes, not four. Watson’s theory places care before cure and is
transformative, whereas Leininger’s transcultural caring views inclusion of culture as caring.

224
Q
A nurse is using Watson’s model to provide care to patients. Which carative factor will the nurse
 use?
a. Maintaining belief
b. Instilling faith-hope
c. Maintaining ethics
d. Instilling values
A

ANS: B
Watson has 10 carative factors, one of which is instilling faith-hope. Maintaining belief is a caring
process of Swanson’s theory. Ethics and values are important in caring but they are not examples of
Watson’s carative factors.

225
Q

The patient is terminal and very near death. When the nurse checks the patient and finds no pulse
or blood pressure, the family begins sobbing and hugging each other. Some family members hold the
patient’s hand. The nurse is overwhelmed by the presence of grief and leaves the room. What isthe
nurse demonstrating?
a. Caring touch
b. Protective touch
c. Therapeutic touch
d. Task-oriented touch

A

ANS: B
Protective touch is also a kind of touch that protects the nurse emotionally. A nurse withdraws or
distances herself or himself from a patient when he or she is unable to tolerate suffering or needs to
escape from a situation that is causing tension. Caring touch is a form of nonverbal communication
that influences a patient’s comfort and security, enhances self-esteem, and improves mental wellbeing. Therapeutic touch is a type of alternative therapy for healing. Task-oriented touch is done
when performing a task or procedure.

226
Q

Which action indicates a nurse is using caring touch with a patient?

a. Inserts a catheter
b. Rubs a patient’s back
c. Prevents a patient from falling
d. Administers an injection

A

ANS: B
Caring touch is the way a nurse holds a patient’s hand, gives a back massage, or gently positions a
patient. Touch that occurs when tasks are being performed, such as insertion of a catheter or
administering an injection, is known as “task-oriented touch.” Touch used to protect the patient
(holding and bracing a patient to avoid a fall) or nurse (withdraws from tension-filled situations) is
known as “protective touch.”

227
Q

The patient is about to undergo a certain procedure and has voiced concern about outcomes and
prognosis. The nurse caring for the patient underwent a similar procedure and stops to listen. Which
response by the nurse may be most beneficial?
a. “I had a similar procedure and I can tell you what I went through.”
b. “I think you’ll be all right, but, of course, there are no sure guarantees.”
c. “I don’t think you have anything to worry about. They do lots of these.”
d. “I can call the doctor and cancel the procedure, if you are really concerned.”

A

ANS: A
When an ill person chooses to tell his story, it involves reaching out to another human being. Telling
the story implies a relationship that develops only if the clinician exchanges his or her stories as well.
Professionals do not routinely take seriously their own need to be known as part of a clinical
relationship. Yet, unless the professional acknowledges this need, there is no reciprocal relationship,
only an interaction. Offering false reassurances and cliches, telling not to worry, or offering to cancel
the procedure does not open up that relationship and dismisses the patient’s concerns.

228
Q
A nurse is providing pain medication to patients after surgery. Which component is key for the
 nurse’s personal philosophy of nursing?
a. Caring
b. Technology
c. Informatics
d. Therapeutics
A

ANS: A
The American Organization of Nurse Executives describes caring and knowledge as the core of
nursing, with caring being a key component of what a nurse brings to a patient experience. While
technology, informatics, and therapeutics are important, they are not the key components of nursing.

229
Q

A nurse provides care that is receptive to patients’ and families’ perceptions of caring. Which
action will the nurse take?
a. Provides clear, accurate information
b. Just performs nursing tasks competently
c. Does as much for the patient as possible
d. Focuses solely on the patient’s diagnosis

A

ANS: A
Research indicates caring behaviors of nurses from the patient’s/families’ perspective include the
following: (1) Providing honest, clear, and accurate information; (2) asking permission before doing
something to a patient; (3) helping patients do as much for themselves as possible; and (4) teaching
the family how to keep the relative physically comfortable. Patients continue to value nurses’
effectiveness in performing tasks, but clearly patients value the affective dimension of nursing care

230
Q

The nurse is caring for a patient who has been sullen and quiet for the past three days. Suddenly,
the patient says, “I’m really nervous about surgery tomorrow, but I’m more worried about how itwill
affect my family.” What should the nurse do first?
a. Assure the patient that everything will be all right.
b. Tell the patient that there is no need to worry.
c. Listen to the patient’s concerns and fears.
d. Inform the patient a social worker is available.

A

ANS: C
Listening to the meaning of what a patient says helps create a mutual relationship. Assuring and
telling a patient not to worry are not truly listening; these do not convey listening. Although
contacting a social worker could be an appropriate measure for this patient, the nurse should first
listen to what the patient is saying.

231
Q

The patient is about to undergo a certain procedure and has voiced concern about outcomes and
prognosis. The nurse caring for the patient underwent a similar procedure and stops to listen.Which
response by the nurse may be most beneficial?
a. “I had a similar procedure and I can tell you what I went through.”
b. “I think you’ll be all right, but, of course, there are no sure guarantees.”
c. “I don’t think you have anything to worry about. They do lots of these.”
d. “I can call the doctor and cancel the procedure, if you are really concerned.”

A

ANS: A
When an ill person chooses to tell his story, it involves reaching out to another human being. Telling
the story implies a relationship that develops only if the clinician exchanges his or her stories as well.
Professionals do not routinely take seriously their own need to be known as part of a clinical
relationship. Yet, unless the professional acknowledges this need, there is no reciprocal relationship,
only an interaction. Offering false reassurances and cliches, telling not to worry, or offering to cancel
the procedure does not open up that relationship and dismisses the patient’s concerns.

232
Q

Which actions by the nurse should be done in order to get to know the patient? (Select all that

apply. )
a. Avoid assumptions
b. Focus on the patient
c. Engage in a caring relationship
d. Form the relationship very quickly
e. Not address spiritual or higher needs

A

ANS: A, B, C
To know a patient means that the nurse avoids assumptions, focuses on the patient, and engages in a
caring relationship with the patient that reveals information and cues that facilitate critical thinking
and clinical judgments. Knowing develops over time as a nurse learns the clinical conditions within a
specialty and the behaviors and physiological responses of patients.

233
Q

Which actions by the nurse indicate compassion and caring to patients? (Select all that apply.)

a. Saying “I’m here”
b. Including the family in care
c. Staying with the patient during a bedside test
d. Relying on monitors and technology
e. Refining work processes on the unit

A

ANS: A, B, C
Our patients tell us that a simple touch, a simple phrase, “I’m here,” or a promise to remain at the
bedside represent caring and compassion. Caring for an individual cannot occur in isolation from that
person’s family. As a nurse it is important to know the family almost as thoroughly as you know a
patient. A reliance on technology and cost-effective health care strategies and efforts to standardize
and refine work processes all undermine the nature of caring.

234
Q

A co-worker asks the nurse to explain spirituality. What is the nurse’s best response?

a. It has a minor effect on health.
b. It is awareness of one’s inner self.
c. It is not as essential as physical needs.
d. It refers to fire or giving of life to a person.

A

ANS: B
Spirituality is often defined as an awareness of one’s inner self and a sense of connection to a higher
being, to nature, or to some purpose greater than oneself. Spirituality is an important factor that helps
individuals achieve the balance needed to maintain health and well-being and to cope with illness.
Florence Nightingale believed that spirituality was a force that provided energy needed to promote a
healthy hospital environment and that caring for a person’s spiritual needs was just as essential as
caring for his or her physical needs. The word spirituality comes from the Latin wordspiritus, which
refers to breath or wind. The spirit gives life to a person.

235
Q

The nurse is admitting a patient to the hospital. The patient is a very spiritual person but does not
practice any specific religion. How will the nurse interpret thisfinding?
a. This indicates a strong religious affiliation.
b. This statement is contradictory.
c. This statement is reasonable.
d. This indicates a lack of hope.

A

ANS: C
The patient’s statement is reasonable and is not contradictory. Many people tend to use the
terms spirituality andreligion interchangeably. Although closely associated, these terms are not
synonymous. Religious practices encompass spirituality, but spirituality does not need to include
religious practice. When a person has the attitude of something to live for and look forward to, hope
is present.

236
Q

A nurse is caring for a Hindu patient. Which action will the nurse take?

a. Allow time to practice the Five Pillars.
b. Allow time to practice Blessingway.
c. Allow time for Holy Communion.
d. Allow time for purity rituals.

A

ANS: D
Hindus practice prayer and purity rituals. Blessingway is a practice of the Navajos that attempts to
remove ill health by means of stories, songs, rituals, prayers, symbols, and sand paintings. Islams
must be able to practice the Five Pillars of Islam. Holy Communion is practiced in the Christian
religion.

237
Q

The nurse and the patient have the same religious affiliation. Which action will the nurse take?

a. Must use a formal assessment tool to determine patient’s beliefs.
b. Assume that both have the same spiritual beliefs.
c. Do not impose personal values on the patient.
d. Skip the spiritual belief assessment

A

ANS: C
It is important not to impose personal value systems on the patient. This is particularly true when the
patient’s values and beliefs are similar to those of the nurse because it then becomes very easy to
make false assumptions. It is not a must to use a formal assessment tool when assessing a patient’s
beliefs. It is important to conduct the spiritual belief assessment; conducting an assessment is
therapeutic because it expresses a level of caring and support.

238
Q
. A nurse makes a connection with the patient when providing spiritual care. Which type of
 connectedness did the nurse experience?
a. Intrapersonal
b. Interpersonal
c. Transpersonal
d. Multi personal
A

ANS: B
Interpersonal means connected with others and the environment. Intrapersonal means connected
within oneself. Transpersonal means connected with God or an unseen higher power. There is no
such term as multi personal for connectedness.

239
Q

A nurse is providing spiritual care to patients. Which action is essential for the nurse to take?

a. Know one’s own personal beliefs.
b. Learn about other religions.
c. Visit churches, temples, mosques, or synagogues.
d. Travel to other areas that do not have the same beliefs.

A

ANS: A
Because each person has a unique spirituality, you need to know your own beliefs so you are able to
care for each patient without bias. While learning about religions, visiting other religious areas of
worship, and traveling to areas that do not have the same beliefs are beneficial, they are not essential.

240
Q

A nurse is evaluating a patient’s spiritual care. Which areas will the nurse include in the evaluation
process? (Select all that apply.)
a. Review the patient’s view of the purpose in life.
b. Ask whether the patient’s expectations were met.
c. Discuss with family and friends the patient’s connectedness.
d. Review the patient’s self-perception regarding spiritual health.
e. Impress on the patient that spiritual health is permanent once obtained.

A

ANS: A, B, C, D
In evaluating care include a review of the patient’s self-perception regarding spiritual health, the
patient’s view of his or her purpose in life, discussion with the family and friends about
connectedness, and determining whether the patient’s expectations were met. Attainment of spiritual
health is a lifelong goal; it is not permanent once obtained.

241
Q

The nurse is caring for a Chinese patient using the Teach-Back technique. Which action by the
nurse indicates successful implementation of thistechnique?
a. Asks, “Does this make sense?”
b. Asks, “Do you think you can do this at home?”
c. Asks, “What will you tell your spouse about changing the dressing?”
d. Asks, “Would you tell me if you don’t understand something so we can go over it?”

A

ANS: C
The Teach-Back technique asks open-ended questions, like what will you tell your spouse about
changing the dressing, to verify a patient’s understanding. When using the Teach-Back technique do
not ask a patient, “Do you understand?” or “Do you have any questions?” Does this make sense and
do you think you can do this at home are closed-ended questions. Would you tell me if you don’t
understand something so we can go over it is not verifying a patient’s understanding about the
teaching.

242
Q
The nurse is caring for an Islam patient who wants a snack. Which action by the nurse
 is most appropriate?
a. Offers a ham sandwich
b. Offers a beef sandwich
c. Offers a kosher sandwich
d. Offers a bacon sandwich
A

ANS: B
Islam religion does allow beef. Islam does not allow pork or alcohol. Ham and bacon are pork.
Kosher is allowed for Judaism.

243
Q

The nurse questions a health care provider’s decision to not tell the patient about a cancer

diagnosis. Which ethical principle is the nurse trying to uphold for the patient?
a. Consequentialism
b. Autonomy
c. Fidelity
d. Justice

A

ANS: B
The nurse is upholding autonomy. Autonomy refers to the freedom to make decisions free of external
control. Respect for patient autonomy refers to the commitment to include patients in decisions about
all aspects of care. Consequentialism is focused on the outcome and is a philosophical approach.
Justice refers to fairness and is most often used in discussions about access to health care resources.
Fidelity refers to the agreement to keep promises.

244
Q

The nurse values autonomy above all other principles. Which patient assignment will the nurse
find most difficult to accept?
a. Older-adult patient who requires dialysis
b. Teenager in labor who requests epidural anesthesia
c. Middle-aged father of three with an advance directive declining life support
d. Family elder who is making the decisions for a young-adult female member

A

ANS: D
Autonomy refers to freedom from external control. A person who values autonomy highly may find
it difficult to accept situations where the patient is not the primary decision maker regarding his or
her care. A teenager requesting an epidural, a father with an advance directive, and an elderly patient
requiring dialysis all describe a patient or family who can make their own decisions and choices
regarding care.

245
Q

A young woman who is pregnant with a fetus exposed to multiple teratogens consents to haveher
fetus undergo serial PUBS (percutaneous umbilical blood sampling) to examine how exposure
affects the fetus over time. Although these tests will not improve the fetus’s outcomes and will
expose it to some risks, the information gathered may help infants in the future. Which ethical
principle is at greatest risk?
a. Fidelity
b. Autonomy
c. Beneficence
d. Nonmaleficence

A

ANS: D
Nonmaleficence is the ethical principle that focuses on avoidance of harm or hurt. Repeated PUBS
may expose the mother and fetus to some risks. Fidelity refers to the agreement to keep promises
(obtain serial PUBS). Autonomy refers to freedom from external control (mother consented), and
beneficence refers to taking positive actions to help others (may help infants in the future).

246
Q

When professionals work together to solve ethical dilemmas, nurses must examine their own

values. What is thebest rationale for thisstep?
a. So fact is separated from opinion
b. So different perspectives are respected
c. So judgmental attitudes can be provoked
d. So the group identifies the one correct solution

A

ANS: B
Values are personal beliefs that influence behavior. To negotiate differences of value, it is important
to be clear about your own values: what you value, why, and how you respect your own values even
as you try to respect those of others whose values differ from yours. Ethical dilemmas are a problem
in that no one right solution exists. It is not to separate fact from opinion. Judgmental attitudes are
not to be used, much less provoked.

247
Q

A nurse is experiencing an ethical dilemma with a patient. Which information indicates the nurse
has a correct understanding of the primary cause of ethicaldilemmas?
a. Unequal power
b. Presence of conflicting values
c. Judgmental perceptions of patients
d. Poor communication with the patient

A

ANS: B
Ethical dilemmas almost always occur in the presence of conflicting values. While unequal power,
judgmental perceptions, and poor communication can contribute to the dilemma, these are not causes
of a dilemma. Without clarification of values, the nurse may not be able to distinguish fact from
opinion or value, and this can lead to judgmental attitudes

248
Q
. A nurse is a member of the ethics committee. Which purposes will the nurse fulfill in this
 committee? (Select all that apply.)
a. Education
b. Case consultation
c. Purchasing power
d. Direct patient care
e. Policy recommendation
A

ANS: A, B, E
An ethics committee devoted to the teaching and processing of ethical issues and dilemmas exists in
most health care facilities. It is generally multidisciplinary and it serves several purposes: education,
policy recommendation, and case consultation. It does not have purchasing power or provide direct
patient care

249
Q

The patient reports to the nurse of being afraid to speak up regarding a desire to end care for fear
of upsetting spouse and children. Which principle in the nursing code of ethics ensures that the nurse
will promote the patient’s cause?
a. Advocacy
b. Responsibility
c. Confidentiality
d. Accountability

A

ANS: A
Nurses advocate for patients when they support the patient’s cause. A nurse’s ability to adequately
advocate for a patient is based on the unique relationship that develops and the opportunity to better
understand the patient’s point of view. Responsibility refers to respecting one’s professional
obligations and following through on promises. Confidentiality deals with privacy issues, and
accountability refers to answering for one’s actions.

250
Q

. The nurse has become aware of missing narcotics in the patient care area. Which ethical principle
obligates the nurse to report the missing medications?
a. Advocacy
b. Responsibility
c. Confidentiality
d. Accountability

A

ANS: B
Responsibility refers to one’s willingness to respect and adhere to one’s professional obligations. Itis
the nurse’s responsibility to report missing narcotics. Accountability refers to the ability to answer
for one’s actions. Advocacy refers to the support of a particular cause. The concept of confidentiality
is very important in health care and involves protecting patients’ personal health information.

251
Q

The nurse hears a health care provider say to the charge nurse that a certain nurse cannot care for
patients because the nurse is stupid and won’t follow orders. The health care provider also writesin
the patient’s medical records that the same nurse, by name, is not to care for any of thepatients
because of incompetence. Which torts has the health care provider committed? (Select all that apply.)
a. Libel
b. Slander
c. Assault
d. Battery
e. Invasion of privacy

A

ANS: A, B
Slander occurred when the health care provider spoke falsely about the nurse, and libel occurred
when the health care provider wrote false information in the chart. Both of these situations could
cause problems for the nurse’s reputation. Invasion of privacy is the release of a patient’s medical
information to an unauthorized person such as a member of the press, the patient’s employer, or the
patient’s family. Assault is any action that places a person in reasonable fear of harmful, imminent,
or unwelcome contact. No actual contact is required for an assault to occur. Battery is any intentional
touching without consent.

252
Q

A new nurse notes that the health care unit keeps a listing of patient names in a closed bookbehind
the front desk of the nursing station so patients can be located easily. Which action
is most appropriate for the nurse to take?
a.
Talk with the nurse manager about the listing being a violation of the Health Insurance Portability and
Accountability Act (HIPAA).
b. Use the book as needed while keeping it away from individuals not involved in patient care.
c. Move the book to the upper ledge of the nursing station for easier access.
d. Ask the nurse manager to move the book to a more secluded area.

A

ANS: B
The book is located where only staff would have access so the nurse can use the book as needed. The
privacy section of the HIPAA provides standards regarding accountability in the health care setting.
These rules include patient rights to consent to the use and disclosure of their protected health
information, to inspect and copy their medical record, and to amend mistaken or incomplete
information. It is not the responsibility of the new nurse to move items used by others on the patient
unit. The listing is protected as long as it is used appropriately as needed to provide care. There is no
need to move the book to a more secluded area.

253
Q

Conjoined twins are in the neonatal department of the community hospital until transfer to the
closest medical center. A photographer from the local newspaper gets off the elevator on the neonatal
floor and wants to take pictures of the infants. Which initial action should the nurse take?
a. Escort the cameraman to the neonatal unit while a few pictures are taken quietly.
b. Tell the cameraman where the hospital’s public relations department is located.
c. Have the cameraman wait for permission from the health care provider.
d. Ask the cameraman how the pictures are to be used in the newspaper

A

ANS: B
In some cases, information about a scientific discovery or a major medical breakthrough or an
unusual situation is newsworthy. In this case, anyone seeking information needs to contact the
hospital’s public relations department to ensure that invasion of privacy does not occur. It is not the
nurse’s responsibility to decide independently the legality of disclosing information. The nurse does
not have the right to allow the cameraman access to the neonatal unit. This would constitute invasion
of privacy. The health care provider has no responsibility regarding this situation and cannot allow
the cameraman on the unit. It is not the nurse’s responsibility to find out how the pictures are to be
used. This is a task for the public relations department.

254
Q
A nurse exchanges information with the oncoming nurse about a patient’s care. Which action did
 the nurse complete?
a. A verbal report
b. An electronic record entry
c. A referral
d. An acuity rating
A

ANS: A
Whether the transfer of patient information occurs through verbal reports, electronic or written
documents, you need to follow some basic principles. Reports are exchanges of information among
caregivers. A patient’s electronic medical record or chart is a confidential, permanent legal
documentation of information relevant to a patient’s health care. Nurses document referrals
(arrangements for the services of another care provider). Nurses use acuity ratings to determine the
hours of care and number of staff required for a given group of patients every shift or every 24 hours.

255
Q

A nurse is auditing and monitoring patients’ health records. Which action is the nursetaking?
a. Determining the degree to which standards of care are met by reviewing patients’ health records
b. Realizing that care not documented in patients’ health records still qualifies as care provided
c. Basing reimbursement upon the diagnosis-related groups documented in patients’ records
d.
Comparing data in patients’ records to determine whether a new treatment had better outcomes than the st
treatmen

A

ANS: A
The auditing and monitoring of patients’ health records involve nurses periodically auditing records
to determine the degree to which standards of care are met and identifying areas needing
improvement and staff development. The mistakes in documentation that commonly result in
malpractice include failing to record nursing actions; this is the aspect of legal documentation. The
financial billing or reimbursement purpose involves diagnosis-related groups (DRGs) as the basis for
establishing reimbursement for patient care. For research purposes, the researcher compares the
patient’s recorded findings to determine whether the new method was more effective than the
standard protocol. Data analysis contributes to evidence-based nursing practice and quality health
care.

256
Q

A preceptor is working with a new nurse on documentation. Which situation will cause the
preceptor to follow up?
a. The new nurse documents only for self.
b. The new nurse charts consecutively on every other line.
c. The new nurse ends each entry with signature and title.
d. The new nurse keeps the password secure.

A

ANS: B
Chart consecutively, line by line (not every other line); every other line is incorrect and must be
corrected by the preceptor. If space is left, draw a line horizontally through it, and place your
signature and credentials at the end. Every other line should not be left blank. All the other behaviors
are correct and need no follow-up. Documenting only for yourself is an appropriate behavior. End
each entry with signature and title/credentials. For computer documentation, keep your password to
yourself.

257
Q

A nurse wants to find all the pertinent patient information in one record, regardless of the number
of times the patient entered the health care system. Which record should the nursefind?
a. Electronic medical record
b. Electronic health record
c. Electronic charting record
d. Electronic problem record

A

ANS: B
The term electronic health record/EHR is increasingly used to refer to a longitudinal (lifetime)
record of all health care encounters for an individual patient by linking all patient data from previous
health encounters. An electronic medical record (EMR) is the legal record that describes a single
encounter or visit created in hospitals and outpatient health care settings that is the source of data for
the EHR. There are no such terms as electronic charting record or electronic problem record that
record the lifetime information of a patient.

258
Q

A nurse wants to find the daily weights of a patient. Which form will the nurse use?

a. Database
b. Progress notes
c. Patient care summary
d. Graphic record and flow sheet

A

ANS: D
Within a computerized documentation system, flow sheets and graphic records allow you to quickly
and easily enter assessment data about a patient, such as vital signs, admission and or daily weights,
and percentage of meals eaten. In the problem-oriented medical record, the database section contains
all available assessment information pertaining to the patient (e.g., history and physical examination,
nursing admission history and ongoing assessment, physical therapy assessment, laboratory reports,
and radiologic test results). Many computerized documentation systems have the ability to generate a
patient care summary document that you review and sometimes print for each patient at the
beginning and/or end of each shift; it includes information such as basic demographic data, health
care provider’s name, primary medical diagnosis, and current orders. Health care team members
monitor and record the progress made toward resolving a patient’s problems in progress notes.

259
Q

A home health nurse is preparing for an initial home visit. Which information should be included
in the patient’s home care medical record?
a. Nursing process form
b. Step-by-step skills manual
c. A list of possible procedures
d. Reports to third-party payers

A

ANS: D
Information in the home care medical record includes patient assessment, referral and intake forms,
interprofessional plan of care, a list of medications, and reports to third-party payers. An
interprofessional plan of care is used rather than a nursing process form. A step-by-step skills manual
and a list of possible procedures are not included in the record.

260
Q

. A nurse in a long-term care setting that is funded by Medicare and Medicaid is completing
standardized protocols for assessment and care planning for reimbursement. Which task is the nurse
completing?
a. A minimum data set
b. An admission assessment and acuity level
c. A focused assessment/specific body system
d. An intake assessment form and auditing phase

A

ANS: A
The Resident Assessment Instrument (RAI), which includes the Minimum Data Set (MDS) and the
Care Area Assessment (CAA), is the data set that is federally mandated for use in long-term care
facilities by CMS. MDS assessment forms are completed upon admission, and then periodically,
within specific guidelines and time frames for all residents in certified nursing homes. The MDS also
determines the reimbursement level under the prospective payment system. A focused assessment is
limited to a specific body system. An admission assessment and acuity level is performed in the
hospital. An intake form is for home health. There is no such thing as an auditing phase in an
assessment intake.

261
Q

A patient has a diagnosis of pneumonia. Which entry should the nurse chart to help with financial
reimbursement?
a.
Used incentive spirometer to encourage coughing and deep breathing. Lung congested upon auscultation
lobes bilaterally. Pulse oximetry 86%. Oxygen per nasal cannula applied at 2 L/min per standing order.
b.
Cooperative, patient coughed and deep breathed using a pillow as a splint. Stated, “felt better.” Finally, pa
no complaints.
c. Breathing without difficulty. Sitting up in bed watching TV. Had a good day.
d. Status unchanged. Remains stable with no abnormal findings. Checked every 2 hours.

A

ANS: A
Accurately documenting services provided, including the supplies and equipment used in a patient’s
care, clarifies the type of treatment a patient received. This documentation also supports accurate and
timely reimbursement to a health care agency and/or patient. None of the other options had
equipment or supplies listed. Avoid using generalized, empty phrases such as “status unchanged” or
“had good day.” Do not enter personal opinions—stating that the patient is cooperative is a personal
opinion and should be avoided. “Finally, patient had no complaints” is a critical comment about the
patient and if charted can be used as evidence of nonprofessional behavior or poor quality of care.

262
Q

A nurse is teaching the staff about health care reimbursement. Which information should the
nurse include in the teaching session?
a. Home health, long-term care, and hospital nurses’ documentation can affect reimbursement for health car
b. A clinical information system must be installed by 2014 to obtain health care reimbursement.
c. A “near miss” helps determine reimbursement issues for health care.
d. HIPAA is the basis for establishing reimbursement for health care.

A

ANS: A
Nurses’ documentation practices in home health, long-term care, and hospitals can determine
reimbursement for health care. A “near miss” is an incident where no property was damaged and no
patient or personnel were injured, but given a slight shift in time or position, damage or injury could
have easily occurred. A clinical information system (CIS) does not have to be installed by 2014 to
obtain reimbursement. CIS programs include monitoring systems; order entry systems; and
laboratory, radiology, and pharmacy systems. Diagnosis-related groups (DRGs) are the basis for
establishing reimbursement for patient care, not HIPAA. Legislation to protect patient privacy
regarding health information is the Health Insurance Portability and Accountability Act (HIPAA).

263
Q
  1. Which behaviors indicate the student nurse has a good understanding of confidentiality and the
    Health Insurance Portability and Accountability Act (HIPAA)? (Select all that apply.)
    a. Writes the patient’s room number and date of birth on a paper for school
    b. Prints/copies material from the patient’s health record for a graded care plan
    c. Reviews assigned patient’s record and another unassigned patient’s record
    d. Gives a change-of-shift report to the oncoming nurse about the patient
    e. Reads the progress notes of assigned patient’s record
    f. Discusses patient care with the hospital volunteer
A

ANS: D, E
When you are a student in a clinical setting, confidentiality and compliance with HIPAA are part of
professional practice. Reading the progress notes of an assigned patient’s record and giving a changeof-shift report to the oncoming nurse about the patient are behaviors that follow HIPAA and
confidentiality guidelines. Do not share information with other patients or health care team members
who are not caring for a patient. Not only is it unethical to view medical records of other patients, but
breaches of confidentiality lead to disciplinary action by employers and dismissal from work or
nursing school. To protect patient confidentiality, ensure that written materials used in your student
clinical practice do not include patient identifiers (e.g., room number, date of birth, demographic
information), and never print material from an electronic health record for personal use.

264
Q

A nurse is developing a plan to reduce data entry errors and maintain confidentiality. Which
guidelines should the nurse include? (Select all that apply.)
a. Bypass the firewall.
b. Implement an automatic sign-off.
c. Create a password with just letters.
d. Use a programmed speed-dial key when faxing.
e. Impose disciplinary actions for inappropriate access.
f. Shred papers containing personal health information (PHI).

A

ANS: B, D, E, F
When faxing, use programmed speed-dial keys to eliminate the chance of a dialing error and
misdirected information. An automatic sign-off is a safety mechanism that logs a user off the
computer system after a specified period of inactivity. Disciplinary action, including loss of
employment, occurs when nurses or other health care personnel inappropriately access patient
information. All papers containing PHI (e.g., Social Security number, date of birth or age, patient’s
name or address) must be destroyed immediately after you use or fax them. Most agencies have
shredders or locked receptacles for shredding and incineration. Strong passwords use combinations
of letters, numbers, and symbols that are difficult to guess. A firewall is a combination of hardware
and software that protects private network resources (e.g., the information system of the hospital)
from outside hackers, network damage, and theft or misuse of information and should not be
bypassed.

265
Q

A nurse is developing a plan to reduce data entry errors and maintain confidentiality. Which
guidelines should the nurse include? (Select all that apply.)
a. Bypass the firewall.
b. Implement an automatic sign-off.
c. Create a password with just letters.
d. Use a programmed speed-dial key when faxing.
e. Impose disciplinary actions for inappropriate access.
f. Shred papers containing personal health information (PHI).

A

ANS: B, D, E, F
When faxing, use programmed speed-dial keys to eliminate the chance of a dialing error and
misdirected information. An automatic sign-off is a safety mechanism that logs a user off the
computer system after a specified period of inactivity. Disciplinary action, including loss of
employment, occurs when nurses or other health care personnel inappropriately access patient
information. All papers containing PHI (e.g., Social Security number, date of birth or age, patient’s
name or address) must be destroyed immediately after you use or fax them. Most agencies have
shredders or locked receptacles for shredding and incineration. Strong passwords use combinations
of letters, numbers, and symbols that are difficult to guess. A firewall is a combination of hardware
and software that protects private network resources (e.g., the information system of the hospital)
from outside hackers, network damage, and theft or misuse of information and should not be
bypassed.

266
Q

A nurse is teaching a patient about patient-controlled analgesia (PCA). Which statement made by
the patient indicates to the nurse that teaching is effective?
a. “I will only need to be on this pain medication.”
b. “I feel less anxiety about the possibility of overdosing.”
c. “I can receive the pain medication as frequently as I need to.”
d. “I need the nurse to notify me when it is time for another dose.”

A

ANS: B
A PCA is a device that allows the patient to determine the level of pain relief delivered, reducing the
risk of overdose. The PCA infusion pumps are designed to deliver a specific dose that is programmed
to be available at specific time intervals (usually in the range of 8 to 15 minutes) when the patient
activates the delivery button. A limit on the number of doses per hour or 4-hour interval may also be
set. This can help decrease a patient’s anxiety related to possible overdose. Its use also often eases
anxiety because the patient is not reliant on the nurse for pain relief. Other medications, such as oral
analgesics, can be given in addition to the PCA machine. One benefit of PCA is that the patient does
not need to rely on the nurse to administer pain medication; the patient determines when to take the
medication.

267
Q

A patient who had a motor vehicle crash 2 days ago is experiencing pain and is receiving patientcontrolled analgesia (PCA). Which assessment finding indicates effective pain management with the
PCA?
a. The patient is sleeping and is difficult to arouse.
b. The patient rates pain at a level of 2 on a 0 to 10 scale.
c. The patient has sufficient medication left in the PCA syringe.
d. The patient presses the control button to deliver pain medication.

A

ANS: B
A level of 2 on a scale of 0 to 10 is evidence of effective pain management. The effectiveness of
pain-relief measures is determined by the patient. If the patient is satisfied with the amount of pain
relief, then pain measures are effective. A patient who is sleeping and is difficult to arouse ispossibly
oversedated; the nurse needs to assess this patient further. The amount of medication left in the PCA
syringe does not indicate whether pain management is effective or not. Pressing the button shows
that the patient knows how to use the PCA but does not evaluate pain management.

268
Q

A nurse is caring for a patient with chronic pain. Which statement by the nurse indicates an
understanding of pain management?
a. “This patient says the pain is a 5 but is not acting like it. I am not going to give any pain medication.”
b. “I need to reassess the patient’s pain 1 hour after administering oral pain medication.”
c. “It wasn’t time for the patient’s medication, so when it was requested, I gave a placebo.”
d. “The patient is sleeping, so I pushed the PCA button.”

A

ANS: B
Be sure to evaluate after an appropriate period of time. For instance, oral medications usually peak in
about 1 hour, whereas IVP medications peak in 15 to 30 minutes. Ask a patient if a medication
alleviates the pain when it is peaking. Because oral medications usually peak in about an hour, you
need to reassess the patient’s pain within an hour of administration. Nurses must believe any patient
report of pain, even if nonverbal communication is not consistent with pain ratings. The patient is the
only person who should push the PCA button. Pushing the PCA when a patient is sleeping is
dangerous and may lead to narcotic overdose or respiratory depression. Giving the patient a placebo
and telling the patient it is medication is unethical.

269
Q

The nurse is administering pain medication for several patients. Which patient does the nurse
administer medication to first?
a. The patient who needs to be premedicated before walking
b. The patient who has a PCA running that needs the syringe replaced
c. The patient who needs to take a scheduled dose of maintenance pain medication
d. The patient who is experiencing 8/10 pain and has an immediate order for pain medication

A

ANS: D
Immediate (STAT) medications need to be given as soon as possible. In addition, this patient is the
priority because of the report of severe pain. The other patients need pain medication, but their
situations are not as high a priority as that of the patient with the STAT medication order

270
Q

The nurse is caring for a group of patients. Which task may the nurse delegate to the nursing
assistive personnel (NAP)?
a. Administer a back massage to a patient with pain.
b. Assessment of pain for a patient reporting abdominal pain.
c. Administer patient-controlled analgesia for a postoperative patient.
d. Assessment of vital signs in a patient receiving epidural analgesia

A

ANS: A
A massage may be delegated to an NAP. Pain assessment is a nursing function and cannot be
delegated to an NAP. Administration of patient-controlled analgesia (PCA) cannot be delegated to an
NAP. Assessment of vital signs is a licensed nursing function; the NAP can take vital signs for a
patient receiving epidural analgesia.

271
Q

The nurse is caring for an infant in the intensive care unit. Which information should the nurse
consider when planning care for this patient?
a. Infants cannot be assessed for pain.
b. Infants respond behaviorally and physiologically to painful stimuli.
c. Infants cannot tolerate analgesics owing to an underdeveloped metabolism.
d. Infants have a decreased sensitivity to pain when compared with older children

A

ANS: B
Infants cannot verbally express their pain, but they do express pain with behavioral cues (facial
expressions, crying) and physiological indicators (changes in vital signs). Infants can tolerate
analgesics, but proper dosing and close monitoring are essential. Infants and older children have the
same sensitivity to pain. Pain can be assessed even though the neonate cannot verbalize; the nurse
can observe behavioral clues. Nurses use behavioral cues and physiological responses to assess pain
in infants.

272
Q

A nurse is planning care for an older-adult patient who is experiencing pain. Which statement made
by the nurse indicates the supervising nurse needs to follow up?
a. “As adults age, their ability to perceive pain decreases.”
b. “Older patients may have low serum albumin in their blood, causing toxic effects of analgesic drugs.”
c. “Patients who have dementia probably experience pain, and their pain is not always well controlled.”
d.
“It is safe to administer opioids to older adults as long as you start with small doses and frequently assess
patient’s response to the medication.”

A

ANS: A
Aging does not affect the ability to perceive pain. This misconception must be corrected by the
supervising nurse. All the other statements are true and require no follow-up. Opioids are safe to use
in older adults as long as they are slowly titrated and the nurse frequently monitors the patient.
Patients with dementia most likely experience unrelieved pain because their pain is difficult to assess.
Older adults frequently eat poorly, resulting in low serum albumin levels. Many drugs are highly
protein bound. In the presence of low serum albumin, more free drug (active form) is available, thus
increasing the risk for side and/or toxic effects.

273
Q

A nurse is caring for a patient who is experiencing pain following abdominal surgery. Which
information is important for the nurse to share with the patient when providing patient education
about effective pain management?
a. “To prevent overdose, you need to wait to ask for pain medication until you begin to experience pain.”
b. “You should take your medication after you walk to make sure you do not fall while you are walking.”
c. “We should work together to create a schedule to provide regular dosing of medication.”
d. “When you experience severe pain, you will need to take oral pain medications.”

A

ANS: C
One way to maximize pain relief while potentially decreasing opioid use is to administer analgesics
around the clock (ATC) rather than on a prn basis. This approach ensures a more constant therapeutic
blood level of an analgesic. Working with the patient to design a schedule allows the patient to be a
full partner in the care provided. The nurse should not wait until pain is experienced because it takes
medications 10-30 minutes to begin to relieve pain. The nurse administers pain medications before
painful activities, such as walking, and administers intravenous medications when a patient is having
severe pain.

274
Q

The nurse is caring for a surgical patient, when the family member asks what perioperative nursing

means. How should the nurse respond?
a. Perioperative nursing occurs in preadmission testing.
b. Perioperative nursing occurs primarily in the postanesthetic care unit.
c. Perioperative nursing includes activities before, during, and after surgery.
d. Perioperative nursing includes activities only during the surgical procedure.

A

ANS: C
Perioperative nursing care occurs before, during, and after surgery. Preadmission testing occurs
before surgery and is considered preoperative. Nursing care provided during the surgical procedure is
considered intraoperative, and in the postanesthetic care unit, it is considered postoperative. All of
these are parts of the perioperative phase, but each individual phase does not explain the term
completely.

275
Q

The nurse is caring for a patient who is scheduled to undergo a surgical procedure. The nurse is
completing an assessment and reviews the patient’s laboratory tests and allergies and prepares the
patient for surgery. In which perioperative nursing phase is the nurse working?
a. Perioperative
b. Preoperative
c. Intraoperative
d. Postoperative

A

ANS: B
Reviewing the patient’s laboratory tests and allergies is done before surgery in the preoperative
phase. Perioperative means before, during, and after surgery. Intraoperative means during the
surgical procedure in the operating suite; postoperative means after the surgery and could occur in
the postanesthesia care unit, in the ambulatory surgical area, or on the hospital unit.

276
Q

The nurse is caring for a patient in the postanesthesia care unit. The patient has developed profuse
bleeding from the surgical site, and the surgeon has determined the need to return to theoperative
area. How will the nurse classify this procedure?
a. Major
b. Urgent
c. Elective
d. Emergency

A

ANS: D
An emergency procedure must be done immediately to save a life or preserve the function of a body
part. An example would be repair of a perforated appendix, repair of a traumatic amputation, or
control of internal hemorrhaging. An urgent procedure is necessary for a patient’s health and often
prevents additional problems from developing. An example would be excision of a cancerous tumor,
removal of a gallbladder for stones, or vascular repair for an obstructed artery. An elective procedure
is performed on the basis of the patient’s choice; it is not essential and is not always necessary for
health. An example would be a bunionectomy, plastic surgery, or hernia reconstruction. A major
procedure involves extensive reconstruction or alteration in body parts; it poses great risks to wellbeing. An example would be a coronary artery bypass or colon resection.

277
Q

The nurse is caring for a patient in the postanesthetic care unit who has under gone a left total knee
arthroplasty. The anesthesia provider has indicated that the patient received a left femoral peripheral
nerve block. Which assessment will be an expected finding for this patient?
a. Sensation decreased in the left leg
b. Patient report of pain in the left foot
c. Pulse decreased at the left posterior tibia
d. Left toes cool to touch and slightly cyanotic

A

ANS: A
Induction of regional anesthesia results in loss of sensation in an area of the body—in this case, the
left leg. The peripheral nerve block influences the portions of sensory pathways that are anesthetized
in the targeted area of the body. Decreased pulse, toes cool to touch, and cyanosis are indications of
decreased blood flow and are not expected findings. Reports of pain in the left foot may indicate that
the block is not working or is subsiding and is not an expected finding in the immediate postoperative
period.

278
Q

The nurse is preparing a patient for surgery. Which goal is a priority for assessing the patient
before surgery?
a. Plan for care after the procedure.
b. Establish a patient’s baseline of normal function.
c. Educate the patient and family about the procedure.
d. Gather appropriate equipment for the patient’s needs

A

ANS: B
The goal of the preoperative assessment is to identify a patient’s normal preoperative function and
the presence of any risks to recognize, prevent, and minimize possible postoperative complications.
Gathering appropriate equipment, planning care, and educating the patient and family are all
important interventions that must be provided for the surgical patient; they are part of the nursing
process but are not the priority reason/goal for completing an assessment of the surgical patient.

279
Q

The nurse is encouraging the postoperative patient to utilize diaphragmatic breathing.
Which priority goal is the nurse trying to achieve?
a. Manage pain
b. Prevent atelectasis
c. Reduce healing time
d. Decrease thrombus formation

A

ANS: B
After surgery, patients may have reduced lung volume and may require greater effort to cough and
deep breathe; inadequate lung expansion can lead to atelectasis and pneumonia. Purposely utilizing
diaphragmatic breathing can decrease this risk. During general anesthesia, the lungs are not fully
inflated during surgery and the cough reflex is suppressed, so mucus collects within airway passages.
Diaphragmatic breathing does not manage pain; in some cases, if splinting and pain medications are
not given, it can cause pain. Diaphragmatic breathing does not reduce healing time or decrease
thrombus formation. Better, more effective interventions are available for these situations.

280
Q
  1. The nurse is caring for a postoperative patient on the medical-surgical floor. Which activity will
    the nurse encourage to prevent venous stasis and the formation ofthrombus?
    a. Diaphragmatic breathing
    b. Incentive spirometry
    c. Leg exercises
    d. Coughing
A

ANS: C
After general anesthesia, circulation slows, and when the rate of blood slows, a greater tendency for
clot formation is noted. Immobilization results in decreased muscular contractions in the lower
extremities; these promote venous stasis. Coughing, diaphragmatic breathing, and incentive
spirometry are utilized to decrease atelectasis and pneumonia.

281
Q

The nurse is caring for a preoperative patient. The nurse teaches the principles and demonstrates
leg exercises for the patient. The patient is unable to perform leg exercises correctly. What is the
nurse’s best next step?
a. Encourage the patient to practice at a later date.
b. Assess for the presence of anxiety, pain, or fatigue.
c. Ask the patient why exercises are not being done.
d. Evaluate the educational methods used to educate the patient.

A

ANS: B
If the patient is unable to perform leg exercises, the nurse should look for circumstances that may be
impacting the patient’s ability to learn. In this case, the patient can be anticipating the upcoming
surgery and may be experiencing anxiety. The patient may also be in pain or may be fatigued; bothof
these can affect the ability to learn. Evaluation of educational methods may be needed, but in this
case, principles and demonstrations are being utilized. Asking anyone “why” can cause
defensiveness and may not help in attaining the answer. The nurse is aware that the patient is unable
to do the exercises. Moving forward without ascertaining that learning has occurred will not help the
patient in meeting goals

282
Q

Which nursing assessment will indicate the patient is performing diaphragmatic breathing
correctly?
a. Hands placed on the border of the rib cage with fingers extended will touch as the chest wall contracts.
b. Hands placed on the chest wall with fingers extended will separate as the chest wall contracts.
c. The patient will feel upward movement of the diaphragm during inspiration.
d. The patient will feel downward movement of the diaphragm during expiration.

A

ANS: A
Positioning the hands along the borders of the rib cage allows the patient to feel movement of the
chest and abdomen as the diaphragm descends and the lungs expand. As the patient takes a deep
breath and slowly exhales, the middle fingers will touch while the chest wall contracts. The fingers
will separate as the chest wall expands. The patient will feel normal downward movement of the
diaphragm during inspiration and normal upward movement during expiration.

283
Q

The nurse is caring for a postoperative patient with an abdominal incision. The nurse provides a
pillow to use during coughing. Which activity is the nurse promoting?
a. Pain relief
b. Splinting
c. Distraction
d. Anxiety reduction

A

ANS: B
Deep breathing and coughing exercises place additional stress on the suture line and cause
discomfort. Splinting incisions with hands and a pillow provides firm support and reduces incisional
pull. Providing a pillow during coughing does not provide distraction or reduce anxiety. Providing a
pillow does not provide pain relief. Coughing can increase anxiety because it can cause pain.
Analgesics provide pain relief

284
Q

The nurse is encouraging a reluctant postoperative patient to deep breathe and cough. Which
explanation can the nurse provide that may encourage the patient to comply?
a. “If you don’t deep breathe and cough, you will get pneumonia.”
b. “You will need to cough only a few times during this shift.”
c. “Let’s try clearing the throat because that will work just as well.”
d. “Deep breathing and coughing will clear out the anesthesia.”

A

ANS: D
Deep breathing and coughing expel retained anesthetic gases and facilitate a patient’s return to
consciousness. Although it is correct that a patient may experience atelectasis and pneumonia if deep
breathing and coughing are not performed, the way this is worded sounds threatening and could be
communicated in a more therapeutic manner. Deep breathing and coughing are encouraged every 2
hours while the patient is awake. Just clearing the throat does not remove mucus from deeper
airways.

285
Q

The nurse is providing preoperative teaching for the ambulatory surgery patient who will be
having a cyst removed from the right arm. Which will be the best explanation for diet progression
after surgery?
a. “Start with clear liquids, soup, and crackers. Advance to a normal diet as tolerated.”
b. “Stay with ice chips for several hours. After that, you can have whatever you want.”
c. “Stay on clear liquids for 24 hours. Then you can progress to a normal diet.”
d. “Start with clear liquids for 2 hours and then full liquids for 2 hours. Then progress to a normal diet.

A

ANS: A
Patients usually receive a normal diet the first evening after surgery unless they have undergone
surgery on GI structures. Implement diet intake while judging the patient’s response. For example,
provide clear liquids such as water, apple juice, broth, or tea after nausea subsides. If the patient
tolerates liquids without nausea, advance the diet as ordered. There is no need to stay on ice chips for
several hours or clear liquids for 24 hours after this procedure. Putting a time frame on the
progression is too prescriptive. Progression should be adjusted for the patient’s needs.

286
Q

The nurse explains the pain relief measures available after surgery during preoperative teaching
for a surgical patient. Which comment from the patient indicates the need for additional educationon
this topic?
a. “I will be asked to rate my pain on a pain scale.”
b. “I will have minimal pain because of the anesthesia.”
c. “I will take the pain medication as the provider prescribes it.”
d. “I will take my pain medications before doing postoperative exercises.”

A

ANS: B
Anesthesia will be provided during the procedure itself, and the patient should not experience pain
during the procedure; however, this will not minimize the pain after surgery. Pain management is
utilized after the postoperative phase. Inform the patient of interventions available for pain relief,
including medication, relaxation, and distraction. The patient needs to know and understand how to
take the medications that the health care provider will prescribe postoperatively. During the stay in
the facility, the level of pain is frequently assessed by the nurses. Coordinating pain medication with
postoperative exercises helps to minimize discomfort and allows the exercises to be more effective.

287
Q

The nurse is making a preoperative education appointment with a patient. The patient asks if a
family member should come to the appointment. Which is the best response by the nurse?
a. “There is no need for an additional person at the appointment.”
b. “Your family can come and wait with you in the waiting room.”
c. “We recommend including family members at this appointment.”
d. “It is required that you have a family member at this appointment

A

ANS: C
Including family members in perioperative education is advisable. Often a family member is a coach
for postoperative exercises when the patient returns from surgery. If anxious relatives do not
understand routine postoperative events, it is likely that their anxiety will heighten the patient’s fears
and concerns. Preoperative preparation of family members before surgery helps to minimize anxiety
and misunderstanding. An additional person is needed at the appointment if at all possible, and he or
she needs to be involved in the process, not just waiting in the waiting room; however, it is certainly
not a requirement for actually completing the surgery that someone comes to this appointment.

288
Q

The nurse is reviewing the surgical consent with the patient during preoperative educationand
finds the patient does not understand what procedure will be completed. What is the nurse’s
best next step?
a. Notify the health care provider about the patient’s question.
b. Explain the procedure that will be completed.
c. Continue with preoperative education.
d. Ask the patient to sign the form

A

ANS: A
Surgery cannot be legally or ethically performed until the patient fully understands the need for a
procedure and all the implications. It is the surgeon’s responsibility to explain the procedure,
associated risks, benefits, alternatives, and possible complications. It is important for the nurse to
pause with preoperative education to notify the health care provider of the patient’s questions. It is
not within the nurse’s scope to explain the procedure. The nurse can certainly reinforce what the
health care provider has explained, but the information needs to come from the health care provider.
It is not prudent to ask a patient to sign a form for a procedure that he/she does not understand.

289
Q

The nurse has administered a preoperative medication to the patient going to surgery. Which
action will the nurse take next?
a. Notify the operating suite that the medication has been given.
b. Instruct the patient to call for help to go to the restroom.
c. Waste any unused medication according to policy.
d. Ask the patient to sign the consent for surgery.

A

ANS: B
Once a preoperative medication has been administered, instruct the patient to call for help when
getting out of bed to prevent falls. For patient safety, explain the purpose of a preoperative
medication and its effects. Notifying the operating suite that the medication has been given may be
part of a facilities procedure but is not the best next step. It is important to have the patient sign
consents before the patient has received medication that may make him/her drowsy. Wasting unused
medication according to policy is important but is not the best next step.

290
Q

The nurse has completed a preoperative assessment for a patient going to surgery andgathers
assessment data. Which will be the most important next step for the nurse to take?
a. Notify the operating suite that the patient has a latex allergy.
b. Document that the patient had a bath at home this morning.
c. Administer the ordered preoperative intravenous antibiotic.
d. Ask the nursing assistive personnel to obtain vital signs.

A

ANS: A
The most important step is notifying the operating suite of the patient’s latex allergy. Many products
that contain latex are used in the operating suite and the postanesthesia care unit (PACU). When
preparing for a patient with this allergy, special considerations are required from preparation of the
room to the types of tubes, gloves, drapes, and instruments utilized. Obtaining vital signs,
documenting, and administering medications are all part of the process and should be done—with the
latex allergy in mind. However, making sure that the patient has a safe environment is the first step.

291
Q

The nurse is caring for a patient in the preoperative holding area of an ambulatory surgerycenter.
Which nursing action will be most appropriate for this area?
a. Count the sterile surgical instruments.
b. Empty the urinary drainage bag.
c. Check the surgical dressing.
d. Apply a warm blanket.

A

ANS: D
The temperature in the preoperative holding area and in adjacent operating suites is usually cold.
Offer the patient an extra warm blanket. Counts are taken by the circulating and scrub nurses in the
operating room. Emptying a urinary drainage bag and checking the surgical dressing occur in the
postanesthesia care unit, not in the holding area.

292
Q

ANS: D
The temperature in the preoperative holding area and in adjacent operating suites is usually cold.
Offer the patient an extra warm blanket. Counts are taken by the circulating and scrub nurses in the
operating room. Emptying a urinary drainage bag and checking the surgical dressing occur in the
postanesthesia care unit, not in the holding area.

A

ANS: D
The temperature in the preoperative holding area and in adjacent operating suites is usually cold.
Offer the patient an extra warm blanket. Counts are taken by the circulating and scrub nurses in the
operating room. Emptying a urinary drainage bag and checking the surgical dressing occur in the
postanesthesia care unit, not in the holding area.

293
Q

The nurse is concerned about the skin integrity of the patient in the intraoperative phase of

surgery. Which action will the nurse take to minimize skin breakdown?
a. Encouraging the patient to bathe before surgery
b. Securing attachments to the operating table with foam padding
c. Periodically adjusting the patient during the surgical procedure
d. Measuring the time a patient is in one position during surgery

A

ANS: B
Although it may be necessary to place a patient in an unusual position, try to maintain correct
alignment and protect the patient from pressure, abrasion, and other injuries. Special mattresses, use
of foam padding, and attachments to the operating suite table provide protection for the extremities
and bony prominences. Bathing before surgery helps to decrease the number of microbes on the skin.
Periodically adjusting the patient during the surgical procedure is impractical and can present a safety
issue with regard to maintaining sterility of the field and maintaining an airway. Measuring the time
the patient is in one position may help with monitoring the situation but does not prevent skin
breakdown.

294
Q

The nurse is assessing a postoperative patient with a history of obstructive sleep apnea for airway

obstruction. Which assessment finding will best alert the nurse to this complication?
a. Drop in pulse oximetry readings
b. Moaning with reports of pain
c. Shallow respirations
d. Disorientation

A

ANS: A
One of the greatest concerns after general anesthesia is airway obstruction, especially in patients with
obstructive sleep apnea. A drop in oxygen saturation by pulse oximetry is a sign of airway
obstruction in patients with obstructive sleep apnea. Weak pharyngeal/laryngeal muscle tone from
anesthetics; secretions in the pharynx, bronchial tree, or trachea; and laryngeal or subglottic edema
also contribute to airway obstruction. In the postanesthetic patient, the tongue is a major cause of
airway obstruction. Shallow respirations are indicative of respiratory depression. Moaning and
reports of pain are common in all surgical patients and are an expected event. Disorientation is
common when first awakening from anesthesia but can be a sign of hypoxia.

295
Q

The nurse is caring for a postoperative patient who has had a minimally invasive carpeltunnel
repair. The patient has a temperature of 97° F and is shivering. Which reason will the
nurse most likely consider as the primary cause when planning care?
a. Anesthesia lowers metabolism.
b. Surgical suites have air currents.
c. The patient is dressed only in a gown.
d. The large open body cavity contributed to heat loss.

A

ANS: A
The operating suite and recovery room environments are extremely cool. The patient’s anesthetically
depressed level of body function results in lowering of metabolism and a fall in body temperature.
Although the patient is dressed in a gown and there are air currents in the operating room, these are
not the primary reasons for the low temperature. Also, the patient in this type of case does not have a
large open body cavity to contribute to heat loss.

296
Q

The nurse is monitoring a patient in the postanesthesia care unit (PACU) for postoperative fluid
and electrolyte imbalance. Which action will be most appropriate for the nurse to take?
a. Encourage copious amounts of water.
b. Start an additional intravenous (IV) line.
c. Measure and record all intake and output.
d. Weigh the patient and compare with preoperative weight.

A

ANS: C
Accurate recording of intake and output assesses renal and circulatory function. Measure and record
all sources of intake and output. Encouraging copious amounts of water in a postoperative patient
might encourage nausea and vomiting. In the PACU, it is impractical to weigh the patient while
waking from surgery, but in the days afterward, it is a good assessment parameter for fluid
imbalance. Starting an additional IV is not necessary and is not important at this juncture.

297
Q

. The nurse is caring for a patient in the postanesthesia care unit. The patient asks for a bedpan and
states to the nurse, “I feel like I need to go to the bathroom, but I can’t.” Which nursingintervention
will be most appropriate initially?
a. Assess the patient for bladder distention.
b. Encourage the patient to wait a minute and try again.
c. Inform the patient that everyone feels this way after surgery.
d. Call the health care provider to obtain an order for catheterization.

A

ANS: A
Depending on the surgery, some patients do not regain voluntary control over urinary function for 6
to 8 hours after anesthesia. Palpate the lower abdomen just above the symphysis pubis for bladder
distention. Another option is to use a bladder scan or
ultrasound to assess bladder volume. The nurse must assess before deciding if the patient can try
again. Not everyone feels as if they need to go but can’t after surgery. Calling the health care
provider is not the initial best action. The nurse needs to have data before calling the provider.

298
Q

The postanesthesia care unit (PACU) nurse transports the inpatient surgical patient to the
medical-surgical floor. Before leaving the floor, the medical-surgical nurse obtains a complete setof
vital signs. What is the rationale for this nursing action?
a. This is done to complete the first action in a head-to-toe assessment.
b. This is done to compare and monitor for vital sign variation during transport.
c. This is done to ensure that the medical-surgical nurse checks on the postoperative patient.
d. This is done to follow hospital policy and procedure for care of the surgical patient.

A

ANS: B
Before the PACU nurse leaves the acute care area, the staff nurse assuming care for the patient takes
a complete set of vital signs to compare with PACU findings. Minor vital sign variations normally
occur after transporting the patient. The PACU nurse reviews the patient’s information with the
medical-surgical nurse, including the surgical and PACU course, physician orders, and the patient’s
condition. While vital signs may or may not be the first action in a head-to-toe assessment, this is not
the rationale for this situation. While following policy or ascertaining that the floor nurse checks on
the patient are good reasons for safe care, they are not the best rationale for obtaining vital signs.

299
Q

The nurse is caring for a group of patients. Which patient will the nurse see first?
a. A patient who had cataract surgery is coughing.
b. A patient who had vascular repair of the right leg is not doing right leg exercises.
c. A patient after knee surgery is wearing intermittent pneumatic compression devices and receiving heparin
d.
A patient after surgery has vital signs taken every 15 minutes twice, every 30 minutes twice, hourly for 2
then every 4 hours.

A

ANS: A
For patients who have had eye, intracranial, or spinal surgery, coughing may be contraindicated
because of the potential increase in intraocular or intracranial pressure. The nurse will need to see
this patient first to control the cough and intraocular pressure. All the rest are normal postoperative
patients. Leg exercise should not be performed on the operative leg with vascular surgery. A patient
after knee surgery should receive heparin and be wearing intermittent pneumatic compression
devices; while the nurse will check on the patient, it does not have to be first. Monitoring vital signs
after surgery is required and this is the standard schedule.

300
Q

The nurse is participating in a “time-out.” In which activities will the nurse be involved? (Select all
that apply.)
a. Verify the correct site.
b. Verify the correct patient.
c. Verify the correct procedure.
d. Perform “time-out” after surgery.
e. Perform the actual marking of the operative site.

A

ANS: A, B, C
A time-out is performed just before starting the procedure for final verification of the correct patient,
procedure, site, and any implants. The marking and time-out most commonly occur in the holding
area, just before the patient enters the OR. The individual performing surgery and who is accountable
for it must personally mark the site, and the patient must be involved if possible.

301
Q

. The nurse is preparing to assist the patient in using the incentive spirometer. Which nursing
intervention should the nurse provide first?
a. Perform hand hygiene.
b. Explain use of the mouthpiece.
c. Instruct the patient to inhale slowly.
d. Place in the reverse Trendelenburg position.

A

ANS: A
Performing hand hygiene reduces microorganisms and should be performed first. Placing the patient
in the correct position such as high Fowler’s for the typical postoperative patient or reverse
Trendelenburg for the bariatric patient would be the next step in the process. Demonstration of use of
the mouthpiece followed by the instruction to inhale slowly would be the last step in this scenario.

302
Q

A nurse encounters a family who experienced the death of their adult child last year. The parents
are talking about the upcoming anniversary of their child’s death. The nurse spends time with them
discussing their child’s life and death. Which nursing principle does the nurse’s
action best demonstrate?
a. Facilitation of normal mourning
b. Pain-management technique
c. Grief evaluation
d. Palliative care

A

ANS: A
Anniversary reactions can reopen grief processes. A nurse should openly acknowledge the loss and
talk about the common renewal of grief feeling around the anniversary of the individual’s death; this
facilitates normal mourning. The nurse is not attempting to alleviate a physical pain. The actions are
of open communication, not evaluation. Palliative care refers to comfort measures for symptom
relief.

303
Q

A cancer patient asks the nurse what the criteria are for hospice care. Which information should
the nurse share with the patient?
a. It is for those needing assistance with pain management.
b. It is for those having a terminal illness, such as cancer.
c. It is for those with completion of an advance directive.
d. It is for those expected to live less than 6 months.

A

ANS: D
Patients accepted into a hospice program usually have less than 6 months to live. Patients with a
terminal illness are not eligible until that point. Palliative care provides assistance with pain
management when a patient is not eligible for hospice care. An advance directive can be completed
by any person, even those who are healthy.

304
Q

In preparation for the eventual death of a female hospice patient of the Muslim faith, the nurse
organizes a meeting of all hospice caregivers. A plan of care to be followed when this patient diesis
prepared. Which information will be included in the plan?
a. Prepare the body for autopsy.
b. Prepare the body for cremation.
c. Allow male Muslims to care for the body after death has occurred.
d. Allow female Muslims to care for the body after death has occurred.

A

ANS: D
Muslims of the same gender prepare the body for burial. Muslim faith discourages cremation and
autopsy to preserve the sanctity of the soul of the deceased and promote burial as soon as possible
after death.

305
Q

Family members gather in the emergency department after learning that a family member was
involved in a motor vehicle accident. After learning of the family member’s unexpected death, the
surviving family members begin to cry and scream in despair. Which phase does the nurse determine
the family is in according to the AttachmentTheory?
a. Numbing
b. Reorganization
c. Yearning and searching
d. Disorganization and despair

A

ANS: C
Yearning and searching characterize the second bereavement phase in the Attachment Theory.
Emotional outbursts of tearful sobbing and acute distress are common in this phase. During the
numbing phase, the family is protected from the full impact of the loss. During disorganization and
despair, the reason why the loss occurred is constantly examined. Reorganization is the last stage of
the Attachment Theory in which the person accepts the change and builds new relationships

306
Q

An Orthodox Jewish rabbi has been pronounced dead. The nursing assistive personnel respectfully
ask family members to leave the room and go home as postmortem care is provided.Which
statement from the supervising nurse is best?
a. “I should have called a male colleague to handle the body.”
b. “Family members stay with the body until burial the next day.”
c. “I wish they would go home because we have work to do here.”
d. “Family will quietly leave after praying and touching the rabbi’s head.”

A

ANS: B
Jewish culture calls for family members or religious officials to stay with the body until the time of
burial. A male provider is unnecessary. Requesting or expecting the family to go home is not
providing culturally sensitive care. Hindus and Muslims usually have persons of the same gender
handle the body after death. Buddhists often say prayers while touching and standing at the head of
the deceased.

307
Q

A palliative team is caring for a dying patient in severe pain. Which action is thepriority?

a. Provide postmortem care for the patient.
b. Support the patient’s nurse in grieving.
c. Teach the patient the stages of grief.
d. Enhance the patient’s quality of life.

A

ANS: D
The primary goal of palliative care is to help patients and families achieve the best quality of life.
Providing support for the patient’s nurse is not the primary obligation when the patient is
experiencing severe pain. Not all collaborative team members on the palliative team would be able to
provide postmortem care, as is the case for nutritionists, social workers, and pharmacists. Teaching
about stages of grief should not be the focus when severe pain is present.

308
Q

A veteran is hospitalized after surgical amputation of both lower extremities owing to injuries
sustained during military service. Which type of loss will the nurse focus the plan of care on forthis
patient?
a. Perceived loss
b. Situational loss
c. Maturational loss
d. Uncomplicated loss

A

ANS: B
Loss of a body part from injury is a situational loss. Maturational losses occur as part of normal life
transitions across the life span. A perceived loss is uniquely defined by the person experiencing the
loss and is less obvious to other people. Uncomplicated loss is not a type of loss; it is a description of
normal grief.

309
Q

“I know it seems strange, but I feel guilty being pregnant after the death of myson last year,” said
a woman during her routine obstetrical examination. The nurse spends extra time with this woman,
helping her realize bonding with this unborn child will not mean she is replacing the one who died.
Which nursing technique does this demonstrate?
a. Providing curative therapy
b. Promoting spirituality
c. Facilitating mourning
d. Eradicating grief

A

ANS: C
The nurse facilitates mourning in family members who are still surviving. By acknowledging the
pregnant woman’s emotions, the nurse helps the mother bond with her fetus and recognizes the
emotions that still exist for the deceased child. The nurse is not attempting to help the patient
eradicate grief, which would be unrealistic. Curative therapy (curing a disease) and spiritual
promotion (belief in a higher power or in the meaning of life) are not addressed by the nurse’s
statement.

310
Q

A patient has had two family members die during the past 2 days. Which coping strategy
is most appropriate for the nurse to suggest to the patient?
a. Writing in a journal
b. Drinking alcohol to go to sleep
c. Exercising vigorously rather than sleeping
d. Avoiding talking with friends and family members

A

ANS: A
Coping strategies may be healthy and effective like talking, journaling, and sharing emotions with
others. They may also be unhealthy and ineffective like increased use of alcohol, drugs, andviolence.
Although exercise is important for self-care, sleep is also important. Shutting oneself away from
friends and family by not talking about the sadness is not effective; the patient should spend time
with people who are nurturing.

311
Q

A female nurse is called into the supervisor’s office regarding her deteriorating work
performance since the loss of her spouse 2 years ago. The woman begins sobbing and says that she is
“falling apart” at home as well. Which type of grief is the female nurse experiencing?
a. Normal grief
b. Perceived grief
c. Complicated grief
d. Disenfranchised grief

A

ANS: C
In complicated grief, a person has a prolonged or significantly difficult time moving forward after a
loss. Normal grief is the most common reaction to death; it involves a complex range of normal
coping strategies. Disenfranchised grief involves a relationship that is not socially sanctioned.
Perceived grief is not a type of grief; perceived loss is a loss that is less obvious to other people.

312
Q

The mother of a child who died recently keeps the child’s room intact. Family members are
encouraging her to redecorate and move forward in life. Which type of grief will the homehealth
nurse recognize the mother is experiencing?
a. Normal
b. End-of-life
c. Abnormal
d. Complicated

A

ANS: A
Family members will grieve differently. One sign of normal grief is keeping the deceased
individual’s room intact as a way to keep that person alive in the minds of survivors. This is
happening after the family member is deceased, so it is not end-of-life grief. It is not abnormal or
complicated grief; the child died recently.

313
Q
A nurse is caring for a dying patient. When is the best time for the nurse to discuss end-of-life
 care?
a. During assessment
b. During planning
c. During implementation
d. During evaluation
A

ANS: A
Because most deaths are now “negotiated” among patients, family members, and the health care
team, discuss end-of-life care preferences early in the assessment phase of the nursing process.Doing
so during the planning, implementation, and evaluation phases is too late.

314
Q

A nurse lets the transplant coordinator make a request for organ and tissue donation from the
patient’s family. What is the primary rationale for the nurse’s action?
a. The nurse is not as knowledgeable as the coordinator.
b. The nurse is uncomfortable asking the family.
c. The nurse does not want to upset the family.
d. The nurse is following a federal law.

A

ANS: D
In accordance with federal law, a specially trained professional (e.g., transplant coordinator or social
worker) makes requests for organ and tissue donation at the time of every death. Given the complex
and sensitive nature of such requests, only specially trained personnel make the requests. Although
the nurse may be less knowledgeable than the coordinator, uncomfortable asking the family, or not
wanting to upset the family, the primary rationale is to be in accordance with federal law.

315
Q

A patient cancels a scheduled appointment because the patient will be attending a Shivah for a
family member. Which response by the nurse is best?
a. “When families come together for end-of-life decisions, it provides connections.”
b. “We will reschedule so the appointment does not fall on the Sabbath.”
c. “Missionary outreach is so important for spiritual comfort.”
d. “I’m so sorry for your loss.”

A

ANS: D
A death has occurred and saying that you are sorry for their loss is appropriate. The Jewish mourning
ritual of Shivah is a time period when normal life activities come to a stop. Those mourningwelcome
friends into the home as a way of honoring the dead and receive support during the mourning period.
Cultural variables can influence a person’s response to grief. It is not when families come together
for end-of-life decisions. It is not because the appointment fell on the Sabbath. It is not about
missionary outreach.

316
Q

During a follow-up visit, a female patient is describing new onset of marital discord withher
terminally ill spouse to the hospice nurse. Which Kübler-Ross stage of dying is the patient
experiencing?
a. Denial
b. Anger
c. Bargaining
d. Depression

A

ANS: B
Kübler-Ross’s traditional theory involves five stages of dying. The anger stage of adjustment to an
impending death can involve resistance, anger at God, anger at people, and anger at the situation.
Denial would involve failure to accept a death. Bargaining is an action to delay acceptance of death
by bartering. Depression would present as withdrawal from others.

317
Q

A patient’s father died a week ago. Both the patient and the patient’s spouse talk about the death.
The patient’s spouse is experiencing headaches and fatigue. The patient is having trouble sleeping,
has no appetite, and gets choked up most of the time. How should the nurse interpret these findings
as the basis for a follow-up assessment?
a. The patient is dying and the spouse is angry.
b. The patient is ill and the spouse is malingering.
c. Both the patient and the spouse are likely in denial.
d. Both the patient and the spouse are likely grieving.

A

ANS: D
Both are likely grieving from the loss of the patient’s father. Symptoms of normal grief include
headache, fatigue, insomnia, appetite disturbance, and choking sensation. Different people manifest
different symptoms. There is no data to support the spouse is angry or malingering. There is no data
to support the patient is dying or ill. Denial is assessed when the person cannot accept the loss; both
talked about the loss

318
Q

A nurse is documenting end-of-life care. Which information will the nurse include in the patient’s
electronic medical record? (Select all that apply.)
a. Reason for the death
b. Time and date of death
c. How ethically the family grieved
d. Location of body identification tags
e. Time of body transfer and destination

A

ANS: B, D, E
Documentation of end-of-life care includes the following: time and date
of death, location of body identification tags, time of body transfer and
destination and personal articles left on and secured to the body. Reason
for the death is not appropriate; this is a medical judgment and not a
nursing judgment. How ethically the family grieved is judgmental and
does not belong in the chart. We must remain open to the varying views
and beliefs of grieving that are in contrast to our own in order to best
support and care for our patients and their families.